SlideShare una empresa de Scribd logo
1 de 44
SỞ GIÁO DỤC VÀ ĐÀO TẠO TỈNH VĨNH PHÚC
PHÒNG GIÁO DỤC VÀ ĐÀO TẠO THÀNH PHỐ VĨNH YÊN
BÁO CÁO CHUYÊN ĐỀ
“NGUYÊN LÍ DIRICHLET
VÀ NGUYÊN LÍ CỰC HẠN
TRONG GIẢI TOÁN TỔ HỢP”
Môn: TOÁN
Tổ bộ môn: KHOA HỌC TỰ NHIÊN
Mã: 30
Người thực hiện: TRẦN THỊ PHI NGA
Điện thoại: 01686187936
Email: ngatran73@gmail.com
Vĩnh Yên, năm 2014
1
2
DANH MỤC CÁC TỪ VIẾT TẮTDANH MỤC CÁC TỪ VIẾT TẮT
Stt Từ viết tắt Từ viết đầy đủ
1 BGH Ban giám hiệu
2 THCS trung học cơ sở
3 BĐT Bất đẳng thức
4 HSG học sinh giỏi
5 GV giáo viên
6 BDHSG Bồi dưỡng học sinh giỏi
7 GTLN Giá trị lớn nhất
8 GTNN Giá trị nhỏ nhất
3
MỤC LỤCMỤC LỤC
I - PHẦN MỞ ĐẦU
1. Lý do chọn đề tài 3
2. Mục đích nghiên cứu đề tài 3
3. Đối tượng và phạm vi nghiên cứu 4
4. Cơ sở lí thuyết và thực trạng của vấn đề 5
II. QUÁ TRÌNH THỰC HIỆN
5. Các bài toán sử dụng nguyên lí cực hạn 8
6. Sử dụng nguyên lí Dirichlet trong giải toán hình học tổ hợp 14
7. Một số dạng toán hình học tổ hợp thường gặp 21
8. Một số dạng bài tổng hợp khác 25
9. Bài tập tổ hợp chọn lọc 33
III – KẾT QUẢ NGHIÊN CỨU 39
III - PHẦN KẾT LUẬN 41
Tài liệu tham khảo 42
4
PHẦN THỨ NHẤT: MỞ ĐẦU
1. Lý do viết đề tài
Trong các kỳ thi học sinh giỏi các cấp tôi thấy rằng học sinh thường mất
điểm khi không giải được các bài tập tổ hợp. Nhiều học sinh cho rằng đó là bài tập
mà các em thường không giải được, do tính chất đặc thù của loại toán mang tính tư
duy và trừu tượng cao. Vì vậy học sinh thường mất nhiều thời gian hoặc không
làm được loại bài này. Qua nhiều năm dạy đội tuyển học sinh giỏi (HSG) tôi rất
trăn trở và suy nghĩ mình phải làm thế nào để học sinh yêu thích giải các bài tập bài
tập tổ hợp hơn. Vì nếu các em có phương pháp giải các bài tập đó một cách thành
thạo thì việc tư duy và thuật toán để giải các loại bài tập khác sẽ nhanh nhẹn hơn,
giúp các em có thể đạt được kết quả cao trong các kỳ thi học sinh giỏi các cấp.
Do vậy tôi mạnh dạn viết chuyên đề “Sử dụng nguyên lí Dirichlet và nguyên
lí cực hạn trong các bài toán tổ hợp”. Nhằm giúp các em có cách nhìn tổng quát và
những suy nghĩ để mở rộng các kiến thức đã học từ những bài toán đơn giản đã
học ở lớp 6. Từ đó các em tự vận dụng và phát triển tư duy với các bài tập tương
tự, tổng quát và liên hệ một cách lô-gic với các dạng toán đã học.
Qua thực tế giảng dạy bồi dưỡng học sinh giỏi nhiều năm, với cách làm trên tôi
thấy rằng học sinh của tôi đã bắt đầu yêu thích các bài tập tổ hợp, các chuyên đề về
bài tập tổ hợp đã lôi cuốn học sinh học tập say mê hơn. Từ đó tôi thấy rằng trong
các kỳ thi học sinh giỏi nếu làm được bài tập tổ hợp là chúng ta có niềm tin rằng
chất lượng đội tuyển sẽ được nâng lên.
1.2. Lý do chủ quan
Đối với các em học sinh, dạng toán tổ hợp (Suy luận lôgic) tuy đã được tiếp
xúc từ chương trình BDHSG của Tiểu học. Xong nó vẫn chỉ là một chương trình
lồng ghép một cách nhẹ nhàng khi BDHSG, tuy đã được học ở các lớp học trước,
nhưng các kỹ năng vận dụng để giải loại bài này của các em vẫn còn chưa đạt hiệu
quả cao.
Trong quá trình giảng dạy và BDHSG ở trường THCS chúng tôi, tôi nhận thấy
dạng toán tổ hợp là một loại bài xuất hiện thường xuyên trong các đề thi HSG ở
mọi lớp học hay cấp học. Tuy nhiên, khi tiếp xúc với dạng bài này HS thường ngại
ngần và khó xuất phát để làm bài. Xuất phát từ thực trạng trên tôi chọn đề tài “Sử
dụng nguyên lí Dirichlet và nguyên lí cực hạn trong các bài toán tổ hợp” cho
chuyên đề của mình.
2. Mục đích nghiên cứu
Trong chuyên đề này trước hết nhằm củng cố cho học sinh lý thuyết về
nguyên lí Dirichlet và nguyên lí cực hạn. Cung cấp cho học sinh một số bài toán cụ
thể và cách tổng quát hóa dạng bài thông qua từng ví dụ.
Giúp cho học sinh có kĩ năng phân loại bài và phương pháp làm từng loại bài
cụ thể ấy.Từ đó rèn cho học sinh tư duy linh hoạt, sáng tạo trong giải toán.
Học sinh thấy được vai trò và ứng dụng rộng rãi của nguyên lí Dirichlet và
nguyên lí cực hạn. Cũng thông qua đề tài này nhằm giúp học sinh có thói quen tìm
tòi trong học toán và sáng tạo khi giải toán.Từ đó tạo cho học sinh có phương pháp
5
học tập đúng đắn, biến cái đã học (kiến thức của thầy) thành cái của bản thân, nắm
bắt nó, vận dụng nó, phát triển nó đúng hướng. Qua đó giúp các em tạo niềm tin,
hưng phấn, hứng thú và say mê học môn toán học.
Sử dụng làm tài liệu tham khảo cho giáo viên và học sinh.
1.4 Đối tượng, phạm vi nghiên cứu:
+) Đối tượng nghiên cứu: Học sinh giỏi lớp 6, 7, 8, 9 và học sinh luyện thi THPT
chuyên.
+) Phạm vi nghiên cứu: Nguyên lí Dirichlet và nguyên lí cực hạn. Các bài tập cơ
bản và nâng cao về nguyên lí Diirchlet và nguyên lí cực hạn trong chương trình
trung học cơ sở.
1.5 Phương pháp nghiên cứu:
+) Phương pháp nghiên cứu lý thuyết
Đọc và nghiên cứu tài liệu, giáo trình về phương pháp dạy học toán, các tài
liệu có liên quan đến nguyên lí Dirichlet và nguyên lí cực hạn cùng ứng dụng của
nó.
+) Phương pháp điều tra
Tìm hiểu thực trạng dạy chuyên đề và bồi dưỡng học sinh giỏi của giáo viên
đồng thời tìm hiểu kết quả học tập của học sinh nhằm xác định tính phổ biến và
nguyên nhân để chuẩn bị cho các bước nghiên cứu tiếp theo.
+) Phương pháp thảo luận
Trao đổi với đồng nghiệp về kinh nghiệm giảng dạy và kĩ thuật vận dụng
nguyên lí Dirichlet và nguyên lí cực hạn .
+) Phương pháp quan sát
Thông qua các tiết dự giờ thao giảng và bồi dưỡng học sinh giỏi của đồng
nghiệp để quan sát trực tiếp tình hình học sinh tiếp thu bài và cách khai thác và xây
dựng các bất đẳng thức phụ của giáo viên.
+) Phương pháp kiểm tra đánh giá
Khi thực hiện chuyên đề khảo sát so sánh kết quả đánh giá học sinh qua
từng giai đoạn để đánh giá hiệu quả của chuyên đề.
1.6 Tình hình nghiên cứu
Trong quá trình giảng dạy bộ môn Toán đặc biệt là trong công tác bồi dưỡng
học sinh giỏi ở trường trung học cơ sở tôi thấy bài toán tổ hợp nói chung và vận
dụng nguyên lí Dirichlet và nguyên lí cực hạn nói riêng là một trong những nội
dung rất quan trọng. Vấn đề này đã có rất nhiều tài liệu tham khảo đề cập đến và
cũng có rất nhiều giáo viên quan tâm nghiên cứu ở những mức độ khác nhau. Kết
quả họ cũng có được những thành công nhất định. Song việc thực hiện được kết
quả như thế nào còn tùy thuộc vào nhiều yếu tố.
1.7 Những vấn đề còn đang tồn tại:
6
Khi chuẩn bị thực hiện chuyên đề này, kĩ năng giải toán tổ hợp của học sinh
còn gặp nhiều khó khăn. Đặc biệt là các bài toán nguyên lí Dirichlet và nguyên lí
cực hạn. Vì thế các em rất thụ động trong các buổi học bồi dưỡng về nội dung này.
Các em học sinh mới chỉ vận dụng được nguyên lí Dirichlet và nguyên lí cực hạn
với những bài toán đơn giản.
Các tài liệu tham khảo về nội dung này mới nêu ra bài toán cụ thể với các bất
ví dụ cụ thể mà chưa có nhiều tài liệu đề cập đến kĩ năng vận dụng nguyên lí
Dirichlet và nguyên lí cực hạn trong giải toán tổ hợp.
1.8 Ứng dụng trong thực tiễn:
Chuyên đề có ứng dụng tốt trong công tác bồi dưỡng học sinh giỏi và công
tác ôn thi vào các trường trung học phổ thông chuyên.
Chuyên đề được phổ biến rộng ở các trường trung học cơ sở trọng điểm
trong Thành phố và trong Tỉnh.
Chuyên đề còn là một tư liệu tốt để giáo viên và học sinh tham khảo.
B. CƠ SỞ LÍ THUYẾT VÀ THỰC TRẠNG CỦA VẤN ĐỀ
2.1 Cơ sở lý luận của vấn đề nghiên cứu:
Khi gặp các bài toán về nguyên lí Dirichlet và nguyên lí cực hạn thường liên
quan nhiều đến các đối tượng là các tập hợp hữu hạn. Vì lẽ đó, các bài toán này
mang mang đặc trưng rõ nét của toán học rời rạc.
Khi giải toán tổ hợp vấn đề cơ bản là xác định được dạng bài và phương
pháp làm cho dạng bài ấy. Từ đó HS áp dụng cho từng bài cụ thể một cách linh
hoạt với các suy luận hợp lý để giải bài toán.
2.2 Thực trạng vấn đề nghiên cứu của chuyên đề.
Trong chương trình toán trung học cơ sở thì nguyên lí Dirichlet và nguyên lí
cực hạn không được học trong chương trình học chính khóa. Tuy nhiên trong các
kỳ thi, đặc biệt là kỳ thi học sinh giỏi các cấp thì nguyên lí Dirichlet và nguyên lí
cực hạn lại được đề cập đến rất nhiều bằng những bài toán hay và khó, đòi hỏi học
sinh phải thực sự linh hoạt, sáng tạo và có kỹ năng sử dụng thành thạo trong suy
luận thì mới gải được loại toán này.
Trong các đề thi HSG, loại bài tổ hợp là rất khó đối với học sinh. Nó không
những khó về biến đổi, khó về suy luận mà còn rất đa dạng về dạng bài và phong
phú về nội dung.
Từ thực tế bồi dưỡng đội tuyển học sinh giỏi, tôi nhận thấy bài toán tổ hợp
mà cụ thể là nguyên lí Dirichlet và nguyên lí cực hạn không những đa dạng về dạng
bài, phong phú về nội dung mà còn là một dạng bài toán khó, luôn gây không ít khó
khăn cho học sinh.
Vậy vấn đề đặt ra ở đây là phải làm sao để tìm ra biện pháp khắc phục thực
trạng trên giúp giáo viên có được một tài liệu tham khảo phù hợp đặc biệt giúp học
sinh hết lúng túng và tự tin hơn khi gặp bài toán tổ hợp. Tôi đã mạnh dạn đưa vấn
đề này ra trong buổi sinh hoạt tổ chuyên môn của tổ Toán để cùng các đồng nghiệp
thảo luận và đưa ra hướng giải quyết.
7
Chương 3: QUÁ TRÌNH THỰC HIỆN
Xuất phát từ những thực trạng trên và nhằm đáp ứng được yêu cầu hiệu quả
của công tác bồi dưỡng học sinh giỏi tôi đã tìm hiểu, nghiên cứu áp dụng chuyên đề
này vào thực tế công tác bồi dưỡng học sinh giỏi. Quá trình nghiên cứu chuyên đề
tôi chia thành ba giai đoạn nghiên cứu như sau:
Giai đoạn 1: Phân dạng bài và xây dựng phương pháp
Giai đoạn 2: Xây dựng, hệ thống, chứng minh và áp dụng các bài toán tổ hợp.
Giai đoạn 3: Luyện đề về dạng bài tổ hợp tổng hợp. Củng cố phương pháp làm bài
3.1 Giai đoạn 1: Phân dạng bài và xây dựng phương pháp
+) Mục đích:
Nhằm thu thập thông tin về tài liệu, giáo viên, học sinh với vấn đề nghiên
cứu
+) Thời gian:
Từ tháng 09 đến tháng 11 năm 2012
+) Cách tiến hành:
Bước 1: Đọc và nghiên cứu các tài liệu về các bài toán tổ hợp
Bước 2: Thực hiện dự giờ bồi dưỡng học sinh giỏi của giáo viên đang bồi dưỡng
học sinh về dạng toán tổ hợp như thế nào?
Bước 3: Thảo luận, trao đổi với đồng nghiệp về cách dạy và vận dụng nguyên lí
Dirichlet và nguyên lí cực hạn để xây dựng phương pháp giảng dạy và giải bài
Bước 4: Kiểm tra sự vận dụng của học sinh
+) Kết quả giai đoạn 1
Về tài liệu: Có rất nhiều tài liệu viết về nội dụng này, trong đó đã có những
tài liệu viết khá chi tiết nhưng với số lượng rất ít. Chủ yếu các tài liệu mới chỉ đưa
ra bài tập và nêu cách chứng minh nó.
Với giáo viên: Thông qua dự giờ thăm lớp tôi nhận thấy số lượng giáo viên
đã giảng dạy cho học sinh dạng toán tổ hợp theo chuyên đề là vẫn ít. Hầu hết giáo
viên mới chỉ đưa ra các bài toán và cách giải cụ thể bài toán đó chứ không đi theo
một hệ thống bài toán.
Với học sinh: Còn khá lúng túng khi gặp các bài toán về tổ hợp. Do vậy kết quả
giải bài tập của học sinh ở dạng toán này còn chưa tốt. Đặc biệt là có rất ít học sinh
có sự sáng tạo trong khai thác bài toán. Tôi đã tiến hành khảo sát tổng số là 25 học
sinh giỏi và thu được kết quả cụ thể sau:
Giỏi Khá Trung bình yếu
1 2 5 17
Bảng 1
3.2 Giai đoạn 2: Luyện đề về dạng bài tổ hợp tổng hợp. Củng cố phương pháp
làm bài
+) Mục đích:
Nhằm cung cấp cho giáo viên, học sinh hệ thống một số dạng bài và phương
pháp giải từng loại bài ấy.
8
Rèn tư duy sáng tạo, linh hoạt trong vận dụng và khai thác các kiến thức toán
học cho học sinh.
+) Thời gian:
Từ tháng 12 năm 2012 đến tháng 2 năm 2013
+) Cách tiến hành:
Sưu tầm và nghiên cứu một số bài toán có nội dung phù hợp với mục đích
của chuyên đề nghiên cứu.
+) Kết quả giai đoạn 2
Tôi đã tiến hành khảo sát tổng số là 25 học sinh giỏi và thu được kết quả cụ thể sau:
Giỏi Khá Trung bình yếu
2 3 8 12
9
NỘI DUNG CHUYÊN ĐỀ
CÁC BÀI TOÁN SỬ DỤNG NGUYÊN LÍ CỰC HẠN
A- Lý thuyết chung
• Nguyên lí cực hạn có dạng đơn giản như sau:
Nguyên lí 1: Trong một tập hợp hữu hạn và khác rỗng các số thực luôn luôn có
thể chọn được số bé nhất và số lớn nhất.
Nguyên lí 2: Trong một tập hợp khác rỗng các số tự nhiên luôn luôn có thể chọn
được số bé nhất.
Nguyên lí này dùng để giải các bài toán mà trong tập hợp có các đối tượng phải
xét của nó tồn tại các đối tượng có GTLN, GTNN theo một nghĩa nào đó. Nguyên
lí cực hạn thường được sử dụng kết hợp với các phương pháp khác đặc biệt là
phương pháp phản chứng. Nguyên lí này được vận dụng trong trường hợp tập các
giá trị cần khảo sát là tập hữu hạn (Nguyên lí 1) hoặc vô hạn nhưng tồn tại GTLN
hoặc GTNN (Nguyên lí 2). Để vận dụng được nguyên lí cực hạn giải các bài tập
hình học tổ hợp, người ta thường dùng một lược đồ chung để giải bài tập như sau:
- Đưa bài toán đang xét về dạng sử dụng nguyên lí 1 hoặc nguyên lí 2 để chứng
tỏ rằng tất cả các giá trị cần khảo sát của bài toán có GTLN hoặc GTNN.
- Xét bài toán tương ứng khi nó nhận GTNN hoặc GTLN này.
- Chỉ ra một mâu thuẫn hoặc đưa ra giá trị lớn hơn hoặc nhỏ hơn GTLN hoặc
GTNN mà ta đang khảo sát. Theo nguyên lí của PP phản chứng ta suy ra điều phải
chứng minh.
B- Vận dụng
Ví dụ 1: Chứng minh rằng bốn đường tròn có đường kính là bốn cạnh của một tứ
giác lồi thì phủ kín tứ giác đã cho
Lấy M là một điểm tùy ý trên tứ giác
lồi. Có hai khả năng xảy ra
1) Nếu M nằm trên đường biên của
tứ giác lồi, tức là M nằm trên một
cạnh của tứ giác ABCD. Khi đó M
nằm trong đường tròn có đường kính
là cạnh ấy. Trong trường hợp này kết
luận của bài toán hiển nhiên đúng.
2) Nếu M nằm bên trong tứ giác lồi ABCD. Khi đó ta có ∠ AMB + ∠ BMC + ∠
CMD + ∠ DMA = 3600
. Theo nguyên lý cực hạn tồn tại max
{ }, , ,AMB BNC CMD DMA BMC∠ ∠ ∠ ∠ = ∠ . Khi đó ∠ BMC ≥ 900
(1). Từ (1) suy ra M
nằm trong hoặc cùng lắm là nằm trên đường tròn đường kính BC. Vậy dĩ nhiên M
bị phủ bởi đường tròn này. Như thế do M là điểm tùy ý của tứ giác ABCD, ta suy
ra bốn hình tròn phủ kín tứ giác lồi đã cho. Đó là điều phải chứng minh.
10
Ví dụ 2: Cho ABC là tam giác nhọn. Lấy một
điểm P bất kỳ trong tam giác. Chứng minh rằng
khoảng cách lớn nhất trong các khoảng cách từ P
tới ba đỉnh A, B, C của tam giác không nhỏ hơn
hai lần khoảng cách bé nhất trong caccs khoảng
cách từ P tới ba cạnh của tam giác đó.
Gọi 1 1 1, ,A B C tương ứng là hình chiếu của P xuống BC, AC, AB. Ta có
( )0
1 1 1 1 1 1 360 1APC C PB BPA A PC CPB B PA∠ + ∠ + ∠ + ∠ + ∠ + ∠ =
Theo nguyên lý cực hạn, tồn tại { }1 1 1 1 1 1ax , , , , ,m APC C PB BPA A PC CPB B PA∠ ∠ ∠ ∠ ∠ ∠
Không giảm tổng quát, cho là :
{ }1 1 1 1 1 1 1ax , , , , ,m APC C PB BPA A PC CPB B PA BPA∠ ∠ ∠ ∠ ∠ ∠ = ∠ (2)
Từ (1) và (2) dễ dàng suy ra ∠ BPA1 ≥ 600
(3)
Từ (3) ta đi đến 1
1
1
os
2
PA
c BPA
PB
∠ = ≤ hay PB ≥ 2PA1 (4) Từ (4) suy ra
{ } 1ax , , 2m PA PB PC PB PA≥ ≥ ≥ { }1 1 12min , ,PA PB PC . Đó là điều pcm.
Ví dụ 3: Trên mặt phẳng có một số điểm có tinh chất là cứ với hai điểm bất kỳ của
hệ điểm luôn tìm được điểm thứ ba trong số các điểm này thẳng hàng với chúng.
Chứng minh rằng tất cả các điểm cảu hệ điểm thẳng hàng.
Giả sử kết luận của bài toán không đúng, tức là các điểm đã cho không thẳng hàng.
Xét tập hợp sau đây { / 0A h h= > và h là khoảng cách từ một điểm đã cho đến một
đường thẳng nối hai điểm của hệ }
Do giả thiết phản chứng nên A ≠Ø. Mặt khác, A là tập hợp có hữu hạn phần tử ( do
có một số hữu hạn điểm đã cho). Theo nguyên lý cực hạn, tồn tại mọt giá trị nhỏ
nhất h*. Giả sử h* là khoảng cách từ một điểm M xuống một đường thẳng đi qua
B,C ( Ở đây M,B, C thuộc vào số các điểm đã cho). Gọi ∆ là đường thẳng nối B,
C. Do M∉∆ ( vì h* > 0), nên theo giả thiết tồn tại điểm D∈∆. Kẻ MH ⊥∆, thì MH
= d*. Rõ ràng trong ba điểm B, C, D pahir có hai điểm cùng phía so với H
Không làm giảm tính tổng quát, ta có thể cho là C, D nằm cùng phía với H và C
nằm trong đoạn HD, Kẻ HE ⊥ MD và CF ⊥ MD. Rõ ràng ta có : CF < HE < MH.
Nói cách khác CF < d*. Chú ý rằng cho C,M,D cùng nằm trong các điểm đã cho,
nên CF ∈ A. Do đó CF < d*. Điều này mâu thuẫn với định nghĩa của d*. Vậy giải
thiết phản chứng là sai, tức là tất cả các điểm đã cho phải thẳng hàng. Đó là đpcm.
11
Ví dụ 4: Trên mặt phẳng cho một số hữu hạn điểm không cùng nằm trên một
đường thẳng. Chứng minh rằng tồn tại ba điểm sao cho đường tròn đi qua ba điểm
đó không chứa điểm nào ở bên trong.
GIẢI: Vì số các điểm đã cho là hữu hạn và chúng không cùng nằm trên một đường
thẳng, nên khi lấy bao lồi hệ điểm, ta sẽ được một đa giác. Giả sử đó là đa giác lồi
1 2... pA A A . Như thế các điểm còn lại đã cho phải nằm trong bao lồi. Gọi 1,k kA A + là hai
đỉnh liên tiếp của của đa giác lồi( nghĩa là xét một cạnh tùy ý 1k kA A + ). Khi ấy mọi
điểm đã cho đều nằm ở một nửa mặt phẳng xác định bởi 1k kA A + . Từ giả thiết suy ra
tập hợp các điểm đã cho không thuộc 1k kA A + là khác rỗng. Vì thế theo nguyên lý
cực hạn, tồn tại C sao cho 1 1 1axk k k kA CA m A A A+ +∠ = ∠ , ở đây giá trị lớn nhất lấy theo
mọi 1,i n= mà
i ≠ k, i ≠ k + 1( giả sử 1 2, ,... nA A A là hệ hữu hạn điểm cho trước). Khi đó đường tròn
ngoại tiếp ta giác
1k kCA A +
là đường tròn cần tìm
Ví dụ 5: Bên trong một hình vuông cạnh 1 cho n điểm sao cho không có ba điểm
thẳng hàng. Chứng minh rằng tồn tại môt tam giác có đỉnh tại các điểm đã cho và
diện tích S của nó thỏa mãn bất đẳng thức
1
2
S
n
<
−
GIẢI:
Xét bao lồi của n điểm nằm bên trong hình vuông. Vì không có
ba điểm nào thẳng hàng, nên bao lồi là đa giác lồi có k
đỉnh( k≤n), ngoài ra các điểm đã cho hoặc là đỉnh của đa giác
lồi, hoặc nằm hẳn bên trong đa giác lồi. Chỉ có hai khả năng
xảy ra
1. Nếu k = n. Khi đó số đường chéo xuất phát từ A1 của đa giác
bao lồi tạo thành cùng các cạnh của đa giác ( n – 2) tam giác.
Gọi S là diện tích tam giác nhỏ nhất trong (n-2) tam giác ấy
Vì tổng các diện tích của (n-2) tam giác nhỏ hơn 1( chú ý 1 là diện tích hình
vuông chứa chọn ( n-2) tam giác này), suy ra
1
2
S
n
<
−
2. Nếu k < n. Khi đó bên trong đa giác bao lồi 1 2... kA A A có (n-k)
điểm Ak+1, Ak+2,...., An. Nối Ak+1 với các đỉnh A1; A2;....Ak. Khi đó có k tam giác
Ak+1A1A2, Ak+1A2A3 ; ....; Ak+1AkA1
12
Vì không có ba điểm nào thẳng hàng, Nên các
điểm Ak+2,...., An phải nằm hẳn trong k tam giác
nói trên. Giả sử Ak+2 nằm hẳn trong tam giác nào
đó. Nối Ak+2 với ba đỉnh của tam giác này, thì từ
một tam giác sẽ có ba tam giác mới . Sau mỗi lần
làm số tam giác tăng lên 2. Như vậy ta đi đến:
k+ 2(n- k – 1) = 2n – k – 2 = (n – 2) + (n – k) tam
giác.mà bên trong mỗi tam giác này không có
điểm nào thuộc n điểm đã cho. Gọi S là diện tích
bé nhất trong các tam giác trên , thế thì:
( ) ( )
1 1
n – 2 n – k 2
S
n
< <
+ − ( Do n – k >0)
Bất đẳng thức
1
2
S
n
<
−
được chứng minh.
Ví dụ 6: Bên trong hình vuông cạnh 1 cho n điểm. Chứng minh rằng tồn tại một
tam giác có đỉnh tại các điểm đã cho hoặc đỉnh của hình vuông sao cho diện tích S
của nó thỏa mãn bất đẳng thức:
1
2( 1)
S
n
≤
+
Giải:
Gọi A, B, C, D là bốn đỉnh hình vuông và A1; A2;....An là n điểm nằm trong hình
vuông. Nối A1 với 4 đỉnh A, B, C, D. Khi đó ta được 4 hình tam giác.
*) Nếu A2 nằm trong một trong 4 tam giác đó ( Giả sử A2 nằm trong tam giác
ADA1) Ta nối A2 với A, D, A1. Sau khi nối xong, số tam giác tăng thêm 2.
*) Nếu A2 nằm trên cạnh chung (Ví dụ A2 ∈ A1D) nối A2 với A và C. Khi đó số tam
giác cũng tăng thêm 2.
Như vậy trong mọi trường hợp, số tam giác sẽ tăng thêm 2
Với các điểm A3;....An ta làm tương tự. Cuối cùng số tam giác được tạo thành là: 4
+ 2(n-1) = 2n + 2 tam giác. Các tam giác trên đều có đỉnh là đỉnh của hình vuông
hoặc n điểm đã cho. Khi đó, tổng diện tích của 2n +2 tam giác này bằng diện tích
hình vuông (bằng 1).
Theo nguyên lý cực hạn, tồn tại tam giác có diện tích nhỏ nhất trong 2n + 2 tam
giác ấy. Gọi diện tích này là S thì
1
2( 1)
S
n
≤
+
(Điều cần chứng minh)
13
Bài tập tương tự:
Bài toán 1.1 . Cho n điểm nằm trong tam giác ABC có diện tích là 1 2
cm . CMR: Từ
n điểm đó cùng với 3 điểm A, B, C luôn tồn tại một tam giác có diện tích không lớn
hơn
21
3 2( 1)
cm
n+ −
.
Bài toán 1.2 (tổng quát) . Cho n điểm nằm trong đa giác lồi m đỉnh có diện tích là
1 2
cm . CMR: Từ n điểm đó cùng với m đỉnh của đa giác, luôn tồn tại một tam giác
có diện tích không lớn hơn
21
2( 1)
cm
m n+ −
.
Ví dụ 7: Trong các ô của bảng vuông kích thước xn n ô vuông, người ta viết các
số sao cho tổng của các số có mặt trong các ô của một “chữ thập” (tức là hình gồm
một hàng và một cột) bất kỳ không nhỏ hơn a. Tính giá trị nhỏ nhất của tổng các ô
trong bảng.
GIẢI:
Lấy một hàng có tổng các số trong hàng đó là nhỏ nhất.
Sau đó xét tổng tất cả có “chữ thập” được lập nên từ các
ô của hàng đó .
Có n “chữ thập” như vậy, theo điều kiện của bài toán. Ta
suy ra, tổng các số ghi ở n “chữ thập” ấy không nhỏ hơn
n.a
Dễ thấy tổng nói trên bằng tổng của tất cả các số trong
bảng cộng thêm (n – 1)lần tổng các số trong hàng lấy ra.
Gọi tổng các số trong bảng là N, tổng các số trong hàng
lấy ra là m, từ suy luận trên ta suy ra:
N +(n – 1).m ≥ n.a (1)
Theo định nghĩa số m, ta có:
N
m
n
≤ (2)
Từ (1) và (2) ta suy ra:
2
.( 1) . .
2 1
N n
N n n a N a
n n
+ − ≥ ⇔ ≥
−
Mặt khác, chọn tất cả các ô trong bảng đều bằng 2 1
a
n−
. Khi đó tổng tất cả các số
ghi trong mọi hình chữ thập là: (2 1).
2 1
a
n a
n
− =
−
Phép ghi như vậy là hợp lệ. Với cách ghi này, tổng các số ghi vào các ô trong bảng
là
2
.
2 1
n a
n −
Vậy giá trị nhỏ nhất của tổng cần tìm là
2
min
2 1
n a
N
n
=
−
Ví dụ 8: Trong không gian cho một số hữu hạn điểm mà trong đó không có 4 điểm
nào trong chúng cùng nằm trên một mặt phẳng sao cho thể tích của mỗi tứ diện tạo
14
ra bởi đỉnh là những điểm đã cho không lớn hơn 1. Chứng minh rằng tất cả các
điểm có thể được phủ bởi một tứ diện có thể tích bằng 27.
GIẢI: Do số lượng điểm đã cho là hữu hạn, nên số lượng các tứ diện tạo thành
cũng hữu hạn. Theo nguyên lý cực hạn, tồn tại tứ diện mà ta sẽ gọi là A1B1C1D1) có
thể tích lớn nhất. Qua các đỉnh A1; B1; C1; D1 dựng các mặt phẳng song song với các
mặt của tứ diện, ta nhận được tứ diện ABCD.Dễ dàng chứng minh được
A1; B1; C1; D1 tương ứng là trọng tâm của các tam
giác BCD,ACD,ABD, ABC. Từ đó ta có:
1 1 1 1
1
27
A B C D
ABCD
V
V
= (1)
Do giả thiết 1 1 1 1
1A B C DV ≤ , nên từ (1) ta suy ra
27ABCDV ≤ (2)
Bây giờ ta chứng minh tất cả các điểm đều nằm
trong tứ diện ABCD.
Giả sử: Tồn tại điểm M(trong số các điểm đã cho), sao cho M không thuộc tứ diện
ABCD (3)
Khi đó ít nhất một đỉnh của tứ diện ABCD (có thể cho đó là đỉnh B) sao cho B và
M nằmtrong hai nửa không gian xác định bởi (ACD).
Suy ra 1 1 1 1 1 1 1MA C D B A C DV V> (1)
Bất đẳng thức (4) chứng tỏ rằng MA1C1D1 là tứ diện tạo bởi 4 đỉnh trong các điểm
đã chốc thể tích lớn hơn thể tích của tứ diện B1A1C1D1. Điều này mâu thuẫn vơi
cách định nghĩa tứ diện A1B1C1D1 (vô lý). Suy ra điều phải chứng minh.
15
II- SỬ DỤNG NGUYÊN LÝ DIRICHLET ĐỂ GIẢI CÁC BÀI TOÁN
HÌNH HỌC TỔ HỢP
A- Nguyên lý:
*) Nguyên lý Dirichlet (Gustav Lejeuve Dirichlet) : Nếu nhốt n con thỏ vào m cái
chuồng ( 2m ≥ ) thì tồn tại một chuồng có ít nhất
1n m
m
+ − 
 
 
con thỏ.
B- Vận dụng:
Ví dụ 1: Cho p là số nguyên tố lớn hơn 5. chứng minh rằng tồn tại một số có dạng
111...11 mà chia hết cho p.
GIẢI:
Xét dãy số 1,11,111,..., 14243
höõsoá1
111....11
p c
Ta chứng minh trong dãy trên phải có số chia hết cho p. Giả sử kết luận ấy không
đúng, tức là không có bất kỳ số nào của dãylại chia hết cho p.
Cho tương ứng mỗi số dư của phép chia cho p . Tập hợp số dư có thể thuộc tập hợp
{1, 2, 3,..., p – 1} (Do 0 không thể thuộc tập hợp này). Ta lại có p số trong dãy số
trên. Vì vậy theo nguyên lý Dirichlet tồn tại ít nhất hai số có cùng số dư khi chia
cho p. Giả sử các số đó là 111...11 (m chữ số 1) và số 111....11 (n chữ số 1) với
( )1 n m p≤ < ≤ . Từ đó ta có
/ / / 1 / 0 / 1
(111...11 111...11) , 111...1000...0 111...1.10n
m c s n c s m n c s n c s m n c s
p hay p Hay p
− −
− M M M14243 14243 123 123 123 (1)
Do p là sô nguyên tố lớn hơn 5 nên (p; 10) = 1, Vì thế từ (1) ta suy ra / 1
111...1
m n c s
p
−
M123 (2)
/ 1
111...1
m n c s−
123 là một số thuộc dãy trên nên từ (2) suy ra mâu thuẫn với giả thiết. Vậy giả
sử phản chứng là sai. Ta suy ra điều phải chứng minh.
Ví dụ 2: Trong hình vuông đơn vị (cạnh bằng 1) có 101 điểm. Chứng minh rằng có
5 điểm đã chọn được phủ bởi hình tròn bán kính
1
7
.
GIẢI:
Chia hình vuông thành 25 hình vuông nhỏ bằng nhau,
mỗi hình vuông có cạnh 0,2. Vì có 101 điểm, mà chỉ có
25 hình vuông, nên theo định lý Dirichlet tồn tại một
hình vuông nhỏ có chứa ít nhất năm điểm ( trong 101
điểm đã cho ). Vì hình vuông này nội tiếp trong đường
tròn có bán kính R =
1
2
25
2 10
= .
Do
2 1
10 7
< nên dĩ nhiên đường tròn đồng tâm với đường
16
tròn ngoại tiếp trên và có bán kính
1
7
chứa ít nhất năm
điểm nói trên . Đó là đpcm
Ví dụ 3: trong hình chữ nhật 3x4 đặt sáu điểm. chứng minh rằng trong số đó luôn
tìm được hai điểm có khoảng cách giữa chúng không lớn hơn 5
GIẢI
Chia hình chữ nhật đã cho thành năm hình ABCD,
DCKFE, KFNM, NFEQR, QEDAS
Vì có 6 điểm nên theo nguyên lý Dirichlet tồn tại một
trong năm hình trên chứa ít nhất 2 trong 6 điểm nói
trên. Theo định lý Pitago thì khoảng cách lớn nhất
giữa hai điểm trong một hình bằng 5.
Ví dụ AC = CE = KE = KM = DS = DQ = QF = FR = 5
Từ đó ta luôn tìm được 2 điểm trong 6 điểm đã cho có khoảng cách không
lớn hơn 5 (Đpcm)
BÀI TOÁN TƯƠNG TỰ 1:
Trong một tam giác đều, cạnh có độ dài bằng 1, đặt năm điểm . Chứng minh rằng
tồn tại hai điểm mà khoảng cách giữa chúng nhỏ hơn hoặc bằng
1
2
Thật vậy, gọi M, N, P tương ứng là trung điểm của
AB, BC, CA .Khi đó AMP, BMN, MNP, MNP,
NPC là bốn tam giác đều bừng nhau cạnh bằng
1
2
.
Chú ý nếu P là tam giác đều cạnh bằng a, thì
d(P)=a
Vì thế d(AMP)=d(BMN) =d(MNP) =d(NPC)=
1
2
Vì năm điểm thuộc vào bốn tam giác, nên theo nguyên lý Dirichlet tồn tại ít nhất
hai điểm, giả sử Ai , Aj thuộc về cùng một tam giác trong bốn tam giác đều nhỏ nói
trên . Ta có AiAj ≤ d(AMP)=
1
2
Đó là đpcm
BÀI TOÁN TƯƠNG TỰ 2 :
Trong hình tròn đường kính 5 có 10 điểm . Chứng minh rằng tồn tại ít nhất hai
điểm mà khoảng cách giữa chúng bé hơn hoặc bằng 2.
GIẢI
Thật vậy: Trong đường tròn tâm O đường kính 5, vẽ đường tròn đồng tâm có
đường kính bằng 2. Chia đường tròn đã cho thành 9 phần(như hình vẽ).
17
Xét một phần bất kỳ, giả sử là hình III – ABCD
Ta thấy ngay, khoảng cách lớn nhất giữa hai điểm của (III) là DA = BC
Do ∠ DOA = 450
nên d2
= DA2
= DO2
+ OA2
-2DO.OA.cos450
2
25 5 2
1 2. .1. 2
2 2 2
d
 
= + − ⇒ < ÷
 
Theo nguyên lý Dirichlet tồn tai ít nhất hai điểm nằm trong môt trong 9 phần (I),
(II), .....(IX) có đường kính không vượt quá 2. Suy ra điều phải chứng minh.
Ví dụ 4: Tìm hình vuông có kích thước bé nhất, để trong hình vuông đó có thể sắp
xếp năm hình tròn có bán kính 1, sao cho không có hai hình tròn nào trong chúng
có điểm trong chung.
GIẢI:
Giả sử hình vuông ABCD có tâm O cạnh là a chứa 5 hình tròn không cắt nhau và
đều có bán kính bằng 1. Vì cả 5 hình tròn này nằm trọn trong hình vuông, nên tâm
của chúng nằm trong hình vuông A1B1C1D1 tâm O cạnh là a-2. Ở đây AB // A1B1.
Các đường thẳng nối nối các trung điểm của các cạnh đối diện của hình vuông
A1B1C1D1 chia A1B1C1D1 thành 4 hình vuông nhỏ. Theo nguyên lý Dirichlet tồn tại
một trong 4 hình vuông nhỏ, mà trong hình vuông này chứa ít nhất hai trong số 5
tâm hình tròn nói trên(Không mất tính tổng quát, giả sử đó là O1O2)
Do trong 5 đường tròn, không có 2 đường tròn nào cắt nhau nên O1O2 ≥ 2 (1)
18
Mặt khác, do O1O2 cùng nằm trong một hình vuông nhỏ(cạnh hình vuông bằng
2
2
a −
nên). Mà O1O2
2
2
2
a −
≤
(2)
Từ (1) và (2) ta suy ra
2
2 2 2 2 2
2
a
a
−
≥ ⇔ ≥ + (3)
Vậy mọi hình vuông cạnh a thỏa mãn điều kiện đề bài sẽ thỏa mãn (3)
Ta xét hình vuông ABCD có a = 2 2 2+ . Và xét 5 hình tròn có tâm O, A1; B1; C1;
D1 (hình vẽ) thì mọi yêu cầu của bài toán được thỏa mãn .
Vậy kích thước bé nhất của cạnh hình vuông thỏa mãn điều kiện đề bài là 2 2 2+
Ví dụ 5: Cho 2014 đường thẳng cùng có tính chất: chia hình vuông thành hai tứ
giác có tỉ số diện tích bằng
2
3
. Chứng minh rằng có ít nhất 504 đường thẳng trong
2014 đường thẳng trên đồng quy.
GIẢI:
Các đường thẳng chia hình vuông thành hai tứ giác nên chúng không thể cắt hai
cạnh kề của hình vuông và không đi qua đỉnh nào của hình vuông.
Giả sử một đường thẳng cắt hai cạnh đối
AD và BC tại M và N. Ta có
1
( )
2 2 22
13 3 IF 3
( )
2
ABMN
MCDN
AB BM ANS EI
S
CD MC ND
+
= ⇒ = ⇔ =
+
(E,F,P,Q lần lượt là trung điểm các cạnh
hình vuông. I,K,G,H lần lượt là những
điểm thỏa mãn
2
IF 3
IE HP GF KQ
HQ GE KP
= = = =
Từ lập luận trên ta suy ra mỗi đường thẳng thỏa mãn yêu cầu của đề bài đều đi qua
một trong 4 điểm G, H, I, K nói trên.
Do có 2014 đường thẳng, nên theo nguyên lý Dirichlet phải tồn tại ít nhất
2014
1 504
4
 
+ = 
 
đường thẳng cùng đi qua một điểm trong 4 điểm G,H, I, K nói trên.
Vậy có ít nhất 504 đường thẳng trong số 2014 đường thẳng đã cho đồng quy.
Ví dụ 6: Cho một bảng kích thước 2n x 2n ô vuông. Người ta đánh dấu vào 3n ô
bất kỳ của bảng. Chứng minh rằng có thể chọn ra n hàng n cột của bảng sao cho
các ô được đánh dấu đều nằm trong n hàng n cột này.
GIẢI:
19
Trong các hàng có ô được đánh dấu. Chọn ra n hàng có số ô được đánh dấu nhiều
nhất. Ta chứng minh rằng số ô được đánh dấu còn lại nhỏ hơn hoặc bằng n.
Giả sử số ô được đánh dấu còn lại lớn hơn hoặc bằng n +1 . Số hàng còn lại chưa
chọn là n hàng. Theo nguyên lý Dirichlet sẽ có ít nhất một hàng (Trong số n hàng
còn lại chứa ít nhất 2 ô đã được đánh dấu).
Từ cách chọn ta suy ra trong n hàng được chọn thì mỗi hàng có ít nhất 2 ô được
đánh dấu. Tức là trên n hàng đã chọn có ít nhất 2n ô đã được đánh dấu.
Như vậy, số ô được đánh dấu lớn hơn hoặc bằng 2n + (n +1) > 3n (vô lý). Suy ra
điều phải chứng minh.
Như vây, sau khi đã chọn ra n hàng với cách chọn như trên. Theo nhận xét sẽ còn
lại không quá n ô được đánbh dấu. Hay cùng lắm sẽ có n cột chứa chúng.
Vậy: có thể chọn ra n hàng n cột của bảng sao cho các ô được đánh dấu đều nằm
trong n hàng n cột này.
Ví dụ 7
Cho 1000 điểm M1, M2. …M1000 trên một mặt phẳng. Vẽ môt đường tròn bán kính 1
tùy ý. Chứng minh rằng tồn tại điểm S trên đường tròn sao cho SM 1 + SM2+ …
+SM1000 ≥ 1000
Giải:
S1
M1
M2
M1000
S2
Xét một đường kính S1S2 tùy ý của đường tròn, ở đây S1 vaf
S2 là hai đầu của
đường kính nên ta có
1 1 2 1 1 2
1 2 2 2
1 1000 2 1000
2
2
...
2
S M S M S S
S M S M
S M S M
 + ≥ =

+ ≥


 + ≥
Cộng từ vế 1000 bất đẳng trên ta có
(S1M1+ S1M2 +…+ S1M1000 ) + (S2M1+ S2M2 +…+ S2M1000 ) ≥ 2000 (1)
Từ (1) và theo nguyên ý Dirichlet suy ra trong hai tổng của vế trái cảu (1) có ít
nhất một tổng lớn hơn hoặc bằng 1000
Giả sử (S1M1+ S1M2 +…+ S1M1000 )≥ 1000, khi đó S = S1. Đó là ddiepcm.
Ví dụ 8:
Cho mỗi điểm trên mặt phẳng được tô bằng một trong hai màu xanh, đỏ. Chứng
minh rằng tồn tại một tam giác mà ba đỉnh và trọng tâm cùng màu
GIẢI:
G
M
P
A
B C
B'
C'
A'
N
Lấy 5 điểm tùy ý sao cho không có ba điểm nào
thảng hàng trên mặt phẳng . Khi đó vì chỉ dùng có
hai màu để tô các đỉnh nên theo nguyên lý
Dirichlet phải tồn tại ba điểm cùng màu. Giả sử ba
điểm đó là ba điểm A, B, C cùng có màu đỏ. Như
vậy ta có tam giác ABC với ba đỉnh màu đỏ. Gọi G
20
là trọng tâm tam giác ABC. Chỉ có hai khả năng
xảy ra:
1) Nếu G là màu đỏ. Khi đó A,B, C, G có cùng màu đỏ và bài toán được chứng
minh
2) Nếu G có màu xanh. Kéo dài GA, GB, GC các đoạn AA’ = 3GA, BB’ = 3GB,
CC’ = 3GC. Khi đó nếu gọi M, N, P tương ứng là các trung điểm của BC, CA, AB
thì AA’ = 3GA = 6GM => AA’ = 2AM. Tương tự B’B = 2BN, C’C = 2CP.Do đó
các tam giác A’BC, B’AC, C’AB tương ứng nhận A,B,C là trọng tâm. Mặt khác, ta
cũng các có tam giác ABC, và A’B’C’ có cùng trọng tâm G. Có hai trường hợp xảy
ra.
a) Nếu A’, B’ C’ cùng màu xanh. Khi đó tam giác A’B’C’ và trọng tâm G có cùng
màu xanh.
b) Nếu ít nhất một trong các điểm A’ B’ C’ có màu điểm. Không mất tính tổng quát
giả sử A’ màu đỏ. Khi đó tam giác A’BC và trọng tâm có màu đỏ. Vậy trong mọi
khả năng luôn tồn tại một tam giác mà ba đỉnh và trọng tâm màu đỏ. Đó là đpcm.
Ví dụ 9:
Một hình lập phương có cạnh bằng 15 chứa 11000 điểm. Chứng minh rằng có một
hình cầu bán kính 1 chứa ít nhất sáu điểm trong số 11000 điểm đã cho.
Giải Chia mỗi cạnh của hình lập phương thành 13 phần bằng nhau. Như thế hình
lập phương đã cho được chia thành 133
= 2197 hình lập phương nhỏ. Do
11000>5.2197 = 10985, nên tồn tại ít nhất một hình lập phương nhỏ, mà hình lập
phương này chứa ít nhất sáu điểm. Như đã biết nếu gọi cạnh hình vuông băng a, thì
hình càu ngoại tiếp nó có bán kinh R, với
1
3
2
R a= . Vì thế hình cầu ngoại tiếp hình
lập phương nhỏ ( cạnh của nó là
15
13
) là
2
1 15 1 15 1 675 1 676 1
3 3 4 1
2 13 2 13 2 196 2 169 2
R
 
= = = < = = ÷
 
. Hình cầu bán kính 1 này dĩ
nhiên chứa ít nhất sáu điểm trong số 11000 điểm đã cho . Đó là đpcm.
Ví dụ 10: Trong hình vuông cạnh 12 chứa 2014 điểm. Chứng minh rằng luôn tồn
tại một tam giác đều cạnh 11 phủ kín 504 điểm trong 2014 điểm đã cho.
GIẢI:
Giả sử hình vuông ABCD có tâm là O và cạnh 12.
Lấy E, F, G, H lần lượt trên các cạnh CD,AB,AD,BC sao
cho AG = DE = CH = BF = 6 2 3− .Khi đó OE = OF = OG
= OH = 4 3
Ta đi chứng minh có thể dùng một tam giác đều cạnh 11
phủ kín một trong các tứ giác OHCE, OEDG, OGAF hoặc
OFBH.
Thật vậy, do OH = OE = 4 3 < 11.
21
Lấy J trên OF sao cho EJ = 11. Ta thấy 06 3
inF 60
24 3
S EC FEC= = ⇒ ∠ =
Trên tia EC lấy K sao cho EK = EJ = 11. Ta có tam giác JEK đều cạnh 11. Ta đi
chứng minh tam giác JEK phủ kín tứ giác OHCE
Gọi giáo điểm của JK với BC là I.
Suy ra IC = CK. 3 = 3 (5 -2 3 ) = 5 3 - 6 > 6 - 2 3 = CH
CH < CI nên H nằm giữa C và I. Suy ra tam giác JEK phủ kín hoàn toàn tứ giác
OHCE.
Do vai trò của các tứ giác OHCE, OEDG, OGAF, OFBH là như nhau.
Áp dụng nguyên lý Dirichlet ta suy ra: luôn tồn tại
2014
1 504
4
 
+ = 
 
điểm trong 2014
điểm đã cho nằm trong một trong các tứ giác OHCE, OEDG, OGAF hoặc OFBH.
Vậy luôn tồn tại một tam giác đều cạnh 11 phủ kín 504 điểm trong 2014 điểm đã
cho.
22
III. MỘT SỐ DẠNG TOÁN HÌNH HỌC TỔ HỢP THƯỜNG GẶP
1.Dạng bài tập tô màu, bảng vuông.
Bài 1: Trong mỗi ô bàn cờ kích thước 5x5 có một con bọ dừa. Vào một thời điểm
nào đó tất cả các con bọ dừa bò sang ô bên cạnh (ngang hoặc dọc). Có thể khẳng
định rằng sau khi các con bọ dừa di chuyển sẽ luôn có ít nhất một ô trong bàn cờ
không có con bọ dừa nào trong đó không?
(Đề thi giao lưu HSG môn Toán lớp 8 TP Vĩnh Yên năm học 2012-2013)
Ta tô đen - trắng các ô bàn cờ như hình vẽ. Khi đó số ô đen nhiều hơn
số ô trắng. Như vậy số con bọ dừa ở ô đen sẽ nhiều hơn số con bọ dừa
ở ô trắng. Do mỗi con bọ dừa chỉ di chuyển sang ô bên cạnh(ngang
hoặc dọc), vì thế sau khi di chuyển các ô đen sẽ chứa các con bọ dừa ở
ô trắng.
Mà số con bọ dừa ở ô đen nhiều hơn số con bọ dừa ở ô
trắng nên sau khi các con bọ dừa bò đi sẽ có ít nhất một ô đen bị bỏ trống .
Vậy : Có thể khẳng định rằng sau khi di chuyển sẽ luôn có ít nhất một ô trong bàn
cờ không có con bọ dừa nào trong đó.
Ví dụ 2:
Trên lưới ô vuông cạnh 1. Người ta tô màu các ô bằng 2 màu đen trắng xen kẽ.
Tính bán kính lớn nhất của đường tròn chỉ đi qua ba ô trắng.
- Xét đường tròn chỉ thuộc một ô trắng: Đường kính
của nó bằng 1 (1)
- Xét đường tròn đi qua nhiều ô trắng => đường
tròn đó phải đi qua các đỉnh của ô trắng. Không mất
tính tổng quát, giả sử đường tròn đi qua đỉnh A của
ô trắng ABCD.
+) Nếu đường tròn đi qua 2 đỉnh liên tiếp của ô trắng ( A, B). Khi đó ta lại xét hai
trường hợp: *) Đường tròn qua A, B, E. Khi đó nó là đường tròn
2
,
2
I
 
 ÷
 ÷
 
*) Đường tròn qua A, B, G. Khi đó nó là đường tròn
10
;
2
K
 
 ÷
 ÷
 
(2)
+) Nếu đường tròn qua 2 đỉnh đối diện của ô trắng, Giả sử là (A, C)
Ta lại xét hai trường hợp:
*) Qua A, C, M (Tương tự qua A, B, G)
*) Qua A, C, N (Tương tự qua A, B, G)
Cả hai trường hợp trên bán kính của đường tròn là
10
2
(3)
23
Từ (1), (2) và (3) ta suy ra bán kính lớn nhất của đường tròn thỏa mãn yêu cầu đề
bài là
10
2
.
Ví dụ 3:
Cho 6 điểm trong đó 3 điểm nào cũng nối được với nhau tạo thành 1 tam giác có
cạnh được tô bởi một trong hai màu xanh hoặc đỏ. CMR: Bao giờ cũng tồn tại một
tam giác có 3 cạnh cùng màu.
Lời giải:
Gọi A là một trong 6 điểm, 5 đoạn thẳng nối A với 5 điểm còn lại được tô
bởi 2 hai màu xanh hoặc đỏ nên tồn tại 3 cạnh cùng màu. Giả sử là AB, AC, AD
Xét 2 trường hợp:
+Trường hợp 1: AB, AC, AD tô màu đỏ.
Xét BCD∆ . Nếu có một cạnh được tô màu đỏ (giả sử BC) thì ABC∆ cùng
màu đỏ (hình 1).
Nếu không có cạnh nào của BCD∆ tô màu đỏ thì BCD∆ có 3 cạnh cùng màu
xanh (hình 2).
+Trường hợp 2: AB, AC, AD tô màu xanh. Chứng minh tương tự.
Vậy luôn tồn tại một tam giác có 3 cạnh cùng màu.
Ví dụ 4:
Trên tờ giấy có kẻ vô hạn các ô vuông và mỗi ô được tô bằng một trong hai màu
xanh hoặc đỏ. Cho bất cứ hình chữ nhật nào kích thước 2x3 thì có đúng hai ô màu
đỏ. Xét hình chữ nhật có kích thước 2013x2014 bất kì. Tính số ô đỏ của nó.
Giải
Ta có nhận xét sau :
Mọi hình chữ nhật 1x3 chứa đúng một ô màu đỏ.
Thật vậy, giả sử kết luận của nhận xét không đúng, tức là tồn
tại hình chữ nhật 1x3 có số ô màu đỏ không khác một. Không
giảm tổng quát giả sử đó là hình chữ nhật AKHD kích thước
1x3 có hai ô đỏ ( nếu không thì không có ô đỏ nào, nhưng
không thể là ba vì trong mọi hình chữ nhật 2x3 có đúng hai ô
đỏ mà thôi )
Trường hợp AKHD không có ô đỏ nào thì lí luận tương tự
Cũng có thể cho là hai ô đỏ của AKHD là ô 7 , ô8
24
Xét hình chữ nhật BFNA. Đó là hình chữ nhật 2x3 , nên theo giả thiết nó có đúng
hai ô đỏ mà 7 và 8 là hai ô đỏ, do đó các ô 1,2,4,5 là màu xanh.
Xét hình chữ nhật BCHK, từ giả thiết và do các ô 1,2,4,5 màu xanh nên các ô 3 ,6
là màu đỏ.
Xét hình chữ nhật ECDM kích thước 2x3, do ô 3,6,8 màu đỏ suy ra mâu thuẫn
Vậy giả thiết phản chứng là sai. Nhận xét được chứng minh.
Vì 2013 chia hết cho 3 và 2013: 3=671. do vậy hình chữ nhật kích thước
2013x2014 chia thành 2014x671 hình chữ nhật 1x3
Vậy số ô đỏ trong một hình chữ nhật kích thước 2013x2014 là 2014x671 ô
Số ô đỏ cần tìm là 1351394 ô
3.Dạng bài tạo đa giác bao.
Ví dụ: Cho một đa giác lồi có diện tích k 2
cm . CMR: Tồn tại một hình bình hành
có diện tích không vượt quá 2k 2
cm chứa toàn bộ đa giác.
Lời giải:
Gọi C là đỉnh cách xa AB nhất (hình vẽ).
+Trường hợp 1: Nếu AC là đường chéo của đa giác lồi.
Qua C kẻ a // b ( bBA ∈, )
Gọi D,E là các đỉnh cách xa AC nhất, qua D kẻ đường thẳng
d // AC, qua E kẻ đường thẳng c // AC. Gọi MNPQ
là hình bình hành tạo bởi a ,b,c,d suy ra các đỉnh của đa giác
nằm trong hoặc trên biên của hình bình hành MNPQ.
Ta chứng minh
2
2MNPQ
S k cm≤ , thật vậy: Gọi Sd là diện tích đa giác
Ta có: ACD ACE d
S S S+ ≤ ⇔ 1
2 MNPQ d
S S≤ = k ⇔ 2
2 ( )MNPQ
S k cm≤ .
+Trường hợp 2: Nếu AC là cạnh của đa giác lồi. Gọi E là đỉnh cách xa AC nhất
( Chứng minh tương tự).
Suy ra ta có điều phải chứng minh.
4.Phương pháp qui nạp toán học:
Để chứng minh mệnh đề An đúng với mọi n ∈N*:
-B1: chỉ ra mệnh đề đúng với n=1 tức là A1 đúng.
-B2: giả sử mệnh đề đúng với n = k ( k ∈ N*) thì Ak đúng.
-B3: chứng minh Ak+1 đúng ( mệnh đề đúng với n=k+1).
Kết luận An đúng với mọi n ∈ N*.
Ví dụ 1:
Người ta dùng loại gạch để lát những căn phòng hình vuông n x n và
luôn để trống 1 ô ở goc phòng.
a) Hãy nêu cách lát căn phòng 4x4, 8x8 ô vuông.
25
b) Chứng tỏ rằng người ta luôn lát được một căn phòng kích thước 2k
x 2k
sao
cho ô trống là một ô bất kỳ.
GIẢI :
a) Xét hình vuông kích thước 4x4 ô vuông. Ta chia hình vuông thành 4 hình vuông
kích thước 2 x 2. Hình thứ nhất ta đặt viên gạch sao chô ô ở góc bỏ trống, 3 hình
còn lại ta lát sao cho ô trống quay vào phần tâm hình vuông. Cuối cùng ta lát viên
gạch vào 3 ô còn thiếu (Hình 2)
-) Để lát hình vuông 8 x 8 ô vuông, ta lát tương tự (Chia hình vuông 8 x 8 thành 4
hình vuông 4 x 4) (H3)
b) Ta chứng minh bằng PP quy nạp.
- Xét với k = 1: Bài toán luôn đúng.
- Giả sử bài toán đúng với với k = n. Nghĩa là, có thể lát được hình vuông 2n
x2n
sao
cho còn trống một ô (i ; j) bất kỳ.
- Ta chứng minh bài toán đúng với k = n +1.
Thật vấy : Xét hình vuông 2n+1
x 2n+1
= 4x(2n
x2n
). Ta chia hình vuông 2n+1
x 2n+1
thành 4 hình vuông 2n
x2n
. Theo giả thiết quy nạp, luôn lát được hình 2n
x2n
sao cho
có thể trống ô (i, j) bất kỳ. Ba hình vuông 2n
x2n
còn lại ta để trống ô ở góc và lát
vào ô trung tâm (tương tự H3) cuối cùng lát viên gạch vào 3 ô trống. Bài toán được
chứng minh.
(H1)
(H2)
(H3)
Ví dụ 2: CMR: số đường chéo của đa giác lồi n cạnh (n ≥4) bằng S n =
2
)3( −nn
.
Lời giải:
Ta chứng minh Sn =
2
)3( −nn
(1) đúng với mọi n ≥4.
+) Ta thấy (1) đúng với n=4 vì S 4 =2, tứ giác có 2 đường chéo.
+) Giả sử khẳng định (1) đúng với n=k (k ≥4) tức là đa giác lồi
k
cạnh có
2
)3( −kk
đường chéo.+) Ta sẽ chứng minh đa giác lồi
26
k+1cạnh có
2
)2)(1( −+ kk
đường chéo
Thật vậy khi thêm đỉnh thứ k+1 (hình vẽ) thì có thêm k-2 đường chéo nối từ A 2+k
đến A 2 , A 3 ,…, A 1−k , ngoài ra cạnh A 1 A k cũng trở thành đường chéo. Do
đó, S 1+k = S k +(k-2)+1=
2
)3( −kk
+k-1=
2
)2)(1( −+ kk
Vậy khẳng định (1) đúng với mọi n thuộc N*, n ≥4.
MỘT SỐ DẠNG BÀI TỔNG HỢP KHÁC
Ví dụ 1: Cho một cái bàn hình chữ nhật. Hai người chơi như sau: người thứ nhất
dùng 1 đồng xu màu trắng đặt lên bàn, sau đó người thứ hai đặt 1 đồng xu đen lên
bàn ở vị trí mà trước đó chưa có đồng xu nào đặt và cứ như vậy cho đến khi không
còn chỗ để đặt đồng xu nào nữa. Biết rằng tất cả các đồng xu là bằng nhau. Người
nào đến lượt đi mà không đặt được đồng xu nào lên bàn thì người đó thua cuộc.
Chứng minh rằng có cách chơi để người thứ nhất luôn luôn thắng cuộc.
(Đề thi HSG lớp 7-TP Vĩnh Yên năm học 2012-2013)
Ta tô đen - trắng các ô bàn cờ như hình vẽ. Khi đó số ô
đen nhiều hơn số ô trắng. Như vậy số con bọ dừa ở ô
đen sẽ nhiều hơn số con bọ dừa ở ô trắng. Do mỗi con
bọ dừa chỉ di chuyển sang ô bên cạnh(ngang hoặc dọc),
vì thế sau khi di chuyển các ô đen sẽ chứa các con bọ
dừa ở ô trắng.
Mà số con bọ dừa ở ô đen nhiều hơn số con bọ dừa ở ô
trắng nên sau khi các con bọ dừa bò đi sẽ có ít nhất một ô đen bị bỏ trống .
Vậy : Có thể khẳng định rằng sau khi di chuyển sẽ luôn có ít nhất một ô trong
bàn cờ không có con bọ dừa nào trong đó.
Ví dụ 2: Trong mỗi ô bàn cờ kích thước 5x5 có một con bọ dừa. Vào một thời
điểm nào đó tất cả các con bọ dừa bò sang ô bên cạnh (ngang hoặc dọc). Có thể
khẳng định rằng sau khi các con bọ dừa di chuyển sẽ luôn có ít nhất một ô trong
bàn cờ không có con bọ dừa nào trong đó không?
(Đề thi HSG lớp 8-TP Vĩnh Yên năm học 2012-2013)
Để đảm bảo thắng cuộc người thứ nhất phải có chiến lược chơi như sau:
+ Đầu tiên anh ấy chiếm vị trí trung tâm, tức là đặt đồng xu trắng
Sao cho tâm của đồng xu trùng với tâm hình chữ nhật (Vị trí A)
+ Giả sử người chơi thứ 2 đặt đồng xu đen lên bàn (Tâm đồng xu là B)
+ Khi đó điểm đối xứng với B qua tâm A là D chắc chắn còn trống.
27
Người thứ nhất đặt đồng xu trắng
sao cho tâm đồng xu trùng D.
+ Luật chơi cứ tiếp tục như vậy.
Nghĩa là sau khi người thứ hai đặt
đồng xu thì người thứ nhất chọn vị
trí đối xứng qua tâm A để đặt đồng
xu của mình (lưu ý các vị trí đối
xứng này luôn chưa có đồng xu
nào đặt trước đó)
Vì vậy, nếu người thứ hai còn đi được thì người thứ nhất vẫn đi được ở bước tiếp
theo. Vì vậy người thứ nhất sẽ không bao giờ thua cuộc.
Do mặt bàn có diện tích hữu hạn, nên nếu thực hiện theo chiến thuật trên thì
người đi trước chắc chắn đảm bảo chiến thắng thuộc về mình.
Ví dụ 3: Có 2008 con gà nhốt vào 1004 cái chuồng, mỗi chuồng có 2 con.Sau mỗi
ngày người ta lại thay đổi vị trí của gà sao cho không có hai con gà nào đã nhốt
chung chuồng trước đólại nằm cùng chuồng lần nữa.Hỏi tối đa có bao nhiêu ngày
làm được như vậy?
(Đề khảo sát HSG Huyện Vĩnh Tường năm học 2008-2009)
Bài giải:
Vẽ đa giác đều 2007 cạnh nội tiếp trong đường tròn. Ký hiệu tâm là 2008, Còn các
đỉnh là 1, 2, 3, ….2007. Ký hiệu đoạn thẳng nối i với j là i – j với , 1,2008i j = .
Xét bán kính 1 – 2008. Do tính chất đa giác đều nên ta thấy ngay 1003 dây cung
sau đây vuông góc với bán kính ấy: 2 – 2007; 3 – 2006; 4 – 2005; ....; 1004 – 1005
Xét bán kính 2 – 2008 . Tương tự ta có 1003 dây cung sau đây vuông góc với bán
kính ấy: 1- 3; 2007 – 4; 2006 – 5...
Xét bán kính 3 – 2008. Tương tự ta có 1003 dây cung sau đây vuông góc với bán
kính ấy: 2 – 4; 1 – 5; 2007 – 6; ....
Cứ làm như vậy và cuối cùng xét bán kính 2007 – 2008, ta có 1003 dây cung sau
đây vuông góc với bán kính ấy: 2006 – 1; 2005 – 2; 2004 – 3; ....
28
Dựa vào nhận xét trên và cách đánh số các con gà từ 1 – 2008, ta chỉ ra cách xếp gà
theo yêu cầu bài toán như sau:
+ Ngày thứ nhất xếp vào chuồng các đôi gà như sau:1 – 2008; 2- 2007; ....; 1004 –
1005
+ Ngày thứ hai xếp vào chuồng các đôi gà như sau: 2- 2007;1-3;2007 – 4;...; 1006
– 1005
+ Ngày thứ ba xếp vào chuồng các đôi gà như sau: 3 – 2008; 2- 4;1-5;....; 1007 –
10056
Tương tự
+ Ngày thứ 2007 xếp vào chuồng các đôi gà như sau: 2007 – 2008; 2006- 1; ....;
1004 – 1003
Mặt khác không có qua 2007 ngày vì mỗi con gà chỉ có thể cùng chuồng với một
trong 2007 con gà còn lại.
Vậy tối đa có 2007 ngày để xếp gà theo yêu cầu của đề bài.
Ví dụ 4: Xét 20 số nguyên dương đầu tiên 1,2,3, ,20.… Hãy tìm số nguyên dương k
nhỏ nhất có tính chất: Với mỗi cách lấy ra k số phân biệt từ 20 số trên, đều lấy được
hai số phân biệt a và b sao cho a b+ là một số nguyên tố.
(Đề tuyển sinh THPT chuyên Vĩnh Phúc 2013-2014)
Xét tập hợp { }2,4,6,8,10,12,14,16,18,20 , ta thấy tổng của hai phần tử bất kì của tập
hợp này đều không phải là số nguyên tố.
Do đó 11k ≥ , ta sẽ chứng minh 11k = là số nhỏ nhất thỏa mãn yêu cầu bài toán.
Thật vậy, ta chia tập hợp { }1,2,3,...,20A = thành 10 cặp số sau:
( ) ( ) ( ) ( ) ( ) ( ) ( ) ( ) ( ) ( )1,2 , 3,16 , 4,19 , 5,6 , 7,10 , 8,9 , 11,20 , 12,17 , 13,18 , 14,15 .
Tổng của hai số trong mỗi cặp số trên là số nguyên tố.
Khi đó mỗi tập con của A có 11 phần tử thì tồn tại ít nhất hai phần tử thuộc cùng
vào một trong 10 cặp số trên. Suy ra trong A luôn có hai phần tử phân biệt có
tổnglà một số nguyên tố.
Ví dụ 5: Hỏi có hay không 16 số tự nhiên, mỗi số có ba chữ số được tạo thành từ
ba chữ số a, b, c thỏa mãn hai số bất kỳ trong chúng không có cùng số dư khi chia
cho 16?
(Đề tuyển sinh THPT chuyên Vĩnh Phúc 2013-2014)
Trả lời: Không tồn tại 16 số như vậy. Thật vậy, giả sử trái lại, tìm được 16 số thỏa
mãn. Khi đó, ta có 16 số dư phân biệt khi chia cho 16:
0,1,2,3,4,5,6,7,8,9,10,11,12,13,14,15; trong đó có 8 số chẵn, 8 số lẻ.
Do đó, ba chữ số a, b, c khác tính chẵn lẻ, giả sử hai chữ số chẵn là a, b và chữ số
lẻ là c.
Có 9 số lẻ được tạo thành từ những chữ số này: , , , , , , , , .aac abc acc bac bbc bcc cac cbc ccc
Gọi 1 2 9, , ,x x x… là các số có hai chữ số thu được từ các số ở trên bằng cách bỏ đi chữ
số c (ở hàng đơn vị). Khi đó ( )mod16 16i jx c x c≡ ⇔/ không là ước của i jx c x c− tức
là i jx x− không chia hết cho 8
29
Nhưng trong 9 số 1 2 9, , ,x x x… chỉ có ba số lẻ , ,ac bc cc nên 8 số bất kỳ trong 9 số
1 2 9, , ,x x x… luôn có hai số có cùng số dư khi chia cho 8, mâu thuẫn.
Tương tự, trường hợp trong ba số a, b, c có hai số lẻ, một số chẵn cũng không xảy
ra
Ví dụ 6:
Cho 2011 điểm nằm trong mặt phẳng, trong đó không có 3 điểm nào thẳng hàng và
mọi tam giác có 3 đỉnh là 3 trong 2011 điểm đã cho đều có diện tích nhỏ hơn 1.
Chứng minh rằng có thể đặt 2011 điểm trên trong một tam giác có diện tích bằng 4.
(Đề khảo sát HSG Huyện Vĩnh Tường năm học 2011-2012)
Giải:
C
A
B
M N
P
D
K E
F
Giả sử tam giác ABC là tam giác có 3 đỉnh
trong 2011 diểm đã cho và có diện tích lớn nhất
=> SABC < 1. Qua A, B, C kẻ các đường thẳng
song song với các cạnh của tam giác ABC ta
được tam giác MNP có SMNP <4.
Ta chứng minh ko có điểm nào trong 2011 điểm
đã cho nằm ngoài tam giác MNP. Thật vậy: Giả
sử có điểm D nằm ngoài tam giác MNP
. Khi đó SDBC > SABC trái với cách chọn tam giác ABC có diện tích lớn nhất.
Suy ra 2011 điểm đã cho không nằm ngoài tam giác MNP có diện tích nhỏ hơn 4.
Vì vậy 2011 điểm đó nằm trong tam giác KEF đồng dạng với tam giác MNP(h.vẽ)
và có diện tích bằng 4.
Ví dụ 7:
Mỗi điểm của mặt phẳng được tô bởi một trong ba màu Đỏ, Xanh, Vàng.
Chứng minh rằng tồn tại hai điểm ,A B được tô bởi cùng một màu mà độ dài
1.=AB (Đề thi HSG Tỉnh Vĩnh Phúc năm học 2010 – 2011)
- Giả sử trái lại, với mọi cách tô, không tồn tại
hai điểm cùng màu mà có khoảng cách bằng 1.
Xét hai điểm , : 3M N MN = thì tồn tại các điểm ,P Q
sao cho các tam giác ,MPQ NPQ là các tam giác đều
có độ dài cạnh bằng 1.
Khi đó, do hai điểm có khoảng cách bằng 1 thì được
tô bởi hai màu khác nhau, nên ,M N phải được tô bởi
cùng một màu,
chẳng hạn tô P: Đỏ, Q: Vàng thì M, N: phải tô cùng màu Xanh, (Hình vẽ).
- Từ đó, nếu điểm M được tô màu Xanh, thì mọi điểm nằm trên đường tròn tâm M,
bán kính 3 đều được tô màu Xanh. Nhưng trên đường tròn này luôn có hai điểm
mà khoảng cách giữa chúng bằng 1. Mâu thuẫn với giả thiết phản chứng.
Từ đó suy ra điều phải chứng minh
30
(Vàng)
(Đỏ)
(Xanh)
(Xanh)
M N
P
Q
Ví dụ 8:
Trong bảng hình vuông gồm 10 10× ô vuông (10 hàng, 10 cột), người ta viết
vào các ô vuông các số tự nhiên từ 1 đến 100 theo cách như sau: ở hàng thứ nhất,
từ trái sang phải, viết các số từ 1 đến 10; ở hàng thứ hai, từ trái sang phải, viết các
số từ 11 đến 20; cứ như vậy cho đến hết hàng thứ 10. Sau đó cắt bảng hình vuông
thành những hình chữ nhật nhỏ kích thức 1 2× hoặc 2 1× . Tính tích số của hai số
trong mỗi hình chữ nhật nhỏ rồi cộng 50 tích lại. Cần phải cắt hình vuông như thế
nào để tổng tìm được nhỏ nhất ? Hãy tính giá trị nhỏ nhất đó.
(Đề KS HSG Huyện Vĩnh Tường năm học 2011-2012)
Giải:
Cắt hình vuông thành các hình chữ nhật cỡ 1 2× hoặc 2 1× thì được tất cả 50 hình.
Giả sử trong hình thứ k có 2 số ,k ka b thì hoặc 1k ka b− = hoặc 10.k ka b− =
Ta có ( )
22 2
2 2
k kk k
k k
a ba b
a b
−+
× = − suy ra ( ) ( )
50 50 50
22 2
1 1 1
1 1
2 2
k k k k k k
k k k
a b a b a b
= = =
× = + − −∑ ∑ ∑
Trong đó ( ) ( )
50
2 2 1 2 2
1
1 1 1 100 101 201
1 2 100 169175
2 2 2 6
k k
k
a b
=
× ×
+ = + + + = × =∑ L
và mỗi số ( )
2
k ka b− hoặc bằng 1 hoặc bằng 100.
Do đó, để tổng thu được là nhỏ nhất, thì ( )
2
100, 1,2,...,500.k ka b k− = ∀ =
Vì vậy, cần cắt hình vuông thành các hình chữ nhật với kích thước 2 1× . Và khi đó
giá trị nhỏ nhất của tổng bằng 169175 25 100 166675− × = .
Ví dụ 9: Cho đa giác lồi 1 2 100A A AK . Tại mỗi đỉnh kA ( 1,2,...,100k = ), người ta ghi
một số thực ka sao cho giá trị tuyệt đối của hiệu hai số trên hai đỉnh kề nhau
chỉ bằng 2 hoặc 3. Tìm giá trị lớn nhất có thể được của giá trị tuyệt đối của
hiệu giữa hai số ghi trên mỗi cặp đỉnh của đa giác đã cho, biết rằng các số ghi
tại các đỉnh đã cho đôi một khác nhau.
(Đề tuyển sinh THPT chuyên Vĩnh Phúc 2011-2012)
Xét đa giác lồi 1 2 100A A AK như hình vẽ. Khi đó
1 2k ka a +− = hoặc 1 3k ka a +− = ( 1,2,...,99k = ).
Không mất tính tổng quát, coi 1a là nhỏ nhất, na
là lớn nhất (dễ thấy 2n ≥ ). Đặt max i j
i j
d a a
≠
= −
khi đó 1nd a a= − . Ta sẽ chứng minh 149.d =
Nằm giữa 1, nA A , theo chiều kim đồng hồ có 2n − đỉnh và có 100 n− đỉnh, theo chiều
ngược kim đồng hồ. Hơn nữa giá trị tuyệt đối của hiệu giữa hai số kề nhau không
vượt quá 3. Do đó ( )1 1 2 2 3 1... 3 1n n nd a a a a a a a a n−= − ≤ − + − + + − ≤ −
và tương tự ta
có
( )3 100 1d n≤ − + . Suy ra
( ) ( )3( 1) 3(100 1) 300
150
2 2
n n
d
− + − +
≤ = =
31
150d = khi và chỉ khi hiệu giữa hai số ghi trên hai đỉnh kề nhau đúng bằng 3 hay ta
có 1 3, 1,2,...,99i ia a i+− = = ( )1 1 2
1 1 2
2
1,...,98
i i i i
i i i i
i i
a a a a
a a a a i
a a
+ + +
+ + +
+
− = −
⇒ − = − ⇒ = =
( ) ( )1 100 1 2 2 3 99 100 1 2 1 100 1 2... 99 99 3 99.3a a a a a a a a a a a a a a⇒ − = − + − + + − = − ⇒ − = − ⇒ =
Điều này không xảy ra suy ra 150d = không thỏa mãn.
Ta xây dựng một trường hợp cho 149d = như sau:
1 2 10, 2, 3k ka a a a −= = = +
với 53 52 12,3, ,52; 2, 3, 54,55, ,100k kk a a a a k−= … = − = − = …
Khi đó hiệu lớn nhất 53 1 149a a− = .
Các số 2 3 53, , ,a a a… có dạng 2 3t+ , các số 54 55 100, , ,a a a… có dạng 147 3k− .
Rõ ràng không tồn tại ,k t sao cho ( )2 3 147 3 3 145t k k l+ = − ⇔ + = ( ,k t ∈¢ ).
Suy ra điều phải chứng minh.
Ví dụ 10: Một số tự nhiên dương được gọi là số “Đẹp”, nếu nó là hợp số và không
chia hết cho 2, 3, 5, 7. Hỏi có tất cả bao nhiêu số tự nhiên “Đẹp” nhỏ hơn 2011
Giải:
Số “Đẹp” là hợp số, không chia hết cho 2,3,5,7, nên nó phải là tích của các số
nguyên tố lớn hơn 10.
Do 133
=2197> 2010 , nên số cần tìm có dạng a b× hoặc a b c× × với a,b, c là các số
nguyên tố 11 a b c≤ ≤ ≤ .
Xét số “Đẹp” dạng a b× với 11 a b≤ ≤ , a,b là các số nguyên tố :
Với a=11 thì 11 181b≤ ≤ , có 38 số
Với a=13 thì 13 151b≤ ≤ , có 31 số
Với a=17 thì 17 113b≤ ≤ , có 24 số
Với a=139 thì 19 103b≤ ≤ , có 20 số
Với a=23 thì 23 83b≤ ≤ , có 15 số
Với a=29 thì 29 71b≤ ≤ , có 12 số
Với a=31 thì 31 61b≤ ≤ , có 8 số
Với a=37 thì 37 53b≤ ≤ , có 5 số
Với a=41 thì 41 47b≤ ≤ , có 3 số
Với a=43 thì b=43, có 1 số
Xét số “Đẹp” dạng a b c× × với 11 13a b c≤ ≤ ≤ ≤ , a,b,c là các số nguyên tố
Có 3 số có dạng này đó là: 11.11.11=1331; 11.11.13=1573 ; 11.13.13=1859
Suy ra số các số “Đẹp” cần tìm là S = 38+31+24+20+15+12+8+5+3+1+3 = 160 số
Ví dụ 11:
Trong một hộp có 2014 viên sỏi. Có hai người tham gia trò chơi, mỗi người lần
lượt phải bốc ít nhất là 11 viên sỏi và nhiều nhất là 20 viên sỏi. Người nào bốc viên
sỏi cuối cùng sẽ thua cuộc. Hãy tìm thuật chơi để đảm bảo người bốc đầu tiên luôn
là người thắng cuộc.
GIẢI:
32
Để đảm bảo thắng cuộc, ở nước đi cuối cùng của mình người bốc sỏi đầu tiên phải
để lại trong hộp 11 viên sỏi. Ở nước đi trước đó phải để lại trong hộp:
11 (20 11) 42+ + = viên sỏi.
Suy ra người bốc sỏi đầu tiên phải đảm bảo trong hộp lúc nào cũng còn 11 31k+
viên sỏi.
Ta có (2014 11):31 64− = dư 19. Như vậy người bốc sỏi đầu tiên ở lần thứ nhất của
mình phải bốc 19 viên.
Tiếp theo, khi đối phương bốc k viên sỏi ( 1, 2, ..., 20k = ) thì người bốc sỏi đầu tiên
phải bốc 31 k− viên sỏi, cuối cùng sẽ để lại 11 viên sỏi cho đối phương.
Ví dụ 12: Có điền được hay không 100 số gồm 10 số -2, 10 số -1, 30 số 0, 40 số 1,
10 số 2 vào các ô của bảng ×10 10 (mỗi ô điền một số và gọi số ở hàng i tính từ
dưới lên trên và cột j tính từ trái sang phải là aij) sao cho thỏa mãn hai điều kiện:
i) Tổng các số trên các hàng, các cột đều bằng m;
ii) Tổng các số aij trong bảng thỏa mãn i  j  2 bằng 5m.
Không điền được.
Thật vậy, giả sử trái lại, điền được các số thỏa mãn.
Khi đó [ ]= × − + − + + + =
1
m 10.( 2) 10.( 1) 30.0 40.1 10.2 3
10
là một số lẻ.
Ta chia các ô của bảng thành 4 loại:
- Loại 1 gồm các ô ở hàng lẻ, cột lẻ.
- Loại 2 gồm các ô ở hàng lẻ, cột chẵn.
- Loại 3 gồm các ô ở hàng chẵn, cột lẻ.
- Loại 4 gồm các ô ở hàng chẵn, cột chẵn.
Kí hiệu Sk là tổng tương ứng trên tất cả các ô loại k. Khi đó
S1 + S2 là tổng các số trên tất cả các hàng lẻ, nên S1 + S2 = 5m.
S2 + S4 là tổng các số trên tất cả các cột chẵn, nên S2 + S4 = 5m.
Loại 1 và loại 4 đều gồm các ô mà i - j chẵn, do đó S1 + S4 = 5m.
Suy ra 2(S1 + S2 + S4) = 15m (1)
Do m lẻ, nên VP(1) lẻ.
Mà VT(1) chẵn: vô lí.
Do đó điều giả sử là sai.
Vậy không thể điền được các số thỏa mãn.
Ví dụ 13:
Có 40 học sinh trong một lớp đứng thành vòng tròn quay mặt vào tâm đường tròn
để tham gia trò chơi đếm số như sau: Mỗi học sinh đếm một trong dãy số tuần hoàn
1,2,1,2,1,2….lần lượt theo chiều kim đồng hồ bắt đầu từ học sinh A(lớp trưởng).
Nếu học sinh nào đếm số 2 thì phải rời ngay khỏi vòng tròn. Việc đếm cứ tiếp tục
như vậy cho đến khi chỉ còn 1 học sinh - Học sinh đó được coi là thắng cuộc. B là
một học sinh giỏi Toán, B đã tìm ngay được vị trí đứng để mình là người thắng
cuộc. Hỏi B đứng ở vị trí nào theo chiều kim đồng hồ kể từ A?
GIẢI:
33
Xét trường hợp lớp chỉ có 32 học sinh thì lớp trưởng ở vị trí thứ nhất sẽ luôn thắng
cuộc.
Như vậy, để B là người thắng cuộc thì B phải đứng ở vị trí thứ nhất sau khi loại
được (40 – 32) = 8 người.
Hay B phải đứng ở vị trí 8 x 2 + 1 = 17 theo chiều kim đồng hồ kể từ vị trí của lớp
trưởng A.
(GV có thể yêu cầu HS tổng quát hóa bài toán với n HS trong lớp. Nếu lớp có 2k
HS thì B có cơ hội thắng cuộc ko?)
34
BÀI TẬP TỔ HỢP CHỌN LỌC
Bài 1. Lát bàn cờ 8 x 8 bằng 21 quân trimino kích thước 1 x 3. Hỏi ô trống còn lại
có thể là ô nào ?
Bài 2. Cho n là số nguyên dương lớn hơn hay bằng 2. Kí hiệu A = {1, 2, …, n}.
Tập con B của tập A được gọi là 1 tập "tốt" nếu B khác rỗng và trung bình cộng
của các phần tử của B là 1 số nguyên. Gọi Tn là số các tập tốt của tập A. Chứng
minh rằng Tn – n là 1 số chẵn.
Bài 3. Trên bàn cờ vua kích thước 8x8 được chia thành 64 ô vuông đơn vị, người ta
bỏ đi một ô vuông đơn vị nào đó ở vị trí hàng thứ m và cột thứ n. Gọi S(m;n) là số
hình chữ nhật được tạo bởi một hay nhiều ô vuông đơn vị của bàn cờ sao cho
không có ô nào trùng với vị trí của ô bị xóa bỏ ban đầu. Tìm giá trị nhỏ nhất và giá
trị lớn nhất của S(m;n).
Bài 4. Ba nhóm đường thẳng song song chia mặt phẳng thành N miền. Hỏi số
đường thẳng của cả 3 nhóm ít nhất là bao nhiêu để N > 2011.
HD:Giả sử số đường thẳng của mỗi nhóm là p, q, r hãy chứng minh số miền tạo
thành không vượt quá: pq + qr + rp + p + q + r + 1.
Ý tưởng chính là: p đường thẳng song song chia mặt phẳng thành p+1 phần. Khi
kẻ thêm 1 đường thẳng không song song với p đường thẳng này thì sẽ tạo ra thêm
p+1 phần. Như thế, sau khi kẻ q đường thẳng nữa thì số phần là (p+1)(q+1).
Cuối cùng, khi đã có 2 họ p + q đường thẳng, khi kẻ thêm đường thẳng của họ thứ
3 sẽ cắt hai họ trên ở nhiều nhất p+q điểm, do đó tạo ra p+q+1 miền mới.
Suy ra số miền mới tối đa là (p+1)(q+1) + r(p+q+1).
Bài 5. Cho 9 điểm nguyên trên mặt phẳng tọa độ, không có 3 điểm thẳng hàng.
Chứng minh ta luôn chọn được 3 điểm thỏa mãn diện tích của tam giác tạo bởi
chúng là số chẵn.
Bài 6. Ta có 15 tấm thẻ được đánh số 1, 2, …, 15. Có bao nhiêu cách chọn ra một
số (ít nhất 1) tấm thẻ sao cho tất cả các số viết trên các tấm thẻ này đều lớn hơn
hoặc bằng số tấm thẻ được chọn.
Bài 7. Trên một đường thẳng nằm ngang, cho 2005 điểm được đánh dấu trắng hoặc
đen. Với mỗi điểm, xác định tổng tất cả các điểm trắng bên phải và điểm đen bên
trái của nó. Biết rằng, trong 2005 tổng trên có đúng một số xuất hiện số lẻ lần. Hãy
tìm tất cả các giá trị có thể có của số này.
Bài 8: Cho số nguyên n ≥ 2. Chứng minh rằng trong mọi họ gồm ít nhất 2n-1
+1 tập
hợp con không rỗng phân biệt của tập hợp {1, 2,…, n} đều tìm được ba tập hợp mà
một trong chúng là hợp của hai tập hợp còn lại.
Bài 9: Trong mặt phẳng cho 2011 điểm sao cho với ba điểm bất kỳ trong số các
điểm đó ta luôn tìm được hai điểm để đoạn thẳng được tạo thành có độ dài bé hơn
1. Chứng minh luôn tồn tại một hình tròn bán kính là 1 chứa không ít hơn 1006
điểm đã cho.
Bài 10. Cho n điểm trong mặt phẳng, trong đó không có ba điểm nào thẳng hàng,
biết rằng ba điểm bất kỳ trong n điểm đã cho tạo thành một tam giác có diện tích
35
không lớn hơn 1. Chứng minh rằng có thể phủ tất cả n điểm đã cho bằng một tam
giác có diện tích không lớn hơn 4.
Xét tam giác ABC có diện tích lớn nhất và vẽ tam giác DEF là tam giác nhận tam
giác ABC là tam giác trung bình. Khi đó DEF là tam giác cần tìm.
Bài 11. Có n đội bóng thi đấu vòng tròn 1 lượt. Hãy lập lịch thi đấu gồm n-1 vòng
đấu sao cho trong mỗi vòng, mỗi đội chỉ thi đấu nhiều nhất 1 trận.
Bài 12. Cho 2n+1 máy tính. Hai máy tính bất kỳ được nối với nhau bởi một sợi
dây. Chứng minh rằng có thể tô các máy tính và các sợi dây bằng 2n+1 màu sao
cho:
i) Các máy tính được tô màu khác nhau
ii) Các sợi dây xuất phát từ cùng một máy tính được tô màu khác nhau
iii) Hai máy tính và sợi dây nối chúng được tô màu khác nhau.
Xếp các máy tính lên đỉnh một 2n+1 giác đều A1A2…A2n+1 và tô màu lần lượt là 1,
2, …, 2n+1. Với mỗi i, Nối đường kính OAi và tô tất cả các cạnh và đường chéo
vuông góc với OAi bằng màu i. Ta sẽ được phép tô thỏa mãn điều kiện.
Bài 13: Có 6n+4 nhà toán học tham dự 1 hội nghị, trong đó có 2n+1 buổi thảo luận.
Mỗi buổi thảo luận đều có 1 bàn tròn cho 4 người ngồi và n bàn tròn cho 6 người
ngồi. Biết rằng 2 người bất kỳ không ngồi cạnh nhau hoặc đối diện nhau quá 1 lần.
a. Hỏi có thể thực hiện được không với n=1?
b. Hỏi có thể thực hiện được không với n>1?
Bài 14. Trong đường tròn đơn vị có 4 điểm sao cho khoảng cách giữa hai điểm bất
kỳ trong chúng lớn hơn 1. Chứng minh rằng có thể kẻ hai đường kính vuông góc
của đường tròn sao cho trong mỗi một góc 1/4 có đúng 1 trong 4 điểm đã cho.
Bài 15. Số nguyên dương A trong cách viết thập phân được gọi là số kỳ quặc nếu
tổng của A và số thu được từ A bằng cách viết theo thứ tự ngược lại là một số có tất
cả các chữ số đều lẻ. Hãy tìm số các số kỳ quặc có 3 chữ số.
Bài 16. Có ba lớp học A, B, C, mỗi lớp có 30 học sinh. Biết rằng một học sinh bất
kỳ đều quen với ít nhất 31 học sinh khác lớp. Chứng minh rằng tồn tại ba học sinh
a, b, c lần lượt thuộc lớp A, B, C sao cho họ đôi một quen nhau.
Chọn học sinh có số bạn quen nhiều nhất ở 1 lớp khác. Giả sử số bạn quen lớn
nhất đó là k. Giả sử đó là học sinh a ở lớp A và a quen với k học sinh ở lớp B. Do a
quen với ít nhất 31 học sinh và a ≤ 30 nên a quen với ít nhất một học sinh ở C, giả
sử đó là c. Theo định nghĩa của k, c quen không quá k học sinh ở A, do đó c quen
với ít nhất 31-k học sinh ở B. Vì tổng số người quen của a và c trong B ít nhất là k
+ 31-k = 31 nên a và c phải có ít nhất một người quen chung trong B. Gỉa sử đó là
b thì a, b, c là 3 học sinh cần tìm.
Bài 17. Phủ bàn cờ 8 x 8 bằng 21 quân trimino 1 x 3. Hỏi ô trống còn lại có thể là
ô nào?
36
HD: Gọi ε là số sao cho ε2
+ ε + 1 = 0 thì ta có ε3
= 1. Đánh số các cột và các
hàng theo thứ từ từ trái sang phải, từ dưới lên trên là 1, 2, …, 8. Ta điền vào ô (i, j)
số εi+j
. Khi đó dễ thấy tổng các số ở các ô mà một quân trimino 1 x 3 bất kỳ phủ sẽ
bằng 0. Như vậy, nếu phủ bàn cờ 8 x 8 bằng 21 quân domino thì tổng các số ở các
ô bị phủ bằng 0. Suy ra số còn lại bằng tổng tất cả các số được ghi. Ta có tổng các
số được ghi bằng
8 8 8 8
2
1 , 8 1 1 1 1
( ) 1i j i j i j
i j i j i j
ε ε ε ε ε ε+ +
≤ ≤ = = = =
  
= = = + = ÷ ÷
  
∑ ∑∑ ∑ ∑
Suy ra số ở ô còn lại phải là số 1. Nếu (i, j) là ô còn lại thì từ đó suy ra i + j chia
hết cho 3.
Làm tương tự như vậy nhưng ở ô (i, j) ghi số εi-j
ta cũng thu được i – j chia hết cho
3. Suy ra cả i và j đều chia hết cho 3. Như vậy (i, j) chỉ có thể là 1 trong 4 cặp (3,
3), (3, 6), (6, 3), (6, 6). Dễ dàng chỉ ra cách phủ cho các TH này (do tính đối xứng,
chỉ cần chỉ cho 1 trường hợp).
Bài 18. Xét số tự nhiên n ≥ 2. Bắt đầu các số 1, 2, …, 2n-1, 2n ta thực hiện các
phép biến đổi như sau: Chọn 2 số a, b sao cho a – b ≥ 2, xóa hai số này và thay
bằng hai số a – 1, b + 1; với bộ số thu được, ta lại thực hiện phép biến đổi tương tự,
và cứ như vậy.
a) Chứng minh rằng sau một số lần thực hiện các phép biến đổi như trên, ta
phải đạt đến trạng thái dừng, tức là không thể thực hiện được một phép
biến đổi nào nữa.
b) Gọi k là số phép biến đổi cần thực hiện để đạt đến trạng thái dừng. Hãy
tìm giá trị lớn nhất và giá trị nhỏ nhất của k.
Bài 19. Một ngũ giác lồi có tất cả các góc trong bằng nhau và các cạnh đều là các
số hữu tỷ. Chứng minh rằng ngũ giác đó là ngũ giác đều.
Bài 20. Hai người A và B cùng chơi một trò chơi. Ban đầu trên bàn có 100 viên
kẹo. Hai người thay phiên nhau bốc kẹo, mỗi lần được bốc k viên với k ∈ {1, 2,
6} . Hỏi ai là người có chiến thuật thắng, người đi trước hay người đi sau?
Bài 21. a) Trên bảng có số 2010. Hai người A và B cùng luân phiên thực hiện trò
chơi sau: Mỗi lần thực hiện, cho phép xoá đi số N đang có trên bảng và thay bằng
N-1 hoặc [N/2]. Ai thu được số 0 trước là thắng cuộc. Hỏi ai là người có chiến
thuật thắng, người đi trước hay người đi sau.
b) Cùng câu hỏi với luật chơi thay đổi như sau: Mỗi lần thực hiện, cho phép xoá
đi số N đang có trên bảng và thay bằng N-1 hoặc [(N+1)/2].
Bài 22. Hình tròn được bởi 5 đường kính thành thành 10 ô bằng nhau. Ban đầu
trong mỗi ô có 1 viên bi. Mỗi lần thực hiện, cho phép chọn 2 viên bi bất kỳ và di
chuyển chúng sang ô bên cạnh, 1 viên theo chiều kim đồng hồ và 1 viên ngược
chiều kim đồng hồ. Hỏi sau một số hữu hạn lần thực hiện, ta có thể chuyển tất cả
các viên bi về cùng 1 ô được không?
37
HD: Ta tô màu các ô bằng hai màu đen trắng xen kẽ nhau. Gọi S là tổng số viên bi
nằm ở các ô đen thì ở trạng thái ban đầu ta có S = 5. Nếu giả sử ngược lại rằng ta
có thể đưa các viên bi về cùng 1 ô thì ở trạng thái cuối cùng này, ta sẽ có S = 0
(nếu ta dồn các viên bi về một ô trắng) hoặc S = 10 (nếu ta dồn các viên bi về một
ô đen).
Bây giờ ta sẽ thu được điều mâu thuẫn nếu ta chứng minh được qua các lần thực
hiện thì tính chẵn lẻ của S sẽ không thay đổi, tức là nếu ban đầu S là số lẻ thì qua
các lần thực hiện, S sẽ luôn là số lẻ (và sẽ không thể bằng 0 hoặc bằng 10).
Nếu nhận xét rằng các ô đen trắng xen kẽ nhau thì điều mà chúng ta cần chứng
minh khá hiển nhiên và chúng tôi xin dành phép chứng minh chi tiết cho bạn đọc.
Bài 23. Có 2n người xếp thành 2 hàng dọc. Hỏi có bao nhiêu cách chọn ra một số
người (ít nhất 1) từ 2n người này, sao cho không có hai người nào đứng kề nhau
được chọn. Hai người đứng kề nhau là hai người có số thứ tự liên tiếp trong một
hàng dọc hoặc có cùng số thứ tự ở hai hàng.
Gọi Sn là số cách chọn ra một số người từ 2n người xếp thành 2 hàng dọc và Tn là
số cách chọn ra một số người từ 2n-1 người xếp thành 2 hàng dọc, trong đó khuyết
một chỗ ở đầu của một hàng. Ta có S1 = 2, T1 = 1.
1 3
4
Hình 1. Sn với n = 5
1 2
Hình 2. Tn với n = 5
Xét 2n người xếp thành 2 hàng dọc (như hình 1). Ta xét các cách chọn thoả mãn
điều kiện đầu bài. Xảy ra các khả năng sau :
1) Người ở vị trí số 1 được chọn : Khi đó người ở vị trí số 2 và số 3 không
được chọn  Có Tn-1 + 1 cách chọn (+1 là do bổ sung cách
chọn « không chọn gì cả » )
2) Người ở vị trí số 2 được chọn : Tương tự, có Tn-1 + 1 cách chọn.
3) Cả hai người ở vị trí số 1 và số 2 đều không được chọn: Có Sn-1 cách
chọn.
Vậy ta có Sn = Sn-1 + 2Tn-1+ 2 (1).
Xét 2n-1 người xếp thành 2 hàng dọc (như hình 2). Ta xét các cách chọn thoả mãn
điều kiện đầu bài. Xảy ra các khả năng sau :
1) Người ở vị trí số 1 được chọn : Khi đó người ở vị trí số 2 không được
chọn  có Tn-1 + 1 cách chọn
2) Người ở vị trí số 1 không được chọn : có Sn-1 cách chọn.
Vậy ta có Tn = Sn-1 + Tn-1 + 1 (2)
38
Từ (1) ta suy ra 2Tn-1 = Sn – Sn-1 – 2, 2Tn = Sn+1 – Sn – 2. Thay vào (2), ta được
Sn+1 – Sn – 2 = 2Sn-1+ Sn – Sn-1 – 2 + 2
Sn+1 = 2Sn + Sn-1 + 2
Từ đây dễ dàng tìm được
2
2)21()21( 11
−−++
=
++ nn
nS
Bài 24. Tìm số tất cả các bộ n số (x1, x2, …, xn) sao cho
(i) xi = ± 1 với i = 1, 2, …, n.
(ii) 0 ≤ x1 + x2 + … + xr < 4 với r = 1, 2, …, n-1 ;
(iii) x1 + x2 + … + xn = 4.
Bài 25. Trong một nhóm gồm 2n+1 người với mỗi n người tồn tại một người khác
n người này quen với tất cả họ. Chứng minh rằng trong nhóm người này có 1 người
quen với tất cả mọi người.
HD: Ta sẽ chứng minh trong nhóm người này có một nhóm n+1 người đôi một
quen nhau (gọi là nhóm A). Từ đó, xét n người còn lại. Theo giả thiết, có 1 người
trong nhóm A quen với n người này. Suy ra người này quen với tất cả mọi người.
Để chứng minh khẳng định về sự tồn tại của nhóm A, ta giả sử k là kích thước (số
người) lớn nhất của một nhóm người đôi một quen nhau. Ta cần chứng minh k ≥
n+1. Giả sử ngược lại k ≤ n. Theo giả thiết,tồn tại 1 người trong số những người
còn lại quen với k người này. Bổ sung người này vào nhóm, ta được nhóm gồm
k+1 người đôi một quen nhau. Mâu thuẫn với điều giả sử k lớn nhất.
Bài 26. Trong một đa giác lồi có chứa không ít hơn m2
+1 điểm nguyên. Chứng
minh rằng trong đa giác lồi này tìm được m+1 điểm nguyên cùng nằm trên một
đường thẳng.
Bài 27. Chứng minh rằng trong 9 người bất kỳ, hoặc có 3 người đôi một quen nhau,
hoặc
có 4 người đôi một không quen nhau.
Bài 28. Chọn ra 69 số nguyên dương từ tập hợp E = {1, 2, …, 100}. Chứng minh
rằng tồn tại 4 số a < b < c < d trong 4 số được chọn sao cho a + b + c = d. Kết
luận bài toán còn đúng không nếu ta thay 69 bằng 68?
HD: Giả sử các số đó là a1 < a2 < … < a69. Xét 2 tập hợp
A = {a2+a3, a2+a4, …., a2+a69}, B = {a3 – a1, …, a69 – a1} thì | A | = | B | = 67. Do
a69 – a2 ≥ 67 nên ta suy ra a2 ≤ a69 – 67 ≤ 100 – 67 = 33. Suy ra a2 + a69 ≤ 133.
Ngoài ra a3 – a1 ≥ 2. Do đây là các số lớn nhất của cả A và B nên ta có
A, B ⊂ {2, 3, …., 133} = C.
Vì | C | = 132 < 134 = |A| + |B| nên từ đây ta suy ra A ∩ B ≠ ∅. Suy ra tồn tại i, j
≥ 3 sao cho
a2 + ai = aj – a1. Rõ ràng i < j và như thế ta có a1 + a2 + ai = aj.
Dựa vào cách giải trên, có thể chỉ ra 68 số mà trong đó không tìm được 4 số thỏa
mãn đều bài, đó là:
39
33, 34, …, 100.
Do tổng ba số nhỏ nhất trong các số này đã là 33 + 34 + 35 = 102 > 100 là số lớn
nhất nên không tồn tại ba số a, b, c, d sao cho a + b + c = d.
Bài 29. Trong một nhóm n người có 3 người đôi một quen nhau và mỗi một người
này quen nhiều hơn 1 nửa số người trong nhóm. Tìm số ít nhất có thể số bộ ba
người đôi một quen nhau.
Bài 30. Các số 1, 2, …, n được viết trên bảng. Mỗi một phút, một học sinh lên
bảng, chọn hai số x và y, xóa chúng đi và viết lên bảng số 2x + 2y. Quá trình này
tiếp diễn cho đến khi trên bảng chỉ còn lại một số. Chứng minh rằng số này không
nhỏ hơn
3
4
9
n
.
Bài 31. Cho tập hợp A gồm các số nguyên dương có tính chất sau
(i) min A = 1, max A = 100.
(ii) Với mọi a thuộc A, a > 1 đều tồn tại b, c thuộc A (b không nhất thiết
khác c) sao cho a = b + c.
Tìm GTNN của | A |.
Bài 32. Trong hệ thống các tuyến xe buýt của thành phố, hai tuyến bất kỳ có chung
đúng một bến, và mỗi tuyến có ít nhất 4 bến. Chứng minh rằng có thể phân các bến
thành hai nhóm sao cho mỗi một tuyến xe buýt đều có bến thuộc cả hai nhóm.
Bài 33. Trong một giải cờ vua có 40 kỳ thủ. Có tổng cộng 80 ván đã được đấu, và
hai kỳ thủ bất kỳ đấu với nhau nhiều nhất một lần. Tìm số nguyên dương n lớn
nhất sao cho trong mọi trường hợp, ta đều tìm được n kỳ thủ chưa hề đấu với nhau.
Bài 34. Trò chơi lô-tô ở Lotoland được tổ chức như sau: Người chơi chọn 6 số khác
nhau từ các số 1, 2, 3, …, 36. Sau đó người ta sẽ bốc ra ngẫu nhiên 6 số từ các số
1, 2, 3, …, 36. Vé nào không chứa bất cứ số nào trong 6 số vừa bốc sẽ là vé thắng
giải. Chứng minh rằng tồn tại một cách mua 9 vé để có thể đảm bảo luôn có ít nhất
một vé thắng nhưng 8 vé thì nói chung không đủ để đảm bảo điều này.
Bài 35. Bàn cờ 12 x 12 được tô màu đen trắng như bình thường. Mỗi một lần thực
hiện ta có thể sơn lại hai ô cạnh nhau theo quy tắc: đen thành xanh, xanh thành
trắng, trắng thành đen. Hỏi ta cần phải dùng tối thiểu bao nhiêu bước để sơn bàn cờ
thành màu đen trắng ngược với bàn cờ ban đầu?
40
C. KẾT QUẢ NGHIÊN CỨU
Qua nghiên cứu giai đoạn 2 của chuyên đề tôi đã thu được một hệ thống các
bài tập trên và có được sự sáng tạo khi vận dụng nó trong chứng minh và khai thác
các bài toán tổ hợp.
Từ kết quả nghiên cứu đó được sự nhất trí Ban giám hiệu, tổ khoa học tự
nhiên. Chuyên đề nghiên cứu của tôi đã được thông qua tổ và áp dụng tại trường.
*) Giai đoạn 3: Luyện đề về dạng bài tổ hợp tổng hợp. Củng cố phương pháp
làm bài
+) Mục đích:
Nhằm nâng cao hiệu quả bồi dưỡng học sinh giỏi và bồi dưỡng học sinh thi
vào trung học phổ thông chuyên.
Trao đổi kinh nghiệm, lấy ý kiến đóng góp từ các đồng nghiệp để bổ sung và
hoàn thiện và phát triển chuyên đề.
+) Thời gian:
Từ tháng 2 năm 2013 đến tháng 4 năm 2013
+) Cách tiến hành:
Thông qua sinh hoạt chuyên môn báo cáo trước tổ, cụm
Trao đổi thảo luận và lấy ý kiến đóng góp của các đồng nghiệp
Áp dụng trực tiếp vào công tác bồi bưỡng học sinh giỏi.
+) Kết quả giai đoạn 3
Sau khi thông qua chuyên đề trước tổ tôi đã tiến hành xin ý kiến đóng góp và
nhận xét của đồng nghiệp và nhận được 100% giáo viên tán thành cách nghiên cứu
này
Trong quá trình áp dụng chuyên đề tôi đã tiến hành khảo sát với đối tượng là
25 học sinh giỏi. Qua kết quả khảo sát đó tôi thu được số học sinh biết vận dụng
nguyên lí Dirichlet và nguyên lí cực hạn phụ tăng lên rõ rệt, hơn nữa đã có em
bước đầu sáng tạo và tổng quát hóa về phương pháp và dạng bài tổ hợp. Kết quả cụ
thể là:
Giỏi Khá Trung bình yếu
6 10 7 2
B3
+) Phân tích kết quả
Từ kết quả điều tra trên tôi có thể khẳng định rằng:
Qua chuyên đề này thì các em học sinh đã biết cách trình bày, suy luận, khái
quát hóa cho loại bài toán tổ hợp.
Học sinh được phát triển tư duy và kỹ năng sáng tạo toán học. Học sinh tự
tin hơn khi gặp các bài toán hay và khó về toán rời rạc. Hình thành phương pháp
tìm lời giải bài toán, tư duy linh hoạt, phương pháp học toán một cách sáng tạo.
Giúp cho học sinh khá giỏi không những hình thành kỹ năng giải toán mà còn giúp
các em rèn luyện các thao tác tư duy: Phân tích, tổng hợp, khái quát hoá, đặc biệt
41
hoá. Hình thành ở các em cách học sáng tạo, qua đó giúp các em đã có phương
pháp tự học, tự nghiên cứu và phát triển khả năng tìm tòi, sáng tạo cho học sinh
Hơn nữa mỗi giáo viên thấy được rằng chỉ có lao động nghiêm túc, khoa học
mới có được hiệu quả cao. Từ đó nâng cao trình được trình độ chuyên môn và nâng
cao năng lực sư phạm của mình trong quá trình giảng dạy đặc biệt là trong công tác
bồi dưỡng học sinh giỏi.
Tóm lại, với phương pháp nghiên cứu này giúp người thầy nâng cao được tay
nghề của mình, xây dựng được hệ thống kiến thức cần có để định hướng cho học
sinh trong quá trình học. Song quan trọng hơn là gây hứng thú học tập bộ môn cho
học sinh, giúp các em có phương pháp học tập môn Toán một cách có hiệu quả.
42
D. KẾT LUẬN
Như vậy xuất phát từ nguyên lí Dirichlet được phát biểu thật đơn giản, bằng
những thao tác tư duy, sự linh hoạt, sáng tạo của mình. Chúng ta sẽ xây dựng
phương pháp và chứng minh được một số bài toán tổ hợp hay và khó. Thông qua
chuyên đề này không chỉ cung cấp cho học sinh một hệ thống các dạng bài mà còn
hình thành cho học kỹ năng phân tích bài toán để từ đó xây dựng và chứng minh
các bài toán tổ hợp tổng quát để áp dụng nhằm phát triển năng lực tư duy sáng tạo
toán học của học sinh, nhất là những học sinh khá giỏi.
Về lý luận: Rèn luyện khả năng tư duy sáng tạo, kỹ năng phân tích tổng hợp,
tính cẩn thận chính xác, tính kiên trì và sự linh hoạt trong vận dụng kiến thức đã
học và viải toán cho học sinh. Giúp các em có hứng thú học tập, ham mê học Toán
và đặc biệt là phát huy năng lực tư duy sáng tạo học sinh khi gặp các dạng toán
khó.
Về thực tiễn: Giúp học sinh vận dụng tốt nguyên lí Dirichlet và nguyên lí cực
hạn trong việc giải các bài toán rời rạc nói chung và hình học tổ hợp nói riêng.
Từ chỗ rất lúng túng khi gặp các bài toán tổ hợp thì nay phần lớn các em đã
tự tin hơn, biết vận dụng những kỹ năng được bồi dưỡng để giải thành thạo các bài
tập chứng minh mang tính phức tạp.
Trong khi viết chuyên đề “Sử dụng nguyên lí Dirichlet và nguyên lí cực hạn
trong các bài toán tổ hợp” tôi không tránh khỏi những thiếu sót cũng như nội dung
chuyên đề chưa thực sự phong phú. Rất mong các bạn đồng nghiệp và các em học
sinh đóng góp thêm các ý kiến để chuyên đề hoàn thiện và có hiệu quả hơn. Tôi xin
chân thành cám ơn.
Vĩnh Yên, ngày 10 tháng 1 năm 2014
Người viết chuyên đề
Trần Thị Phi Nga
43
CÁC TÀI LIỆU THAM KHẢO.
[1] Báo toán tuổi thơ, toán học và tuổi trẻ
[2] Phan Huy Khải (năm 2007) Các bài toán hình học tổ hợp.
[3] Trịnh Đình Long (năm 2006) Bài giảng cho học sinh chuyên toán- Trường
ĐHKHTN TP Hồ Chí Minh: Nguyên lí Dirichlet và các bài toán số học.
[4] Vũ Hữu Bình (năm 1998): Phương pháp giảng dạy môn toán - NXB GD
[5] Website http://baigiang.violet.vn/
[6] Website http://math.net.vn/
44

Más contenido relacionado

La actualidad más candente

10 dạng tích phân thường gặp thanh tùng
10 dạng tích phân thường gặp   thanh tùng10 dạng tích phân thường gặp   thanh tùng
10 dạng tích phân thường gặp thanh tùng
Trần Hà
 
Các phương pháp hay giải Phuong trinh-vo-ty
Các phương pháp hay giải Phuong trinh-vo-tyCác phương pháp hay giải Phuong trinh-vo-ty
Các phương pháp hay giải Phuong trinh-vo-ty
roggerbob
 
Chuyen de toan logic roi rac li thuyet to hop
Chuyen de toan logic  roi rac li thuyet to hopChuyen de toan logic  roi rac li thuyet to hop
Chuyen de toan logic roi rac li thuyet to hop
lephucduc06011999
 
Chuyên đề nhị thức newton và ứng dụng
Chuyên đề nhị thức newton và ứng dụngChuyên đề nhị thức newton và ứng dụng
Chuyên đề nhị thức newton và ứng dụng
Thế Giới Tinh Hoa
 
Chứng minh bất đẳng thức bằng phương pháp chọn điểm rơi. (1)
Chứng minh bất đẳng thức bằng phương pháp chọn điểm rơi. (1)Chứng minh bất đẳng thức bằng phương pháp chọn điểm rơi. (1)
Chứng minh bất đẳng thức bằng phương pháp chọn điểm rơi. (1)
Sao Băng Lạnh Giá
 
Chuyên đề phương tích và ứng dụng
Chuyên đề phương tích và ứng dụngChuyên đề phương tích và ứng dụng
Chuyên đề phương tích và ứng dụng
lovemathforever
 

La actualidad más candente (20)

Bai giang-toan-kinh-te-tin-hoc
Bai giang-toan-kinh-te-tin-hocBai giang-toan-kinh-te-tin-hoc
Bai giang-toan-kinh-te-tin-hoc
 
Bộ sưu tập bất đẳng thức của võ quốc bá cẩn
Bộ sưu tập bất đẳng thức của võ quốc bá cẩnBộ sưu tập bất đẳng thức của võ quốc bá cẩn
Bộ sưu tập bất đẳng thức của võ quốc bá cẩn
 
Bài tập tích phân suy rộng.
Bài tập tích phân suy rộng.Bài tập tích phân suy rộng.
Bài tập tích phân suy rộng.
 
Bìa tập đại số tổ hợp
Bìa tập đại số tổ hợpBìa tập đại số tổ hợp
Bìa tập đại số tổ hợp
 
Bt dai so hoang
Bt dai so hoangBt dai so hoang
Bt dai so hoang
 
10 dạng tích phân thường gặp thanh tùng
10 dạng tích phân thường gặp   thanh tùng10 dạng tích phân thường gặp   thanh tùng
10 dạng tích phân thường gặp thanh tùng
 
SỐ NGUYÊN - BÀI TẬP CƠ BẢN VÀ NÂNG CAO TOÁN LỚP 6
SỐ NGUYÊN - BÀI TẬP CƠ BẢN VÀ NÂNG CAO TOÁN LỚP 6SỐ NGUYÊN - BÀI TẬP CƠ BẢN VÀ NÂNG CAO TOÁN LỚP 6
SỐ NGUYÊN - BÀI TẬP CƠ BẢN VÀ NÂNG CAO TOÁN LỚP 6
 
Các phương pháp hay giải Phuong trinh-vo-ty
Các phương pháp hay giải Phuong trinh-vo-tyCác phương pháp hay giải Phuong trinh-vo-ty
Các phương pháp hay giải Phuong trinh-vo-ty
 
Bài toán liên quan về Phân số tối giản trong Toán lớp 6
Bài toán liên quan về Phân số tối giản trong Toán lớp 6Bài toán liên quan về Phân số tối giản trong Toán lớp 6
Bài toán liên quan về Phân số tối giản trong Toán lớp 6
 
Toán lớp 5 - Chuyên đề về phân số
Toán lớp 5 - Chuyên đề về phân sốToán lớp 5 - Chuyên đề về phân số
Toán lớp 5 - Chuyên đề về phân số
 
Chuyen de toan logic roi rac li thuyet to hop
Chuyen de toan logic  roi rac li thuyet to hopChuyen de toan logic  roi rac li thuyet to hop
Chuyen de toan logic roi rac li thuyet to hop
 
TUYỂN TẬP 13 CHUYÊN ĐỀ NÂNG CAO PHÁT TRIỂN VÀ BỒI DƯỠNG HSG TOÁN LỚP 6
TUYỂN TẬP 13 CHUYÊN ĐỀ NÂNG CAO PHÁT TRIỂN VÀ BỒI DƯỠNG HSG TOÁN LỚP 6TUYỂN TẬP 13 CHUYÊN ĐỀ NÂNG CAO PHÁT TRIỂN VÀ BỒI DƯỠNG HSG TOÁN LỚP 6
TUYỂN TẬP 13 CHUYÊN ĐỀ NÂNG CAO PHÁT TRIỂN VÀ BỒI DƯỠNG HSG TOÁN LỚP 6
 
Chuyên đề nhị thức newton và ứng dụng
Chuyên đề nhị thức newton và ứng dụngChuyên đề nhị thức newton và ứng dụng
Chuyên đề nhị thức newton và ứng dụng
 
Chứng minh bất đẳng thức bằng phương pháp chọn điểm rơi. (1)
Chứng minh bất đẳng thức bằng phương pháp chọn điểm rơi. (1)Chứng minh bất đẳng thức bằng phương pháp chọn điểm rơi. (1)
Chứng minh bất đẳng thức bằng phương pháp chọn điểm rơi. (1)
 
Chuyên đề phương tích và ứng dụng
Chuyên đề phương tích và ứng dụngChuyên đề phương tích và ứng dụng
Chuyên đề phương tích và ứng dụng
 
Bdt võ quốc bá cẩn
Bdt  võ quốc bá cẩnBdt  võ quốc bá cẩn
Bdt võ quốc bá cẩn
 
Ch3 ma tran
Ch3 ma tranCh3 ma tran
Ch3 ma tran
 
Slide bài giảng đầy đủ về phần mềm Geogebra
Slide bài giảng đầy đủ về phần mềm GeogebraSlide bài giảng đầy đủ về phần mềm Geogebra
Slide bài giảng đầy đủ về phần mềm Geogebra
 
Hinh hoc-affine
Hinh hoc-affineHinh hoc-affine
Hinh hoc-affine
 
kỹ thuật giải phương trình hàm
kỹ thuật giải phương trình hàmkỹ thuật giải phương trình hàm
kỹ thuật giải phương trình hàm
 

Similar a Cđ nguyên lí đi rich lê

Bvlv ngan-k22
Bvlv ngan-k22Bvlv ngan-k22
Bvlv ngan-k22
Lê Ngân
 
Ppdh noi dung tu nghien cuu nhom 2
Ppdh noi dung tu nghien cuu nhom 2Ppdh noi dung tu nghien cuu nhom 2
Ppdh noi dung tu nghien cuu nhom 2
transuong
 
Đồ án lý thuyết phương pháp dạy học tin học thầy Lê Đức Long
Đồ án lý thuyết phương pháp dạy học tin học thầy Lê Đức LongĐồ án lý thuyết phương pháp dạy học tin học thầy Lê Đức Long
Đồ án lý thuyết phương pháp dạy học tin học thầy Lê Đức Long
thaihoc2202
 
Nghien cuu khoa hoc nop
Nghien cuu khoa hoc nopNghien cuu khoa hoc nop
Nghien cuu khoa hoc nop
tranthikimngan
 
Ppdh noi dung tu nghien cuu nhóm 2
Ppdh noi dung tu nghien cuu nhóm 2Ppdh noi dung tu nghien cuu nhóm 2
Ppdh noi dung tu nghien cuu nhóm 2
gaunaunguyen
 
Bai tap dstt hv bcvt - bookbooming
Bai tap dstt   hv bcvt - bookboomingBai tap dstt   hv bcvt - bookbooming
Bai tap dstt hv bcvt - bookbooming
bookbooming
 

Similar a Cđ nguyên lí đi rich lê (20)

Mẫu nguyên tử bohr và quảng phổ của hidro
Mẫu nguyên tử bohr và quảng phổ của hidroMẫu nguyên tử bohr và quảng phổ của hidro
Mẫu nguyên tử bohr và quảng phổ của hidro
 
Kinh nghiệm hướng dẫn học sinh phương pháp giải bài tập di truyền môn sinh họ...
Kinh nghiệm hướng dẫn học sinh phương pháp giải bài tập di truyền môn sinh họ...Kinh nghiệm hướng dẫn học sinh phương pháp giải bài tập di truyền môn sinh họ...
Kinh nghiệm hướng dẫn học sinh phương pháp giải bài tập di truyền môn sinh họ...
 
Chuyên Đề Dạy Và Học Môn Giáo Dục Công Dân Theo Định Hướng Thi Trắc Nghiệm Kh...
Chuyên Đề Dạy Và Học Môn Giáo Dục Công Dân Theo Định Hướng Thi Trắc Nghiệm Kh...Chuyên Đề Dạy Và Học Môn Giáo Dục Công Dân Theo Định Hướng Thi Trắc Nghiệm Kh...
Chuyên Đề Dạy Và Học Môn Giáo Dục Công Dân Theo Định Hướng Thi Trắc Nghiệm Kh...
 
NGHIÊN CỨU VỀ PHƯƠNG PHÁP TỰ HỌC CỦA SINH VIÊN TRƯỜNG ĐẠI HỌC
NGHIÊN CỨU VỀ PHƯƠNG PHÁP TỰ HỌC CỦA SINH VIÊN TRƯỜNG ĐẠI HỌCNGHIÊN CỨU VỀ PHƯƠNG PHÁP TỰ HỌC CỦA SINH VIÊN TRƯỜNG ĐẠI HỌC
NGHIÊN CỨU VỀ PHƯƠNG PHÁP TỰ HỌC CỦA SINH VIÊN TRƯỜNG ĐẠI HỌC
 
Thực trạng kỹ năng thảo luận nhóm của học sinh trường thpt hòa vang
Thực trạng kỹ năng thảo luận nhóm của học sinh trường thpt hòa vangThực trạng kỹ năng thảo luận nhóm của học sinh trường thpt hòa vang
Thực trạng kỹ năng thảo luận nhóm của học sinh trường thpt hòa vang
 
Bvlv ngan-k22
Bvlv ngan-k22Bvlv ngan-k22
Bvlv ngan-k22
 
K2pi.net.vn --skkn-toan-thpt dang thuc hua(pkc) (1)
K2pi.net.vn --skkn-toan-thpt dang thuc hua(pkc) (1)K2pi.net.vn --skkn-toan-thpt dang thuc hua(pkc) (1)
K2pi.net.vn --skkn-toan-thpt dang thuc hua(pkc) (1)
 
SKKN Một số biện pháp giúp học sinh lớp 2 giải bài toán có lời văn theo mô hì...
SKKN Một số biện pháp giúp học sinh lớp 2 giải bài toán có lời văn theo mô hì...SKKN Một số biện pháp giúp học sinh lớp 2 giải bài toán có lời văn theo mô hì...
SKKN Một số biện pháp giúp học sinh lớp 2 giải bài toán có lời văn theo mô hì...
 
Ppdh noi dung tu nghien cuu nhom 2
Ppdh noi dung tu nghien cuu nhom 2Ppdh noi dung tu nghien cuu nhom 2
Ppdh noi dung tu nghien cuu nhom 2
 
Đồ án lý thuyết phương pháp dạy học tin học thầy Lê Đức Long
Đồ án lý thuyết phương pháp dạy học tin học thầy Lê Đức LongĐồ án lý thuyết phương pháp dạy học tin học thầy Lê Đức Long
Đồ án lý thuyết phương pháp dạy học tin học thầy Lê Đức Long
 
Toan a2 bai tap
Toan a2   bai tapToan a2   bai tap
Toan a2 bai tap
 
Luận văn thạc sĩ sư phạm sinh học.
Luận văn thạc sĩ sư phạm sinh học.Luận văn thạc sĩ sư phạm sinh học.
Luận văn thạc sĩ sư phạm sinh học.
 
Nghien cuu khoa hoc nop
Nghien cuu khoa hoc nopNghien cuu khoa hoc nop
Nghien cuu khoa hoc nop
 
Một số biện pháp nâng cao hiệu quả dạy học hóa học chương oxi lưu huỳnh lớp...
Một số biện pháp nâng cao hiệu quả dạy học hóa học chương oxi   lưu huỳnh lớp...Một số biện pháp nâng cao hiệu quả dạy học hóa học chương oxi   lưu huỳnh lớp...
Một số biện pháp nâng cao hiệu quả dạy học hóa học chương oxi lưu huỳnh lớp...
 
Một số biện pháp nâng cao hiệu quả dạy học hóa học chương oxi lưu huỳnh lớp...
Một số biện pháp nâng cao hiệu quả dạy học hóa học chương oxi   lưu huỳnh lớp...Một số biện pháp nâng cao hiệu quả dạy học hóa học chương oxi   lưu huỳnh lớp...
Một số biện pháp nâng cao hiệu quả dạy học hóa học chương oxi lưu huỳnh lớp...
 
Một số biện pháp nâng cao hiệu quả dạy học hóa học chương oxi lưu huỳnh lớp...
Một số biện pháp nâng cao hiệu quả dạy học hóa học chương oxi   lưu huỳnh lớp...Một số biện pháp nâng cao hiệu quả dạy học hóa học chương oxi   lưu huỳnh lớp...
Một số biện pháp nâng cao hiệu quả dạy học hóa học chương oxi lưu huỳnh lớp...
 
Một số biện pháp nâng cao hiệu quả dạy học hóa học chương oxi lưu huỳnh lớp...
Một số biện pháp nâng cao hiệu quả dạy học hóa học chương oxi   lưu huỳnh lớp...Một số biện pháp nâng cao hiệu quả dạy học hóa học chương oxi   lưu huỳnh lớp...
Một số biện pháp nâng cao hiệu quả dạy học hóa học chương oxi lưu huỳnh lớp...
 
Đề tài: Nâng cao hiệu quả dạy học hóa học chương Oxi-Lưu huỳnh
Đề tài: Nâng cao hiệu quả dạy học hóa học chương Oxi-Lưu huỳnhĐề tài: Nâng cao hiệu quả dạy học hóa học chương Oxi-Lưu huỳnh
Đề tài: Nâng cao hiệu quả dạy học hóa học chương Oxi-Lưu huỳnh
 
Ppdh noi dung tu nghien cuu nhóm 2
Ppdh noi dung tu nghien cuu nhóm 2Ppdh noi dung tu nghien cuu nhóm 2
Ppdh noi dung tu nghien cuu nhóm 2
 
Bai tap dstt hv bcvt - bookbooming
Bai tap dstt   hv bcvt - bookboomingBai tap dstt   hv bcvt - bookbooming
Bai tap dstt hv bcvt - bookbooming
 

Más de Cảnh

Tim chu so tan cung cua mot luy thua
Tim chu so tan cung cua mot luy thuaTim chu so tan cung cua mot luy thua
Tim chu so tan cung cua mot luy thua
Cảnh
 
Mot so phuong phap giai phuong trinh nghiem nguyen
Mot so phuong phap giai phuong trinh nghiem nguyenMot so phuong phap giai phuong trinh nghiem nguyen
Mot so phuong phap giai phuong trinh nghiem nguyen
Cảnh
 
Giai pt chua dau gia tri tuyet doi o thcs
Giai pt chua dau gia tri tuyet doi o thcsGiai pt chua dau gia tri tuyet doi o thcs
Giai pt chua dau gia tri tuyet doi o thcs
Cảnh
 
Giải 30 bài toán dãy số hay gặp
Giải 30 bài toán dãy số hay gặpGiải 30 bài toán dãy số hay gặp
Giải 30 bài toán dãy số hay gặp
Cảnh
 
đề Thi hsg toán 8 có đáp án
đề Thi hsg toán 8 có đáp ánđề Thi hsg toán 8 có đáp án
đề Thi hsg toán 8 có đáp án
Cảnh
 
Day cac phan so viet theo qui luat
Day cac phan so viet theo qui luatDay cac phan so viet theo qui luat
Day cac phan so viet theo qui luat
Cảnh
 
Chuyen de toan suy luan logic
Chuyen de toan suy luan logicChuyen de toan suy luan logic
Chuyen de toan suy luan logic
Cảnh
 
Chuyen de phuong trinh nghiem nguyen
Chuyen de phuong trinh nghiem nguyenChuyen de phuong trinh nghiem nguyen
Chuyen de phuong trinh nghiem nguyen
Cảnh
 
Chuyen de mon toan nguyen tac dirichlet
Chuyen de mon toan nguyen tac dirichletChuyen de mon toan nguyen tac dirichlet
Chuyen de mon toan nguyen tac dirichlet
Cảnh
 
Chuyen de day cac so viet theo quy luat
Chuyen de day cac so viet theo quy luatChuyen de day cac so viet theo quy luat
Chuyen de day cac so viet theo quy luat
Cảnh
 
Cđ van dung bdt giai pt hpt
Cđ van dung bdt giai pt hptCđ van dung bdt giai pt hpt
Cđ van dung bdt giai pt hpt
Cảnh
 
Cđ thuật toán tương tự trong bồi dưỡng hsg toán 8
Cđ thuật toán tương tự trong bồi dưỡng hsg toán 8Cđ thuật toán tương tự trong bồi dưỡng hsg toán 8
Cđ thuật toán tương tự trong bồi dưỡng hsg toán 8
Cảnh
 
Cđ ôxít axít tác dụng với bazơ
Cđ ôxít axít tác dụng với bazơCđ ôxít axít tác dụng với bazơ
Cđ ôxít axít tác dụng với bazơ
Cảnh
 
Cđ một số ứng dụng định lí mê nê la uýt và xê va
Cđ một số ứng dụng định lí mê nê la uýt và xê vaCđ một số ứng dụng định lí mê nê la uýt và xê va
Cđ một số ứng dụng định lí mê nê la uýt và xê va
Cảnh
 
Cđ một số kỹ thuật sd bđt cauchy
Cđ một số kỹ thuật sd bđt cauchyCđ một số kỹ thuật sd bđt cauchy
Cđ một số kỹ thuật sd bđt cauchy
Cảnh
 

Más de Cảnh (20)

Vận dụng hằng đẳng thức vào giải toán
Vận dụng hằng đẳng thức vào giải toánVận dụng hằng đẳng thức vào giải toán
Vận dụng hằng đẳng thức vào giải toán
 
Tông hợp hpt
Tông hợp hptTông hợp hpt
Tông hợp hpt
 
Tim chu so tan cung cua mot luy thua
Tim chu so tan cung cua mot luy thuaTim chu so tan cung cua mot luy thua
Tim chu so tan cung cua mot luy thua
 
So chinh phuong lớp 6
So chinh phuong lớp 6So chinh phuong lớp 6
So chinh phuong lớp 6
 
Mot so phuong phap giai phuong trinh nghiem nguyen
Mot so phuong phap giai phuong trinh nghiem nguyenMot so phuong phap giai phuong trinh nghiem nguyen
Mot so phuong phap giai phuong trinh nghiem nguyen
 
Kỹ thuật giải hpt
Kỹ thuật giải hptKỹ thuật giải hpt
Kỹ thuật giải hpt
 
Giai pt chua dau gia tri tuyet doi o thcs
Giai pt chua dau gia tri tuyet doi o thcsGiai pt chua dau gia tri tuyet doi o thcs
Giai pt chua dau gia tri tuyet doi o thcs
 
Giải 30 bài toán dãy số hay gặp
Giải 30 bài toán dãy số hay gặpGiải 30 bài toán dãy số hay gặp
Giải 30 bài toán dãy số hay gặp
 
đề Thi hsg toán 8 có đáp án
đề Thi hsg toán 8 có đáp ánđề Thi hsg toán 8 có đáp án
đề Thi hsg toán 8 có đáp án
 
Day cac phan so viet theo qui luat
Day cac phan so viet theo qui luatDay cac phan so viet theo qui luat
Day cac phan so viet theo qui luat
 
Chuyen de toan suy luan logic
Chuyen de toan suy luan logicChuyen de toan suy luan logic
Chuyen de toan suy luan logic
 
Chuyen de phuong trinh nghiem nguyen
Chuyen de phuong trinh nghiem nguyenChuyen de phuong trinh nghiem nguyen
Chuyen de phuong trinh nghiem nguyen
 
Chuyen de mon toan nguyen tac dirichlet
Chuyen de mon toan nguyen tac dirichletChuyen de mon toan nguyen tac dirichlet
Chuyen de mon toan nguyen tac dirichlet
 
Chuyen de day cac so viet theo quy luat
Chuyen de day cac so viet theo quy luatChuyen de day cac so viet theo quy luat
Chuyen de day cac so viet theo quy luat
 
Cđ van dung bdt giai pt hpt
Cđ van dung bdt giai pt hptCđ van dung bdt giai pt hpt
Cđ van dung bdt giai pt hpt
 
Cđ tìm lời giải hh9
Cđ tìm lời giải hh9Cđ tìm lời giải hh9
Cđ tìm lời giải hh9
 
Cđ thuật toán tương tự trong bồi dưỡng hsg toán 8
Cđ thuật toán tương tự trong bồi dưỡng hsg toán 8Cđ thuật toán tương tự trong bồi dưỡng hsg toán 8
Cđ thuật toán tương tự trong bồi dưỡng hsg toán 8
 
Cđ ôxít axít tác dụng với bazơ
Cđ ôxít axít tác dụng với bazơCđ ôxít axít tác dụng với bazơ
Cđ ôxít axít tác dụng với bazơ
 
Cđ một số ứng dụng định lí mê nê la uýt và xê va
Cđ một số ứng dụng định lí mê nê la uýt và xê vaCđ một số ứng dụng định lí mê nê la uýt và xê va
Cđ một số ứng dụng định lí mê nê la uýt và xê va
 
Cđ một số kỹ thuật sd bđt cauchy
Cđ một số kỹ thuật sd bđt cauchyCđ một số kỹ thuật sd bđt cauchy
Cđ một số kỹ thuật sd bđt cauchy
 

Último

SD-05_Xây dựng website bán váy Lolita Alice - Phùng Thị Thúy Hiền PH 2 7 8 6 ...
SD-05_Xây dựng website bán váy Lolita Alice - Phùng Thị Thúy Hiền PH 2 7 8 6 ...SD-05_Xây dựng website bán váy Lolita Alice - Phùng Thị Thúy Hiền PH 2 7 8 6 ...
SD-05_Xây dựng website bán váy Lolita Alice - Phùng Thị Thúy Hiền PH 2 7 8 6 ...
ChuThNgnFEFPLHN
 
Bài tập nhóm Kỹ Năng Gỉai Quyết Tranh Chấp Lao Động (1).pptx
Bài tập nhóm Kỹ Năng Gỉai Quyết Tranh Chấp Lao Động (1).pptxBài tập nhóm Kỹ Năng Gỉai Quyết Tranh Chấp Lao Động (1).pptx
Bài tập nhóm Kỹ Năng Gỉai Quyết Tranh Chấp Lao Động (1).pptx
DungxPeach
 
bài tập lớn môn kiến trúc máy tính và hệ điều hành
bài tập lớn môn kiến trúc máy tính và hệ điều hànhbài tập lớn môn kiến trúc máy tính và hệ điều hành
bài tập lớn môn kiến trúc máy tính và hệ điều hành
dangdinhkien2k4
 
26 Truyện Ngắn Sơn Nam (Sơn Nam) thuviensach.vn.pdf
26 Truyện Ngắn Sơn Nam (Sơn Nam) thuviensach.vn.pdf26 Truyện Ngắn Sơn Nam (Sơn Nam) thuviensach.vn.pdf
26 Truyện Ngắn Sơn Nam (Sơn Nam) thuviensach.vn.pdf
ltbdieu
 

Último (20)

SD-05_Xây dựng website bán váy Lolita Alice - Phùng Thị Thúy Hiền PH 2 7 8 6 ...
SD-05_Xây dựng website bán váy Lolita Alice - Phùng Thị Thúy Hiền PH 2 7 8 6 ...SD-05_Xây dựng website bán váy Lolita Alice - Phùng Thị Thúy Hiền PH 2 7 8 6 ...
SD-05_Xây dựng website bán váy Lolita Alice - Phùng Thị Thúy Hiền PH 2 7 8 6 ...
 
bài thi bảo vệ nền tảng tư tưởng của Đảng.docx
bài thi bảo vệ nền tảng tư tưởng của Đảng.docxbài thi bảo vệ nền tảng tư tưởng của Đảng.docx
bài thi bảo vệ nền tảng tư tưởng của Đảng.docx
 
Bài học phòng cháy chữa cháy - PCCC tại tòa nhà
Bài học phòng cháy chữa cháy - PCCC tại tòa nhàBài học phòng cháy chữa cháy - PCCC tại tòa nhà
Bài học phòng cháy chữa cháy - PCCC tại tòa nhà
 
Bài tập nhóm Kỹ Năng Gỉai Quyết Tranh Chấp Lao Động (1).pptx
Bài tập nhóm Kỹ Năng Gỉai Quyết Tranh Chấp Lao Động (1).pptxBài tập nhóm Kỹ Năng Gỉai Quyết Tranh Chấp Lao Động (1).pptx
Bài tập nhóm Kỹ Năng Gỉai Quyết Tranh Chấp Lao Động (1).pptx
 
Bài giảng môn Truyền thông đa phương tiện
Bài giảng môn Truyền thông đa phương tiệnBài giảng môn Truyền thông đa phương tiện
Bài giảng môn Truyền thông đa phương tiện
 
powerpoint mẫu họp phụ huynh cuối kì 2 học sinh lớp 7 bgs
powerpoint mẫu họp phụ huynh cuối kì 2 học sinh lớp 7 bgspowerpoint mẫu họp phụ huynh cuối kì 2 học sinh lớp 7 bgs
powerpoint mẫu họp phụ huynh cuối kì 2 học sinh lớp 7 bgs
 
các nội dung phòng chống xâm hại tình dục ở trẻ em
các nội dung phòng chống xâm hại tình dục ở trẻ emcác nội dung phòng chống xâm hại tình dục ở trẻ em
các nội dung phòng chống xâm hại tình dục ở trẻ em
 
TUYỂN TẬP ĐỀ THI GIỮA KÌ, CUỐI KÌ 2 MÔN VẬT LÍ LỚP 11 THEO HÌNH THỨC THI MỚI ...
TUYỂN TẬP ĐỀ THI GIỮA KÌ, CUỐI KÌ 2 MÔN VẬT LÍ LỚP 11 THEO HÌNH THỨC THI MỚI ...TUYỂN TẬP ĐỀ THI GIỮA KÌ, CUỐI KÌ 2 MÔN VẬT LÍ LỚP 11 THEO HÌNH THỨC THI MỚI ...
TUYỂN TẬP ĐỀ THI GIỮA KÌ, CUỐI KÌ 2 MÔN VẬT LÍ LỚP 11 THEO HÌNH THỨC THI MỚI ...
 
Access: Chuong III Thiet ke truy van Query.ppt
Access: Chuong III Thiet ke truy van Query.pptAccess: Chuong III Thiet ke truy van Query.ppt
Access: Chuong III Thiet ke truy van Query.ppt
 
30 ĐỀ PHÁT TRIỂN THEO CẤU TRÚC ĐỀ MINH HỌA BGD NGÀY 22-3-2024 KỲ THI TỐT NGHI...
30 ĐỀ PHÁT TRIỂN THEO CẤU TRÚC ĐỀ MINH HỌA BGD NGÀY 22-3-2024 KỲ THI TỐT NGHI...30 ĐỀ PHÁT TRIỂN THEO CẤU TRÚC ĐỀ MINH HỌA BGD NGÀY 22-3-2024 KỲ THI TỐT NGHI...
30 ĐỀ PHÁT TRIỂN THEO CẤU TRÚC ĐỀ MINH HỌA BGD NGÀY 22-3-2024 KỲ THI TỐT NGHI...
 
30 ĐỀ PHÁT TRIỂN THEO CẤU TRÚC ĐỀ MINH HỌA BGD NGÀY 22-3-2024 KỲ THI TỐT NGHI...
30 ĐỀ PHÁT TRIỂN THEO CẤU TRÚC ĐỀ MINH HỌA BGD NGÀY 22-3-2024 KỲ THI TỐT NGHI...30 ĐỀ PHÁT TRIỂN THEO CẤU TRÚC ĐỀ MINH HỌA BGD NGÀY 22-3-2024 KỲ THI TỐT NGHI...
30 ĐỀ PHÁT TRIỂN THEO CẤU TRÚC ĐỀ MINH HỌA BGD NGÀY 22-3-2024 KỲ THI TỐT NGHI...
 
TÀI LIỆU BỒI DƯỠNG HỌC SINH GIỎI LÝ LUẬN VĂN HỌC NĂM HỌC 2023-2024 - MÔN NGỮ ...
TÀI LIỆU BỒI DƯỠNG HỌC SINH GIỎI LÝ LUẬN VĂN HỌC NĂM HỌC 2023-2024 - MÔN NGỮ ...TÀI LIỆU BỒI DƯỠNG HỌC SINH GIỎI LÝ LUẬN VĂN HỌC NĂM HỌC 2023-2024 - MÔN NGỮ ...
TÀI LIỆU BỒI DƯỠNG HỌC SINH GIỎI LÝ LUẬN VĂN HỌC NĂM HỌC 2023-2024 - MÔN NGỮ ...
 
TUYỂN TẬP 50 ĐỀ LUYỆN THI TUYỂN SINH LỚP 10 THPT MÔN TOÁN NĂM 2024 CÓ LỜI GIẢ...
TUYỂN TẬP 50 ĐỀ LUYỆN THI TUYỂN SINH LỚP 10 THPT MÔN TOÁN NĂM 2024 CÓ LỜI GIẢ...TUYỂN TẬP 50 ĐỀ LUYỆN THI TUYỂN SINH LỚP 10 THPT MÔN TOÁN NĂM 2024 CÓ LỜI GIẢ...
TUYỂN TẬP 50 ĐỀ LUYỆN THI TUYỂN SINH LỚP 10 THPT MÔN TOÁN NĂM 2024 CÓ LỜI GIẢ...
 
bài tập lớn môn kiến trúc máy tính và hệ điều hành
bài tập lớn môn kiến trúc máy tính và hệ điều hànhbài tập lớn môn kiến trúc máy tính và hệ điều hành
bài tập lớn môn kiến trúc máy tính và hệ điều hành
 
26 Truyện Ngắn Sơn Nam (Sơn Nam) thuviensach.vn.pdf
26 Truyện Ngắn Sơn Nam (Sơn Nam) thuviensach.vn.pdf26 Truyện Ngắn Sơn Nam (Sơn Nam) thuviensach.vn.pdf
26 Truyện Ngắn Sơn Nam (Sơn Nam) thuviensach.vn.pdf
 
TÀI LIỆU BỒI DƯỠNG HỌC SINH GIỎI KỸ NĂNG VIẾT ĐOẠN VĂN NGHỊ LUẬN XÃ HỘI 200 C...
TÀI LIỆU BỒI DƯỠNG HỌC SINH GIỎI KỸ NĂNG VIẾT ĐOẠN VĂN NGHỊ LUẬN XÃ HỘI 200 C...TÀI LIỆU BỒI DƯỠNG HỌC SINH GIỎI KỸ NĂNG VIẾT ĐOẠN VĂN NGHỊ LUẬN XÃ HỘI 200 C...
TÀI LIỆU BỒI DƯỠNG HỌC SINH GIỎI KỸ NĂNG VIẾT ĐOẠN VĂN NGHỊ LUẬN XÃ HỘI 200 C...
 
kinh tế chính trị mác lênin chương hai và hàng hoá và sxxhh
kinh tế chính trị mác lênin chương hai và hàng hoá và sxxhhkinh tế chính trị mác lênin chương hai và hàng hoá và sxxhh
kinh tế chính trị mác lênin chương hai và hàng hoá và sxxhh
 
ĐỀ CHÍNH THỨC KỲ THI TUYỂN SINH VÀO LỚP 10 THPT CÁC TỈNH THÀNH NĂM HỌC 2020 –...
ĐỀ CHÍNH THỨC KỲ THI TUYỂN SINH VÀO LỚP 10 THPT CÁC TỈNH THÀNH NĂM HỌC 2020 –...ĐỀ CHÍNH THỨC KỲ THI TUYỂN SINH VÀO LỚP 10 THPT CÁC TỈNH THÀNH NĂM HỌC 2020 –...
ĐỀ CHÍNH THỨC KỲ THI TUYỂN SINH VÀO LỚP 10 THPT CÁC TỈNH THÀNH NĂM HỌC 2020 –...
 
Danh sách sinh viên tốt nghiệp Đại học - Cao đẳng Trường Đại học Phú Yên năm ...
Danh sách sinh viên tốt nghiệp Đại học - Cao đẳng Trường Đại học Phú Yên năm ...Danh sách sinh viên tốt nghiệp Đại học - Cao đẳng Trường Đại học Phú Yên năm ...
Danh sách sinh viên tốt nghiệp Đại học - Cao đẳng Trường Đại học Phú Yên năm ...
 
3-BẢNG MÃ LỖI CỦA CÁC HÃNG ĐIỀU HÒA .pdf - ĐIỆN LẠNH BÁCH KHOA HÀ NỘI
3-BẢNG MÃ LỖI CỦA CÁC HÃNG ĐIỀU HÒA .pdf - ĐIỆN LẠNH BÁCH KHOA HÀ NỘI3-BẢNG MÃ LỖI CỦA CÁC HÃNG ĐIỀU HÒA .pdf - ĐIỆN LẠNH BÁCH KHOA HÀ NỘI
3-BẢNG MÃ LỖI CỦA CÁC HÃNG ĐIỀU HÒA .pdf - ĐIỆN LẠNH BÁCH KHOA HÀ NỘI
 

Cđ nguyên lí đi rich lê

  • 1. SỞ GIÁO DỤC VÀ ĐÀO TẠO TỈNH VĨNH PHÚC PHÒNG GIÁO DỤC VÀ ĐÀO TẠO THÀNH PHỐ VĨNH YÊN BÁO CÁO CHUYÊN ĐỀ “NGUYÊN LÍ DIRICHLET VÀ NGUYÊN LÍ CỰC HẠN TRONG GIẢI TOÁN TỔ HỢP” Môn: TOÁN Tổ bộ môn: KHOA HỌC TỰ NHIÊN Mã: 30 Người thực hiện: TRẦN THỊ PHI NGA Điện thoại: 01686187936 Email: ngatran73@gmail.com Vĩnh Yên, năm 2014 1
  • 2. 2
  • 3. DANH MỤC CÁC TỪ VIẾT TẮTDANH MỤC CÁC TỪ VIẾT TẮT Stt Từ viết tắt Từ viết đầy đủ 1 BGH Ban giám hiệu 2 THCS trung học cơ sở 3 BĐT Bất đẳng thức 4 HSG học sinh giỏi 5 GV giáo viên 6 BDHSG Bồi dưỡng học sinh giỏi 7 GTLN Giá trị lớn nhất 8 GTNN Giá trị nhỏ nhất 3
  • 4. MỤC LỤCMỤC LỤC I - PHẦN MỞ ĐẦU 1. Lý do chọn đề tài 3 2. Mục đích nghiên cứu đề tài 3 3. Đối tượng và phạm vi nghiên cứu 4 4. Cơ sở lí thuyết và thực trạng của vấn đề 5 II. QUÁ TRÌNH THỰC HIỆN 5. Các bài toán sử dụng nguyên lí cực hạn 8 6. Sử dụng nguyên lí Dirichlet trong giải toán hình học tổ hợp 14 7. Một số dạng toán hình học tổ hợp thường gặp 21 8. Một số dạng bài tổng hợp khác 25 9. Bài tập tổ hợp chọn lọc 33 III – KẾT QUẢ NGHIÊN CỨU 39 III - PHẦN KẾT LUẬN 41 Tài liệu tham khảo 42 4
  • 5. PHẦN THỨ NHẤT: MỞ ĐẦU 1. Lý do viết đề tài Trong các kỳ thi học sinh giỏi các cấp tôi thấy rằng học sinh thường mất điểm khi không giải được các bài tập tổ hợp. Nhiều học sinh cho rằng đó là bài tập mà các em thường không giải được, do tính chất đặc thù của loại toán mang tính tư duy và trừu tượng cao. Vì vậy học sinh thường mất nhiều thời gian hoặc không làm được loại bài này. Qua nhiều năm dạy đội tuyển học sinh giỏi (HSG) tôi rất trăn trở và suy nghĩ mình phải làm thế nào để học sinh yêu thích giải các bài tập bài tập tổ hợp hơn. Vì nếu các em có phương pháp giải các bài tập đó một cách thành thạo thì việc tư duy và thuật toán để giải các loại bài tập khác sẽ nhanh nhẹn hơn, giúp các em có thể đạt được kết quả cao trong các kỳ thi học sinh giỏi các cấp. Do vậy tôi mạnh dạn viết chuyên đề “Sử dụng nguyên lí Dirichlet và nguyên lí cực hạn trong các bài toán tổ hợp”. Nhằm giúp các em có cách nhìn tổng quát và những suy nghĩ để mở rộng các kiến thức đã học từ những bài toán đơn giản đã học ở lớp 6. Từ đó các em tự vận dụng và phát triển tư duy với các bài tập tương tự, tổng quát và liên hệ một cách lô-gic với các dạng toán đã học. Qua thực tế giảng dạy bồi dưỡng học sinh giỏi nhiều năm, với cách làm trên tôi thấy rằng học sinh của tôi đã bắt đầu yêu thích các bài tập tổ hợp, các chuyên đề về bài tập tổ hợp đã lôi cuốn học sinh học tập say mê hơn. Từ đó tôi thấy rằng trong các kỳ thi học sinh giỏi nếu làm được bài tập tổ hợp là chúng ta có niềm tin rằng chất lượng đội tuyển sẽ được nâng lên. 1.2. Lý do chủ quan Đối với các em học sinh, dạng toán tổ hợp (Suy luận lôgic) tuy đã được tiếp xúc từ chương trình BDHSG của Tiểu học. Xong nó vẫn chỉ là một chương trình lồng ghép một cách nhẹ nhàng khi BDHSG, tuy đã được học ở các lớp học trước, nhưng các kỹ năng vận dụng để giải loại bài này của các em vẫn còn chưa đạt hiệu quả cao. Trong quá trình giảng dạy và BDHSG ở trường THCS chúng tôi, tôi nhận thấy dạng toán tổ hợp là một loại bài xuất hiện thường xuyên trong các đề thi HSG ở mọi lớp học hay cấp học. Tuy nhiên, khi tiếp xúc với dạng bài này HS thường ngại ngần và khó xuất phát để làm bài. Xuất phát từ thực trạng trên tôi chọn đề tài “Sử dụng nguyên lí Dirichlet và nguyên lí cực hạn trong các bài toán tổ hợp” cho chuyên đề của mình. 2. Mục đích nghiên cứu Trong chuyên đề này trước hết nhằm củng cố cho học sinh lý thuyết về nguyên lí Dirichlet và nguyên lí cực hạn. Cung cấp cho học sinh một số bài toán cụ thể và cách tổng quát hóa dạng bài thông qua từng ví dụ. Giúp cho học sinh có kĩ năng phân loại bài và phương pháp làm từng loại bài cụ thể ấy.Từ đó rèn cho học sinh tư duy linh hoạt, sáng tạo trong giải toán. Học sinh thấy được vai trò và ứng dụng rộng rãi của nguyên lí Dirichlet và nguyên lí cực hạn. Cũng thông qua đề tài này nhằm giúp học sinh có thói quen tìm tòi trong học toán và sáng tạo khi giải toán.Từ đó tạo cho học sinh có phương pháp 5
  • 6. học tập đúng đắn, biến cái đã học (kiến thức của thầy) thành cái của bản thân, nắm bắt nó, vận dụng nó, phát triển nó đúng hướng. Qua đó giúp các em tạo niềm tin, hưng phấn, hứng thú và say mê học môn toán học. Sử dụng làm tài liệu tham khảo cho giáo viên và học sinh. 1.4 Đối tượng, phạm vi nghiên cứu: +) Đối tượng nghiên cứu: Học sinh giỏi lớp 6, 7, 8, 9 và học sinh luyện thi THPT chuyên. +) Phạm vi nghiên cứu: Nguyên lí Dirichlet và nguyên lí cực hạn. Các bài tập cơ bản và nâng cao về nguyên lí Diirchlet và nguyên lí cực hạn trong chương trình trung học cơ sở. 1.5 Phương pháp nghiên cứu: +) Phương pháp nghiên cứu lý thuyết Đọc và nghiên cứu tài liệu, giáo trình về phương pháp dạy học toán, các tài liệu có liên quan đến nguyên lí Dirichlet và nguyên lí cực hạn cùng ứng dụng của nó. +) Phương pháp điều tra Tìm hiểu thực trạng dạy chuyên đề và bồi dưỡng học sinh giỏi của giáo viên đồng thời tìm hiểu kết quả học tập của học sinh nhằm xác định tính phổ biến và nguyên nhân để chuẩn bị cho các bước nghiên cứu tiếp theo. +) Phương pháp thảo luận Trao đổi với đồng nghiệp về kinh nghiệm giảng dạy và kĩ thuật vận dụng nguyên lí Dirichlet và nguyên lí cực hạn . +) Phương pháp quan sát Thông qua các tiết dự giờ thao giảng và bồi dưỡng học sinh giỏi của đồng nghiệp để quan sát trực tiếp tình hình học sinh tiếp thu bài và cách khai thác và xây dựng các bất đẳng thức phụ của giáo viên. +) Phương pháp kiểm tra đánh giá Khi thực hiện chuyên đề khảo sát so sánh kết quả đánh giá học sinh qua từng giai đoạn để đánh giá hiệu quả của chuyên đề. 1.6 Tình hình nghiên cứu Trong quá trình giảng dạy bộ môn Toán đặc biệt là trong công tác bồi dưỡng học sinh giỏi ở trường trung học cơ sở tôi thấy bài toán tổ hợp nói chung và vận dụng nguyên lí Dirichlet và nguyên lí cực hạn nói riêng là một trong những nội dung rất quan trọng. Vấn đề này đã có rất nhiều tài liệu tham khảo đề cập đến và cũng có rất nhiều giáo viên quan tâm nghiên cứu ở những mức độ khác nhau. Kết quả họ cũng có được những thành công nhất định. Song việc thực hiện được kết quả như thế nào còn tùy thuộc vào nhiều yếu tố. 1.7 Những vấn đề còn đang tồn tại: 6
  • 7. Khi chuẩn bị thực hiện chuyên đề này, kĩ năng giải toán tổ hợp của học sinh còn gặp nhiều khó khăn. Đặc biệt là các bài toán nguyên lí Dirichlet và nguyên lí cực hạn. Vì thế các em rất thụ động trong các buổi học bồi dưỡng về nội dung này. Các em học sinh mới chỉ vận dụng được nguyên lí Dirichlet và nguyên lí cực hạn với những bài toán đơn giản. Các tài liệu tham khảo về nội dung này mới nêu ra bài toán cụ thể với các bất ví dụ cụ thể mà chưa có nhiều tài liệu đề cập đến kĩ năng vận dụng nguyên lí Dirichlet và nguyên lí cực hạn trong giải toán tổ hợp. 1.8 Ứng dụng trong thực tiễn: Chuyên đề có ứng dụng tốt trong công tác bồi dưỡng học sinh giỏi và công tác ôn thi vào các trường trung học phổ thông chuyên. Chuyên đề được phổ biến rộng ở các trường trung học cơ sở trọng điểm trong Thành phố và trong Tỉnh. Chuyên đề còn là một tư liệu tốt để giáo viên và học sinh tham khảo. B. CƠ SỞ LÍ THUYẾT VÀ THỰC TRẠNG CỦA VẤN ĐỀ 2.1 Cơ sở lý luận của vấn đề nghiên cứu: Khi gặp các bài toán về nguyên lí Dirichlet và nguyên lí cực hạn thường liên quan nhiều đến các đối tượng là các tập hợp hữu hạn. Vì lẽ đó, các bài toán này mang mang đặc trưng rõ nét của toán học rời rạc. Khi giải toán tổ hợp vấn đề cơ bản là xác định được dạng bài và phương pháp làm cho dạng bài ấy. Từ đó HS áp dụng cho từng bài cụ thể một cách linh hoạt với các suy luận hợp lý để giải bài toán. 2.2 Thực trạng vấn đề nghiên cứu của chuyên đề. Trong chương trình toán trung học cơ sở thì nguyên lí Dirichlet và nguyên lí cực hạn không được học trong chương trình học chính khóa. Tuy nhiên trong các kỳ thi, đặc biệt là kỳ thi học sinh giỏi các cấp thì nguyên lí Dirichlet và nguyên lí cực hạn lại được đề cập đến rất nhiều bằng những bài toán hay và khó, đòi hỏi học sinh phải thực sự linh hoạt, sáng tạo và có kỹ năng sử dụng thành thạo trong suy luận thì mới gải được loại toán này. Trong các đề thi HSG, loại bài tổ hợp là rất khó đối với học sinh. Nó không những khó về biến đổi, khó về suy luận mà còn rất đa dạng về dạng bài và phong phú về nội dung. Từ thực tế bồi dưỡng đội tuyển học sinh giỏi, tôi nhận thấy bài toán tổ hợp mà cụ thể là nguyên lí Dirichlet và nguyên lí cực hạn không những đa dạng về dạng bài, phong phú về nội dung mà còn là một dạng bài toán khó, luôn gây không ít khó khăn cho học sinh. Vậy vấn đề đặt ra ở đây là phải làm sao để tìm ra biện pháp khắc phục thực trạng trên giúp giáo viên có được một tài liệu tham khảo phù hợp đặc biệt giúp học sinh hết lúng túng và tự tin hơn khi gặp bài toán tổ hợp. Tôi đã mạnh dạn đưa vấn đề này ra trong buổi sinh hoạt tổ chuyên môn của tổ Toán để cùng các đồng nghiệp thảo luận và đưa ra hướng giải quyết. 7
  • 8. Chương 3: QUÁ TRÌNH THỰC HIỆN Xuất phát từ những thực trạng trên và nhằm đáp ứng được yêu cầu hiệu quả của công tác bồi dưỡng học sinh giỏi tôi đã tìm hiểu, nghiên cứu áp dụng chuyên đề này vào thực tế công tác bồi dưỡng học sinh giỏi. Quá trình nghiên cứu chuyên đề tôi chia thành ba giai đoạn nghiên cứu như sau: Giai đoạn 1: Phân dạng bài và xây dựng phương pháp Giai đoạn 2: Xây dựng, hệ thống, chứng minh và áp dụng các bài toán tổ hợp. Giai đoạn 3: Luyện đề về dạng bài tổ hợp tổng hợp. Củng cố phương pháp làm bài 3.1 Giai đoạn 1: Phân dạng bài và xây dựng phương pháp +) Mục đích: Nhằm thu thập thông tin về tài liệu, giáo viên, học sinh với vấn đề nghiên cứu +) Thời gian: Từ tháng 09 đến tháng 11 năm 2012 +) Cách tiến hành: Bước 1: Đọc và nghiên cứu các tài liệu về các bài toán tổ hợp Bước 2: Thực hiện dự giờ bồi dưỡng học sinh giỏi của giáo viên đang bồi dưỡng học sinh về dạng toán tổ hợp như thế nào? Bước 3: Thảo luận, trao đổi với đồng nghiệp về cách dạy và vận dụng nguyên lí Dirichlet và nguyên lí cực hạn để xây dựng phương pháp giảng dạy và giải bài Bước 4: Kiểm tra sự vận dụng của học sinh +) Kết quả giai đoạn 1 Về tài liệu: Có rất nhiều tài liệu viết về nội dụng này, trong đó đã có những tài liệu viết khá chi tiết nhưng với số lượng rất ít. Chủ yếu các tài liệu mới chỉ đưa ra bài tập và nêu cách chứng minh nó. Với giáo viên: Thông qua dự giờ thăm lớp tôi nhận thấy số lượng giáo viên đã giảng dạy cho học sinh dạng toán tổ hợp theo chuyên đề là vẫn ít. Hầu hết giáo viên mới chỉ đưa ra các bài toán và cách giải cụ thể bài toán đó chứ không đi theo một hệ thống bài toán. Với học sinh: Còn khá lúng túng khi gặp các bài toán về tổ hợp. Do vậy kết quả giải bài tập của học sinh ở dạng toán này còn chưa tốt. Đặc biệt là có rất ít học sinh có sự sáng tạo trong khai thác bài toán. Tôi đã tiến hành khảo sát tổng số là 25 học sinh giỏi và thu được kết quả cụ thể sau: Giỏi Khá Trung bình yếu 1 2 5 17 Bảng 1 3.2 Giai đoạn 2: Luyện đề về dạng bài tổ hợp tổng hợp. Củng cố phương pháp làm bài +) Mục đích: Nhằm cung cấp cho giáo viên, học sinh hệ thống một số dạng bài và phương pháp giải từng loại bài ấy. 8
  • 9. Rèn tư duy sáng tạo, linh hoạt trong vận dụng và khai thác các kiến thức toán học cho học sinh. +) Thời gian: Từ tháng 12 năm 2012 đến tháng 2 năm 2013 +) Cách tiến hành: Sưu tầm và nghiên cứu một số bài toán có nội dung phù hợp với mục đích của chuyên đề nghiên cứu. +) Kết quả giai đoạn 2 Tôi đã tiến hành khảo sát tổng số là 25 học sinh giỏi và thu được kết quả cụ thể sau: Giỏi Khá Trung bình yếu 2 3 8 12 9
  • 10. NỘI DUNG CHUYÊN ĐỀ CÁC BÀI TOÁN SỬ DỤNG NGUYÊN LÍ CỰC HẠN A- Lý thuyết chung • Nguyên lí cực hạn có dạng đơn giản như sau: Nguyên lí 1: Trong một tập hợp hữu hạn và khác rỗng các số thực luôn luôn có thể chọn được số bé nhất và số lớn nhất. Nguyên lí 2: Trong một tập hợp khác rỗng các số tự nhiên luôn luôn có thể chọn được số bé nhất. Nguyên lí này dùng để giải các bài toán mà trong tập hợp có các đối tượng phải xét của nó tồn tại các đối tượng có GTLN, GTNN theo một nghĩa nào đó. Nguyên lí cực hạn thường được sử dụng kết hợp với các phương pháp khác đặc biệt là phương pháp phản chứng. Nguyên lí này được vận dụng trong trường hợp tập các giá trị cần khảo sát là tập hữu hạn (Nguyên lí 1) hoặc vô hạn nhưng tồn tại GTLN hoặc GTNN (Nguyên lí 2). Để vận dụng được nguyên lí cực hạn giải các bài tập hình học tổ hợp, người ta thường dùng một lược đồ chung để giải bài tập như sau: - Đưa bài toán đang xét về dạng sử dụng nguyên lí 1 hoặc nguyên lí 2 để chứng tỏ rằng tất cả các giá trị cần khảo sát của bài toán có GTLN hoặc GTNN. - Xét bài toán tương ứng khi nó nhận GTNN hoặc GTLN này. - Chỉ ra một mâu thuẫn hoặc đưa ra giá trị lớn hơn hoặc nhỏ hơn GTLN hoặc GTNN mà ta đang khảo sát. Theo nguyên lí của PP phản chứng ta suy ra điều phải chứng minh. B- Vận dụng Ví dụ 1: Chứng minh rằng bốn đường tròn có đường kính là bốn cạnh của một tứ giác lồi thì phủ kín tứ giác đã cho Lấy M là một điểm tùy ý trên tứ giác lồi. Có hai khả năng xảy ra 1) Nếu M nằm trên đường biên của tứ giác lồi, tức là M nằm trên một cạnh của tứ giác ABCD. Khi đó M nằm trong đường tròn có đường kính là cạnh ấy. Trong trường hợp này kết luận của bài toán hiển nhiên đúng. 2) Nếu M nằm bên trong tứ giác lồi ABCD. Khi đó ta có ∠ AMB + ∠ BMC + ∠ CMD + ∠ DMA = 3600 . Theo nguyên lý cực hạn tồn tại max { }, , ,AMB BNC CMD DMA BMC∠ ∠ ∠ ∠ = ∠ . Khi đó ∠ BMC ≥ 900 (1). Từ (1) suy ra M nằm trong hoặc cùng lắm là nằm trên đường tròn đường kính BC. Vậy dĩ nhiên M bị phủ bởi đường tròn này. Như thế do M là điểm tùy ý của tứ giác ABCD, ta suy ra bốn hình tròn phủ kín tứ giác lồi đã cho. Đó là điều phải chứng minh. 10
  • 11. Ví dụ 2: Cho ABC là tam giác nhọn. Lấy một điểm P bất kỳ trong tam giác. Chứng minh rằng khoảng cách lớn nhất trong các khoảng cách từ P tới ba đỉnh A, B, C của tam giác không nhỏ hơn hai lần khoảng cách bé nhất trong caccs khoảng cách từ P tới ba cạnh của tam giác đó. Gọi 1 1 1, ,A B C tương ứng là hình chiếu của P xuống BC, AC, AB. Ta có ( )0 1 1 1 1 1 1 360 1APC C PB BPA A PC CPB B PA∠ + ∠ + ∠ + ∠ + ∠ + ∠ = Theo nguyên lý cực hạn, tồn tại { }1 1 1 1 1 1ax , , , , ,m APC C PB BPA A PC CPB B PA∠ ∠ ∠ ∠ ∠ ∠ Không giảm tổng quát, cho là : { }1 1 1 1 1 1 1ax , , , , ,m APC C PB BPA A PC CPB B PA BPA∠ ∠ ∠ ∠ ∠ ∠ = ∠ (2) Từ (1) và (2) dễ dàng suy ra ∠ BPA1 ≥ 600 (3) Từ (3) ta đi đến 1 1 1 os 2 PA c BPA PB ∠ = ≤ hay PB ≥ 2PA1 (4) Từ (4) suy ra { } 1ax , , 2m PA PB PC PB PA≥ ≥ ≥ { }1 1 12min , ,PA PB PC . Đó là điều pcm. Ví dụ 3: Trên mặt phẳng có một số điểm có tinh chất là cứ với hai điểm bất kỳ của hệ điểm luôn tìm được điểm thứ ba trong số các điểm này thẳng hàng với chúng. Chứng minh rằng tất cả các điểm cảu hệ điểm thẳng hàng. Giả sử kết luận của bài toán không đúng, tức là các điểm đã cho không thẳng hàng. Xét tập hợp sau đây { / 0A h h= > và h là khoảng cách từ một điểm đã cho đến một đường thẳng nối hai điểm của hệ } Do giả thiết phản chứng nên A ≠Ø. Mặt khác, A là tập hợp có hữu hạn phần tử ( do có một số hữu hạn điểm đã cho). Theo nguyên lý cực hạn, tồn tại mọt giá trị nhỏ nhất h*. Giả sử h* là khoảng cách từ một điểm M xuống một đường thẳng đi qua B,C ( Ở đây M,B, C thuộc vào số các điểm đã cho). Gọi ∆ là đường thẳng nối B, C. Do M∉∆ ( vì h* > 0), nên theo giả thiết tồn tại điểm D∈∆. Kẻ MH ⊥∆, thì MH = d*. Rõ ràng trong ba điểm B, C, D pahir có hai điểm cùng phía so với H Không làm giảm tính tổng quát, ta có thể cho là C, D nằm cùng phía với H và C nằm trong đoạn HD, Kẻ HE ⊥ MD và CF ⊥ MD. Rõ ràng ta có : CF < HE < MH. Nói cách khác CF < d*. Chú ý rằng cho C,M,D cùng nằm trong các điểm đã cho, nên CF ∈ A. Do đó CF < d*. Điều này mâu thuẫn với định nghĩa của d*. Vậy giải thiết phản chứng là sai, tức là tất cả các điểm đã cho phải thẳng hàng. Đó là đpcm. 11
  • 12. Ví dụ 4: Trên mặt phẳng cho một số hữu hạn điểm không cùng nằm trên một đường thẳng. Chứng minh rằng tồn tại ba điểm sao cho đường tròn đi qua ba điểm đó không chứa điểm nào ở bên trong. GIẢI: Vì số các điểm đã cho là hữu hạn và chúng không cùng nằm trên một đường thẳng, nên khi lấy bao lồi hệ điểm, ta sẽ được một đa giác. Giả sử đó là đa giác lồi 1 2... pA A A . Như thế các điểm còn lại đã cho phải nằm trong bao lồi. Gọi 1,k kA A + là hai đỉnh liên tiếp của của đa giác lồi( nghĩa là xét một cạnh tùy ý 1k kA A + ). Khi ấy mọi điểm đã cho đều nằm ở một nửa mặt phẳng xác định bởi 1k kA A + . Từ giả thiết suy ra tập hợp các điểm đã cho không thuộc 1k kA A + là khác rỗng. Vì thế theo nguyên lý cực hạn, tồn tại C sao cho 1 1 1axk k k kA CA m A A A+ +∠ = ∠ , ở đây giá trị lớn nhất lấy theo mọi 1,i n= mà i ≠ k, i ≠ k + 1( giả sử 1 2, ,... nA A A là hệ hữu hạn điểm cho trước). Khi đó đường tròn ngoại tiếp ta giác 1k kCA A + là đường tròn cần tìm Ví dụ 5: Bên trong một hình vuông cạnh 1 cho n điểm sao cho không có ba điểm thẳng hàng. Chứng minh rằng tồn tại môt tam giác có đỉnh tại các điểm đã cho và diện tích S của nó thỏa mãn bất đẳng thức 1 2 S n < − GIẢI: Xét bao lồi của n điểm nằm bên trong hình vuông. Vì không có ba điểm nào thẳng hàng, nên bao lồi là đa giác lồi có k đỉnh( k≤n), ngoài ra các điểm đã cho hoặc là đỉnh của đa giác lồi, hoặc nằm hẳn bên trong đa giác lồi. Chỉ có hai khả năng xảy ra 1. Nếu k = n. Khi đó số đường chéo xuất phát từ A1 của đa giác bao lồi tạo thành cùng các cạnh của đa giác ( n – 2) tam giác. Gọi S là diện tích tam giác nhỏ nhất trong (n-2) tam giác ấy Vì tổng các diện tích của (n-2) tam giác nhỏ hơn 1( chú ý 1 là diện tích hình vuông chứa chọn ( n-2) tam giác này), suy ra 1 2 S n < − 2. Nếu k < n. Khi đó bên trong đa giác bao lồi 1 2... kA A A có (n-k) điểm Ak+1, Ak+2,...., An. Nối Ak+1 với các đỉnh A1; A2;....Ak. Khi đó có k tam giác Ak+1A1A2, Ak+1A2A3 ; ....; Ak+1AkA1 12
  • 13. Vì không có ba điểm nào thẳng hàng, Nên các điểm Ak+2,...., An phải nằm hẳn trong k tam giác nói trên. Giả sử Ak+2 nằm hẳn trong tam giác nào đó. Nối Ak+2 với ba đỉnh của tam giác này, thì từ một tam giác sẽ có ba tam giác mới . Sau mỗi lần làm số tam giác tăng lên 2. Như vậy ta đi đến: k+ 2(n- k – 1) = 2n – k – 2 = (n – 2) + (n – k) tam giác.mà bên trong mỗi tam giác này không có điểm nào thuộc n điểm đã cho. Gọi S là diện tích bé nhất trong các tam giác trên , thế thì: ( ) ( ) 1 1 n – 2 n – k 2 S n < < + − ( Do n – k >0) Bất đẳng thức 1 2 S n < − được chứng minh. Ví dụ 6: Bên trong hình vuông cạnh 1 cho n điểm. Chứng minh rằng tồn tại một tam giác có đỉnh tại các điểm đã cho hoặc đỉnh của hình vuông sao cho diện tích S của nó thỏa mãn bất đẳng thức: 1 2( 1) S n ≤ + Giải: Gọi A, B, C, D là bốn đỉnh hình vuông và A1; A2;....An là n điểm nằm trong hình vuông. Nối A1 với 4 đỉnh A, B, C, D. Khi đó ta được 4 hình tam giác. *) Nếu A2 nằm trong một trong 4 tam giác đó ( Giả sử A2 nằm trong tam giác ADA1) Ta nối A2 với A, D, A1. Sau khi nối xong, số tam giác tăng thêm 2. *) Nếu A2 nằm trên cạnh chung (Ví dụ A2 ∈ A1D) nối A2 với A và C. Khi đó số tam giác cũng tăng thêm 2. Như vậy trong mọi trường hợp, số tam giác sẽ tăng thêm 2 Với các điểm A3;....An ta làm tương tự. Cuối cùng số tam giác được tạo thành là: 4 + 2(n-1) = 2n + 2 tam giác. Các tam giác trên đều có đỉnh là đỉnh của hình vuông hoặc n điểm đã cho. Khi đó, tổng diện tích của 2n +2 tam giác này bằng diện tích hình vuông (bằng 1). Theo nguyên lý cực hạn, tồn tại tam giác có diện tích nhỏ nhất trong 2n + 2 tam giác ấy. Gọi diện tích này là S thì 1 2( 1) S n ≤ + (Điều cần chứng minh) 13
  • 14. Bài tập tương tự: Bài toán 1.1 . Cho n điểm nằm trong tam giác ABC có diện tích là 1 2 cm . CMR: Từ n điểm đó cùng với 3 điểm A, B, C luôn tồn tại một tam giác có diện tích không lớn hơn 21 3 2( 1) cm n+ − . Bài toán 1.2 (tổng quát) . Cho n điểm nằm trong đa giác lồi m đỉnh có diện tích là 1 2 cm . CMR: Từ n điểm đó cùng với m đỉnh của đa giác, luôn tồn tại một tam giác có diện tích không lớn hơn 21 2( 1) cm m n+ − . Ví dụ 7: Trong các ô của bảng vuông kích thước xn n ô vuông, người ta viết các số sao cho tổng của các số có mặt trong các ô của một “chữ thập” (tức là hình gồm một hàng và một cột) bất kỳ không nhỏ hơn a. Tính giá trị nhỏ nhất của tổng các ô trong bảng. GIẢI: Lấy một hàng có tổng các số trong hàng đó là nhỏ nhất. Sau đó xét tổng tất cả có “chữ thập” được lập nên từ các ô của hàng đó . Có n “chữ thập” như vậy, theo điều kiện của bài toán. Ta suy ra, tổng các số ghi ở n “chữ thập” ấy không nhỏ hơn n.a Dễ thấy tổng nói trên bằng tổng của tất cả các số trong bảng cộng thêm (n – 1)lần tổng các số trong hàng lấy ra. Gọi tổng các số trong bảng là N, tổng các số trong hàng lấy ra là m, từ suy luận trên ta suy ra: N +(n – 1).m ≥ n.a (1) Theo định nghĩa số m, ta có: N m n ≤ (2) Từ (1) và (2) ta suy ra: 2 .( 1) . . 2 1 N n N n n a N a n n + − ≥ ⇔ ≥ − Mặt khác, chọn tất cả các ô trong bảng đều bằng 2 1 a n− . Khi đó tổng tất cả các số ghi trong mọi hình chữ thập là: (2 1). 2 1 a n a n − = − Phép ghi như vậy là hợp lệ. Với cách ghi này, tổng các số ghi vào các ô trong bảng là 2 . 2 1 n a n − Vậy giá trị nhỏ nhất của tổng cần tìm là 2 min 2 1 n a N n = − Ví dụ 8: Trong không gian cho một số hữu hạn điểm mà trong đó không có 4 điểm nào trong chúng cùng nằm trên một mặt phẳng sao cho thể tích của mỗi tứ diện tạo 14
  • 15. ra bởi đỉnh là những điểm đã cho không lớn hơn 1. Chứng minh rằng tất cả các điểm có thể được phủ bởi một tứ diện có thể tích bằng 27. GIẢI: Do số lượng điểm đã cho là hữu hạn, nên số lượng các tứ diện tạo thành cũng hữu hạn. Theo nguyên lý cực hạn, tồn tại tứ diện mà ta sẽ gọi là A1B1C1D1) có thể tích lớn nhất. Qua các đỉnh A1; B1; C1; D1 dựng các mặt phẳng song song với các mặt của tứ diện, ta nhận được tứ diện ABCD.Dễ dàng chứng minh được A1; B1; C1; D1 tương ứng là trọng tâm của các tam giác BCD,ACD,ABD, ABC. Từ đó ta có: 1 1 1 1 1 27 A B C D ABCD V V = (1) Do giả thiết 1 1 1 1 1A B C DV ≤ , nên từ (1) ta suy ra 27ABCDV ≤ (2) Bây giờ ta chứng minh tất cả các điểm đều nằm trong tứ diện ABCD. Giả sử: Tồn tại điểm M(trong số các điểm đã cho), sao cho M không thuộc tứ diện ABCD (3) Khi đó ít nhất một đỉnh của tứ diện ABCD (có thể cho đó là đỉnh B) sao cho B và M nằmtrong hai nửa không gian xác định bởi (ACD). Suy ra 1 1 1 1 1 1 1MA C D B A C DV V> (1) Bất đẳng thức (4) chứng tỏ rằng MA1C1D1 là tứ diện tạo bởi 4 đỉnh trong các điểm đã chốc thể tích lớn hơn thể tích của tứ diện B1A1C1D1. Điều này mâu thuẫn vơi cách định nghĩa tứ diện A1B1C1D1 (vô lý). Suy ra điều phải chứng minh. 15
  • 16. II- SỬ DỤNG NGUYÊN LÝ DIRICHLET ĐỂ GIẢI CÁC BÀI TOÁN HÌNH HỌC TỔ HỢP A- Nguyên lý: *) Nguyên lý Dirichlet (Gustav Lejeuve Dirichlet) : Nếu nhốt n con thỏ vào m cái chuồng ( 2m ≥ ) thì tồn tại một chuồng có ít nhất 1n m m + −      con thỏ. B- Vận dụng: Ví dụ 1: Cho p là số nguyên tố lớn hơn 5. chứng minh rằng tồn tại một số có dạng 111...11 mà chia hết cho p. GIẢI: Xét dãy số 1,11,111,..., 14243 höõsoá1 111....11 p c Ta chứng minh trong dãy trên phải có số chia hết cho p. Giả sử kết luận ấy không đúng, tức là không có bất kỳ số nào của dãylại chia hết cho p. Cho tương ứng mỗi số dư của phép chia cho p . Tập hợp số dư có thể thuộc tập hợp {1, 2, 3,..., p – 1} (Do 0 không thể thuộc tập hợp này). Ta lại có p số trong dãy số trên. Vì vậy theo nguyên lý Dirichlet tồn tại ít nhất hai số có cùng số dư khi chia cho p. Giả sử các số đó là 111...11 (m chữ số 1) và số 111....11 (n chữ số 1) với ( )1 n m p≤ < ≤ . Từ đó ta có / / / 1 / 0 / 1 (111...11 111...11) , 111...1000...0 111...1.10n m c s n c s m n c s n c s m n c s p hay p Hay p − − − M M M14243 14243 123 123 123 (1) Do p là sô nguyên tố lớn hơn 5 nên (p; 10) = 1, Vì thế từ (1) ta suy ra / 1 111...1 m n c s p − M123 (2) / 1 111...1 m n c s− 123 là một số thuộc dãy trên nên từ (2) suy ra mâu thuẫn với giả thiết. Vậy giả sử phản chứng là sai. Ta suy ra điều phải chứng minh. Ví dụ 2: Trong hình vuông đơn vị (cạnh bằng 1) có 101 điểm. Chứng minh rằng có 5 điểm đã chọn được phủ bởi hình tròn bán kính 1 7 . GIẢI: Chia hình vuông thành 25 hình vuông nhỏ bằng nhau, mỗi hình vuông có cạnh 0,2. Vì có 101 điểm, mà chỉ có 25 hình vuông, nên theo định lý Dirichlet tồn tại một hình vuông nhỏ có chứa ít nhất năm điểm ( trong 101 điểm đã cho ). Vì hình vuông này nội tiếp trong đường tròn có bán kính R = 1 2 25 2 10 = . Do 2 1 10 7 < nên dĩ nhiên đường tròn đồng tâm với đường 16
  • 17. tròn ngoại tiếp trên và có bán kính 1 7 chứa ít nhất năm điểm nói trên . Đó là đpcm Ví dụ 3: trong hình chữ nhật 3x4 đặt sáu điểm. chứng minh rằng trong số đó luôn tìm được hai điểm có khoảng cách giữa chúng không lớn hơn 5 GIẢI Chia hình chữ nhật đã cho thành năm hình ABCD, DCKFE, KFNM, NFEQR, QEDAS Vì có 6 điểm nên theo nguyên lý Dirichlet tồn tại một trong năm hình trên chứa ít nhất 2 trong 6 điểm nói trên. Theo định lý Pitago thì khoảng cách lớn nhất giữa hai điểm trong một hình bằng 5. Ví dụ AC = CE = KE = KM = DS = DQ = QF = FR = 5 Từ đó ta luôn tìm được 2 điểm trong 6 điểm đã cho có khoảng cách không lớn hơn 5 (Đpcm) BÀI TOÁN TƯƠNG TỰ 1: Trong một tam giác đều, cạnh có độ dài bằng 1, đặt năm điểm . Chứng minh rằng tồn tại hai điểm mà khoảng cách giữa chúng nhỏ hơn hoặc bằng 1 2 Thật vậy, gọi M, N, P tương ứng là trung điểm của AB, BC, CA .Khi đó AMP, BMN, MNP, MNP, NPC là bốn tam giác đều bừng nhau cạnh bằng 1 2 . Chú ý nếu P là tam giác đều cạnh bằng a, thì d(P)=a Vì thế d(AMP)=d(BMN) =d(MNP) =d(NPC)= 1 2 Vì năm điểm thuộc vào bốn tam giác, nên theo nguyên lý Dirichlet tồn tại ít nhất hai điểm, giả sử Ai , Aj thuộc về cùng một tam giác trong bốn tam giác đều nhỏ nói trên . Ta có AiAj ≤ d(AMP)= 1 2 Đó là đpcm BÀI TOÁN TƯƠNG TỰ 2 : Trong hình tròn đường kính 5 có 10 điểm . Chứng minh rằng tồn tại ít nhất hai điểm mà khoảng cách giữa chúng bé hơn hoặc bằng 2. GIẢI Thật vậy: Trong đường tròn tâm O đường kính 5, vẽ đường tròn đồng tâm có đường kính bằng 2. Chia đường tròn đã cho thành 9 phần(như hình vẽ). 17
  • 18. Xét một phần bất kỳ, giả sử là hình III – ABCD Ta thấy ngay, khoảng cách lớn nhất giữa hai điểm của (III) là DA = BC Do ∠ DOA = 450 nên d2 = DA2 = DO2 + OA2 -2DO.OA.cos450 2 25 5 2 1 2. .1. 2 2 2 2 d   = + − ⇒ < ÷   Theo nguyên lý Dirichlet tồn tai ít nhất hai điểm nằm trong môt trong 9 phần (I), (II), .....(IX) có đường kính không vượt quá 2. Suy ra điều phải chứng minh. Ví dụ 4: Tìm hình vuông có kích thước bé nhất, để trong hình vuông đó có thể sắp xếp năm hình tròn có bán kính 1, sao cho không có hai hình tròn nào trong chúng có điểm trong chung. GIẢI: Giả sử hình vuông ABCD có tâm O cạnh là a chứa 5 hình tròn không cắt nhau và đều có bán kính bằng 1. Vì cả 5 hình tròn này nằm trọn trong hình vuông, nên tâm của chúng nằm trong hình vuông A1B1C1D1 tâm O cạnh là a-2. Ở đây AB // A1B1. Các đường thẳng nối nối các trung điểm của các cạnh đối diện của hình vuông A1B1C1D1 chia A1B1C1D1 thành 4 hình vuông nhỏ. Theo nguyên lý Dirichlet tồn tại một trong 4 hình vuông nhỏ, mà trong hình vuông này chứa ít nhất hai trong số 5 tâm hình tròn nói trên(Không mất tính tổng quát, giả sử đó là O1O2) Do trong 5 đường tròn, không có 2 đường tròn nào cắt nhau nên O1O2 ≥ 2 (1) 18
  • 19. Mặt khác, do O1O2 cùng nằm trong một hình vuông nhỏ(cạnh hình vuông bằng 2 2 a − nên). Mà O1O2 2 2 2 a − ≤ (2) Từ (1) và (2) ta suy ra 2 2 2 2 2 2 2 a a − ≥ ⇔ ≥ + (3) Vậy mọi hình vuông cạnh a thỏa mãn điều kiện đề bài sẽ thỏa mãn (3) Ta xét hình vuông ABCD có a = 2 2 2+ . Và xét 5 hình tròn có tâm O, A1; B1; C1; D1 (hình vẽ) thì mọi yêu cầu của bài toán được thỏa mãn . Vậy kích thước bé nhất của cạnh hình vuông thỏa mãn điều kiện đề bài là 2 2 2+ Ví dụ 5: Cho 2014 đường thẳng cùng có tính chất: chia hình vuông thành hai tứ giác có tỉ số diện tích bằng 2 3 . Chứng minh rằng có ít nhất 504 đường thẳng trong 2014 đường thẳng trên đồng quy. GIẢI: Các đường thẳng chia hình vuông thành hai tứ giác nên chúng không thể cắt hai cạnh kề của hình vuông và không đi qua đỉnh nào của hình vuông. Giả sử một đường thẳng cắt hai cạnh đối AD và BC tại M và N. Ta có 1 ( ) 2 2 22 13 3 IF 3 ( ) 2 ABMN MCDN AB BM ANS EI S CD MC ND + = ⇒ = ⇔ = + (E,F,P,Q lần lượt là trung điểm các cạnh hình vuông. I,K,G,H lần lượt là những điểm thỏa mãn 2 IF 3 IE HP GF KQ HQ GE KP = = = = Từ lập luận trên ta suy ra mỗi đường thẳng thỏa mãn yêu cầu của đề bài đều đi qua một trong 4 điểm G, H, I, K nói trên. Do có 2014 đường thẳng, nên theo nguyên lý Dirichlet phải tồn tại ít nhất 2014 1 504 4   + =    đường thẳng cùng đi qua một điểm trong 4 điểm G,H, I, K nói trên. Vậy có ít nhất 504 đường thẳng trong số 2014 đường thẳng đã cho đồng quy. Ví dụ 6: Cho một bảng kích thước 2n x 2n ô vuông. Người ta đánh dấu vào 3n ô bất kỳ của bảng. Chứng minh rằng có thể chọn ra n hàng n cột của bảng sao cho các ô được đánh dấu đều nằm trong n hàng n cột này. GIẢI: 19
  • 20. Trong các hàng có ô được đánh dấu. Chọn ra n hàng có số ô được đánh dấu nhiều nhất. Ta chứng minh rằng số ô được đánh dấu còn lại nhỏ hơn hoặc bằng n. Giả sử số ô được đánh dấu còn lại lớn hơn hoặc bằng n +1 . Số hàng còn lại chưa chọn là n hàng. Theo nguyên lý Dirichlet sẽ có ít nhất một hàng (Trong số n hàng còn lại chứa ít nhất 2 ô đã được đánh dấu). Từ cách chọn ta suy ra trong n hàng được chọn thì mỗi hàng có ít nhất 2 ô được đánh dấu. Tức là trên n hàng đã chọn có ít nhất 2n ô đã được đánh dấu. Như vậy, số ô được đánh dấu lớn hơn hoặc bằng 2n + (n +1) > 3n (vô lý). Suy ra điều phải chứng minh. Như vây, sau khi đã chọn ra n hàng với cách chọn như trên. Theo nhận xét sẽ còn lại không quá n ô được đánbh dấu. Hay cùng lắm sẽ có n cột chứa chúng. Vậy: có thể chọn ra n hàng n cột của bảng sao cho các ô được đánh dấu đều nằm trong n hàng n cột này. Ví dụ 7 Cho 1000 điểm M1, M2. …M1000 trên một mặt phẳng. Vẽ môt đường tròn bán kính 1 tùy ý. Chứng minh rằng tồn tại điểm S trên đường tròn sao cho SM 1 + SM2+ … +SM1000 ≥ 1000 Giải: S1 M1 M2 M1000 S2 Xét một đường kính S1S2 tùy ý của đường tròn, ở đây S1 vaf S2 là hai đầu của đường kính nên ta có 1 1 2 1 1 2 1 2 2 2 1 1000 2 1000 2 2 ... 2 S M S M S S S M S M S M S M  + ≥ =  + ≥    + ≥ Cộng từ vế 1000 bất đẳng trên ta có (S1M1+ S1M2 +…+ S1M1000 ) + (S2M1+ S2M2 +…+ S2M1000 ) ≥ 2000 (1) Từ (1) và theo nguyên ý Dirichlet suy ra trong hai tổng của vế trái cảu (1) có ít nhất một tổng lớn hơn hoặc bằng 1000 Giả sử (S1M1+ S1M2 +…+ S1M1000 )≥ 1000, khi đó S = S1. Đó là ddiepcm. Ví dụ 8: Cho mỗi điểm trên mặt phẳng được tô bằng một trong hai màu xanh, đỏ. Chứng minh rằng tồn tại một tam giác mà ba đỉnh và trọng tâm cùng màu GIẢI: G M P A B C B' C' A' N Lấy 5 điểm tùy ý sao cho không có ba điểm nào thảng hàng trên mặt phẳng . Khi đó vì chỉ dùng có hai màu để tô các đỉnh nên theo nguyên lý Dirichlet phải tồn tại ba điểm cùng màu. Giả sử ba điểm đó là ba điểm A, B, C cùng có màu đỏ. Như vậy ta có tam giác ABC với ba đỉnh màu đỏ. Gọi G 20
  • 21. là trọng tâm tam giác ABC. Chỉ có hai khả năng xảy ra: 1) Nếu G là màu đỏ. Khi đó A,B, C, G có cùng màu đỏ và bài toán được chứng minh 2) Nếu G có màu xanh. Kéo dài GA, GB, GC các đoạn AA’ = 3GA, BB’ = 3GB, CC’ = 3GC. Khi đó nếu gọi M, N, P tương ứng là các trung điểm của BC, CA, AB thì AA’ = 3GA = 6GM => AA’ = 2AM. Tương tự B’B = 2BN, C’C = 2CP.Do đó các tam giác A’BC, B’AC, C’AB tương ứng nhận A,B,C là trọng tâm. Mặt khác, ta cũng các có tam giác ABC, và A’B’C’ có cùng trọng tâm G. Có hai trường hợp xảy ra. a) Nếu A’, B’ C’ cùng màu xanh. Khi đó tam giác A’B’C’ và trọng tâm G có cùng màu xanh. b) Nếu ít nhất một trong các điểm A’ B’ C’ có màu điểm. Không mất tính tổng quát giả sử A’ màu đỏ. Khi đó tam giác A’BC và trọng tâm có màu đỏ. Vậy trong mọi khả năng luôn tồn tại một tam giác mà ba đỉnh và trọng tâm màu đỏ. Đó là đpcm. Ví dụ 9: Một hình lập phương có cạnh bằng 15 chứa 11000 điểm. Chứng minh rằng có một hình cầu bán kính 1 chứa ít nhất sáu điểm trong số 11000 điểm đã cho. Giải Chia mỗi cạnh của hình lập phương thành 13 phần bằng nhau. Như thế hình lập phương đã cho được chia thành 133 = 2197 hình lập phương nhỏ. Do 11000>5.2197 = 10985, nên tồn tại ít nhất một hình lập phương nhỏ, mà hình lập phương này chứa ít nhất sáu điểm. Như đã biết nếu gọi cạnh hình vuông băng a, thì hình càu ngoại tiếp nó có bán kinh R, với 1 3 2 R a= . Vì thế hình cầu ngoại tiếp hình lập phương nhỏ ( cạnh của nó là 15 13 ) là 2 1 15 1 15 1 675 1 676 1 3 3 4 1 2 13 2 13 2 196 2 169 2 R   = = = < = = ÷   . Hình cầu bán kính 1 này dĩ nhiên chứa ít nhất sáu điểm trong số 11000 điểm đã cho . Đó là đpcm. Ví dụ 10: Trong hình vuông cạnh 12 chứa 2014 điểm. Chứng minh rằng luôn tồn tại một tam giác đều cạnh 11 phủ kín 504 điểm trong 2014 điểm đã cho. GIẢI: Giả sử hình vuông ABCD có tâm là O và cạnh 12. Lấy E, F, G, H lần lượt trên các cạnh CD,AB,AD,BC sao cho AG = DE = CH = BF = 6 2 3− .Khi đó OE = OF = OG = OH = 4 3 Ta đi chứng minh có thể dùng một tam giác đều cạnh 11 phủ kín một trong các tứ giác OHCE, OEDG, OGAF hoặc OFBH. Thật vậy, do OH = OE = 4 3 < 11. 21
  • 22. Lấy J trên OF sao cho EJ = 11. Ta thấy 06 3 inF 60 24 3 S EC FEC= = ⇒ ∠ = Trên tia EC lấy K sao cho EK = EJ = 11. Ta có tam giác JEK đều cạnh 11. Ta đi chứng minh tam giác JEK phủ kín tứ giác OHCE Gọi giáo điểm của JK với BC là I. Suy ra IC = CK. 3 = 3 (5 -2 3 ) = 5 3 - 6 > 6 - 2 3 = CH CH < CI nên H nằm giữa C và I. Suy ra tam giác JEK phủ kín hoàn toàn tứ giác OHCE. Do vai trò của các tứ giác OHCE, OEDG, OGAF, OFBH là như nhau. Áp dụng nguyên lý Dirichlet ta suy ra: luôn tồn tại 2014 1 504 4   + =    điểm trong 2014 điểm đã cho nằm trong một trong các tứ giác OHCE, OEDG, OGAF hoặc OFBH. Vậy luôn tồn tại một tam giác đều cạnh 11 phủ kín 504 điểm trong 2014 điểm đã cho. 22
  • 23. III. MỘT SỐ DẠNG TOÁN HÌNH HỌC TỔ HỢP THƯỜNG GẶP 1.Dạng bài tập tô màu, bảng vuông. Bài 1: Trong mỗi ô bàn cờ kích thước 5x5 có một con bọ dừa. Vào một thời điểm nào đó tất cả các con bọ dừa bò sang ô bên cạnh (ngang hoặc dọc). Có thể khẳng định rằng sau khi các con bọ dừa di chuyển sẽ luôn có ít nhất một ô trong bàn cờ không có con bọ dừa nào trong đó không? (Đề thi giao lưu HSG môn Toán lớp 8 TP Vĩnh Yên năm học 2012-2013) Ta tô đen - trắng các ô bàn cờ như hình vẽ. Khi đó số ô đen nhiều hơn số ô trắng. Như vậy số con bọ dừa ở ô đen sẽ nhiều hơn số con bọ dừa ở ô trắng. Do mỗi con bọ dừa chỉ di chuyển sang ô bên cạnh(ngang hoặc dọc), vì thế sau khi di chuyển các ô đen sẽ chứa các con bọ dừa ở ô trắng. Mà số con bọ dừa ở ô đen nhiều hơn số con bọ dừa ở ô trắng nên sau khi các con bọ dừa bò đi sẽ có ít nhất một ô đen bị bỏ trống . Vậy : Có thể khẳng định rằng sau khi di chuyển sẽ luôn có ít nhất một ô trong bàn cờ không có con bọ dừa nào trong đó. Ví dụ 2: Trên lưới ô vuông cạnh 1. Người ta tô màu các ô bằng 2 màu đen trắng xen kẽ. Tính bán kính lớn nhất của đường tròn chỉ đi qua ba ô trắng. - Xét đường tròn chỉ thuộc một ô trắng: Đường kính của nó bằng 1 (1) - Xét đường tròn đi qua nhiều ô trắng => đường tròn đó phải đi qua các đỉnh của ô trắng. Không mất tính tổng quát, giả sử đường tròn đi qua đỉnh A của ô trắng ABCD. +) Nếu đường tròn đi qua 2 đỉnh liên tiếp của ô trắng ( A, B). Khi đó ta lại xét hai trường hợp: *) Đường tròn qua A, B, E. Khi đó nó là đường tròn 2 , 2 I    ÷  ÷   *) Đường tròn qua A, B, G. Khi đó nó là đường tròn 10 ; 2 K    ÷  ÷   (2) +) Nếu đường tròn qua 2 đỉnh đối diện của ô trắng, Giả sử là (A, C) Ta lại xét hai trường hợp: *) Qua A, C, M (Tương tự qua A, B, G) *) Qua A, C, N (Tương tự qua A, B, G) Cả hai trường hợp trên bán kính của đường tròn là 10 2 (3) 23
  • 24. Từ (1), (2) và (3) ta suy ra bán kính lớn nhất của đường tròn thỏa mãn yêu cầu đề bài là 10 2 . Ví dụ 3: Cho 6 điểm trong đó 3 điểm nào cũng nối được với nhau tạo thành 1 tam giác có cạnh được tô bởi một trong hai màu xanh hoặc đỏ. CMR: Bao giờ cũng tồn tại một tam giác có 3 cạnh cùng màu. Lời giải: Gọi A là một trong 6 điểm, 5 đoạn thẳng nối A với 5 điểm còn lại được tô bởi 2 hai màu xanh hoặc đỏ nên tồn tại 3 cạnh cùng màu. Giả sử là AB, AC, AD Xét 2 trường hợp: +Trường hợp 1: AB, AC, AD tô màu đỏ. Xét BCD∆ . Nếu có một cạnh được tô màu đỏ (giả sử BC) thì ABC∆ cùng màu đỏ (hình 1). Nếu không có cạnh nào của BCD∆ tô màu đỏ thì BCD∆ có 3 cạnh cùng màu xanh (hình 2). +Trường hợp 2: AB, AC, AD tô màu xanh. Chứng minh tương tự. Vậy luôn tồn tại một tam giác có 3 cạnh cùng màu. Ví dụ 4: Trên tờ giấy có kẻ vô hạn các ô vuông và mỗi ô được tô bằng một trong hai màu xanh hoặc đỏ. Cho bất cứ hình chữ nhật nào kích thước 2x3 thì có đúng hai ô màu đỏ. Xét hình chữ nhật có kích thước 2013x2014 bất kì. Tính số ô đỏ của nó. Giải Ta có nhận xét sau : Mọi hình chữ nhật 1x3 chứa đúng một ô màu đỏ. Thật vậy, giả sử kết luận của nhận xét không đúng, tức là tồn tại hình chữ nhật 1x3 có số ô màu đỏ không khác một. Không giảm tổng quát giả sử đó là hình chữ nhật AKHD kích thước 1x3 có hai ô đỏ ( nếu không thì không có ô đỏ nào, nhưng không thể là ba vì trong mọi hình chữ nhật 2x3 có đúng hai ô đỏ mà thôi ) Trường hợp AKHD không có ô đỏ nào thì lí luận tương tự Cũng có thể cho là hai ô đỏ của AKHD là ô 7 , ô8 24
  • 25. Xét hình chữ nhật BFNA. Đó là hình chữ nhật 2x3 , nên theo giả thiết nó có đúng hai ô đỏ mà 7 và 8 là hai ô đỏ, do đó các ô 1,2,4,5 là màu xanh. Xét hình chữ nhật BCHK, từ giả thiết và do các ô 1,2,4,5 màu xanh nên các ô 3 ,6 là màu đỏ. Xét hình chữ nhật ECDM kích thước 2x3, do ô 3,6,8 màu đỏ suy ra mâu thuẫn Vậy giả thiết phản chứng là sai. Nhận xét được chứng minh. Vì 2013 chia hết cho 3 và 2013: 3=671. do vậy hình chữ nhật kích thước 2013x2014 chia thành 2014x671 hình chữ nhật 1x3 Vậy số ô đỏ trong một hình chữ nhật kích thước 2013x2014 là 2014x671 ô Số ô đỏ cần tìm là 1351394 ô 3.Dạng bài tạo đa giác bao. Ví dụ: Cho một đa giác lồi có diện tích k 2 cm . CMR: Tồn tại một hình bình hành có diện tích không vượt quá 2k 2 cm chứa toàn bộ đa giác. Lời giải: Gọi C là đỉnh cách xa AB nhất (hình vẽ). +Trường hợp 1: Nếu AC là đường chéo của đa giác lồi. Qua C kẻ a // b ( bBA ∈, ) Gọi D,E là các đỉnh cách xa AC nhất, qua D kẻ đường thẳng d // AC, qua E kẻ đường thẳng c // AC. Gọi MNPQ là hình bình hành tạo bởi a ,b,c,d suy ra các đỉnh của đa giác nằm trong hoặc trên biên của hình bình hành MNPQ. Ta chứng minh 2 2MNPQ S k cm≤ , thật vậy: Gọi Sd là diện tích đa giác Ta có: ACD ACE d S S S+ ≤ ⇔ 1 2 MNPQ d S S≤ = k ⇔ 2 2 ( )MNPQ S k cm≤ . +Trường hợp 2: Nếu AC là cạnh của đa giác lồi. Gọi E là đỉnh cách xa AC nhất ( Chứng minh tương tự). Suy ra ta có điều phải chứng minh. 4.Phương pháp qui nạp toán học: Để chứng minh mệnh đề An đúng với mọi n ∈N*: -B1: chỉ ra mệnh đề đúng với n=1 tức là A1 đúng. -B2: giả sử mệnh đề đúng với n = k ( k ∈ N*) thì Ak đúng. -B3: chứng minh Ak+1 đúng ( mệnh đề đúng với n=k+1). Kết luận An đúng với mọi n ∈ N*. Ví dụ 1: Người ta dùng loại gạch để lát những căn phòng hình vuông n x n và luôn để trống 1 ô ở goc phòng. a) Hãy nêu cách lát căn phòng 4x4, 8x8 ô vuông. 25
  • 26. b) Chứng tỏ rằng người ta luôn lát được một căn phòng kích thước 2k x 2k sao cho ô trống là một ô bất kỳ. GIẢI : a) Xét hình vuông kích thước 4x4 ô vuông. Ta chia hình vuông thành 4 hình vuông kích thước 2 x 2. Hình thứ nhất ta đặt viên gạch sao chô ô ở góc bỏ trống, 3 hình còn lại ta lát sao cho ô trống quay vào phần tâm hình vuông. Cuối cùng ta lát viên gạch vào 3 ô còn thiếu (Hình 2) -) Để lát hình vuông 8 x 8 ô vuông, ta lát tương tự (Chia hình vuông 8 x 8 thành 4 hình vuông 4 x 4) (H3) b) Ta chứng minh bằng PP quy nạp. - Xét với k = 1: Bài toán luôn đúng. - Giả sử bài toán đúng với với k = n. Nghĩa là, có thể lát được hình vuông 2n x2n sao cho còn trống một ô (i ; j) bất kỳ. - Ta chứng minh bài toán đúng với k = n +1. Thật vấy : Xét hình vuông 2n+1 x 2n+1 = 4x(2n x2n ). Ta chia hình vuông 2n+1 x 2n+1 thành 4 hình vuông 2n x2n . Theo giả thiết quy nạp, luôn lát được hình 2n x2n sao cho có thể trống ô (i, j) bất kỳ. Ba hình vuông 2n x2n còn lại ta để trống ô ở góc và lát vào ô trung tâm (tương tự H3) cuối cùng lát viên gạch vào 3 ô trống. Bài toán được chứng minh. (H1) (H2) (H3) Ví dụ 2: CMR: số đường chéo của đa giác lồi n cạnh (n ≥4) bằng S n = 2 )3( −nn . Lời giải: Ta chứng minh Sn = 2 )3( −nn (1) đúng với mọi n ≥4. +) Ta thấy (1) đúng với n=4 vì S 4 =2, tứ giác có 2 đường chéo. +) Giả sử khẳng định (1) đúng với n=k (k ≥4) tức là đa giác lồi k cạnh có 2 )3( −kk đường chéo.+) Ta sẽ chứng minh đa giác lồi 26
  • 27. k+1cạnh có 2 )2)(1( −+ kk đường chéo Thật vậy khi thêm đỉnh thứ k+1 (hình vẽ) thì có thêm k-2 đường chéo nối từ A 2+k đến A 2 , A 3 ,…, A 1−k , ngoài ra cạnh A 1 A k cũng trở thành đường chéo. Do đó, S 1+k = S k +(k-2)+1= 2 )3( −kk +k-1= 2 )2)(1( −+ kk Vậy khẳng định (1) đúng với mọi n thuộc N*, n ≥4. MỘT SỐ DẠNG BÀI TỔNG HỢP KHÁC Ví dụ 1: Cho một cái bàn hình chữ nhật. Hai người chơi như sau: người thứ nhất dùng 1 đồng xu màu trắng đặt lên bàn, sau đó người thứ hai đặt 1 đồng xu đen lên bàn ở vị trí mà trước đó chưa có đồng xu nào đặt và cứ như vậy cho đến khi không còn chỗ để đặt đồng xu nào nữa. Biết rằng tất cả các đồng xu là bằng nhau. Người nào đến lượt đi mà không đặt được đồng xu nào lên bàn thì người đó thua cuộc. Chứng minh rằng có cách chơi để người thứ nhất luôn luôn thắng cuộc. (Đề thi HSG lớp 7-TP Vĩnh Yên năm học 2012-2013) Ta tô đen - trắng các ô bàn cờ như hình vẽ. Khi đó số ô đen nhiều hơn số ô trắng. Như vậy số con bọ dừa ở ô đen sẽ nhiều hơn số con bọ dừa ở ô trắng. Do mỗi con bọ dừa chỉ di chuyển sang ô bên cạnh(ngang hoặc dọc), vì thế sau khi di chuyển các ô đen sẽ chứa các con bọ dừa ở ô trắng. Mà số con bọ dừa ở ô đen nhiều hơn số con bọ dừa ở ô trắng nên sau khi các con bọ dừa bò đi sẽ có ít nhất một ô đen bị bỏ trống . Vậy : Có thể khẳng định rằng sau khi di chuyển sẽ luôn có ít nhất một ô trong bàn cờ không có con bọ dừa nào trong đó. Ví dụ 2: Trong mỗi ô bàn cờ kích thước 5x5 có một con bọ dừa. Vào một thời điểm nào đó tất cả các con bọ dừa bò sang ô bên cạnh (ngang hoặc dọc). Có thể khẳng định rằng sau khi các con bọ dừa di chuyển sẽ luôn có ít nhất một ô trong bàn cờ không có con bọ dừa nào trong đó không? (Đề thi HSG lớp 8-TP Vĩnh Yên năm học 2012-2013) Để đảm bảo thắng cuộc người thứ nhất phải có chiến lược chơi như sau: + Đầu tiên anh ấy chiếm vị trí trung tâm, tức là đặt đồng xu trắng Sao cho tâm của đồng xu trùng với tâm hình chữ nhật (Vị trí A) + Giả sử người chơi thứ 2 đặt đồng xu đen lên bàn (Tâm đồng xu là B) + Khi đó điểm đối xứng với B qua tâm A là D chắc chắn còn trống. 27
  • 28. Người thứ nhất đặt đồng xu trắng sao cho tâm đồng xu trùng D. + Luật chơi cứ tiếp tục như vậy. Nghĩa là sau khi người thứ hai đặt đồng xu thì người thứ nhất chọn vị trí đối xứng qua tâm A để đặt đồng xu của mình (lưu ý các vị trí đối xứng này luôn chưa có đồng xu nào đặt trước đó) Vì vậy, nếu người thứ hai còn đi được thì người thứ nhất vẫn đi được ở bước tiếp theo. Vì vậy người thứ nhất sẽ không bao giờ thua cuộc. Do mặt bàn có diện tích hữu hạn, nên nếu thực hiện theo chiến thuật trên thì người đi trước chắc chắn đảm bảo chiến thắng thuộc về mình. Ví dụ 3: Có 2008 con gà nhốt vào 1004 cái chuồng, mỗi chuồng có 2 con.Sau mỗi ngày người ta lại thay đổi vị trí của gà sao cho không có hai con gà nào đã nhốt chung chuồng trước đólại nằm cùng chuồng lần nữa.Hỏi tối đa có bao nhiêu ngày làm được như vậy? (Đề khảo sát HSG Huyện Vĩnh Tường năm học 2008-2009) Bài giải: Vẽ đa giác đều 2007 cạnh nội tiếp trong đường tròn. Ký hiệu tâm là 2008, Còn các đỉnh là 1, 2, 3, ….2007. Ký hiệu đoạn thẳng nối i với j là i – j với , 1,2008i j = . Xét bán kính 1 – 2008. Do tính chất đa giác đều nên ta thấy ngay 1003 dây cung sau đây vuông góc với bán kính ấy: 2 – 2007; 3 – 2006; 4 – 2005; ....; 1004 – 1005 Xét bán kính 2 – 2008 . Tương tự ta có 1003 dây cung sau đây vuông góc với bán kính ấy: 1- 3; 2007 – 4; 2006 – 5... Xét bán kính 3 – 2008. Tương tự ta có 1003 dây cung sau đây vuông góc với bán kính ấy: 2 – 4; 1 – 5; 2007 – 6; .... Cứ làm như vậy và cuối cùng xét bán kính 2007 – 2008, ta có 1003 dây cung sau đây vuông góc với bán kính ấy: 2006 – 1; 2005 – 2; 2004 – 3; .... 28
  • 29. Dựa vào nhận xét trên và cách đánh số các con gà từ 1 – 2008, ta chỉ ra cách xếp gà theo yêu cầu bài toán như sau: + Ngày thứ nhất xếp vào chuồng các đôi gà như sau:1 – 2008; 2- 2007; ....; 1004 – 1005 + Ngày thứ hai xếp vào chuồng các đôi gà như sau: 2- 2007;1-3;2007 – 4;...; 1006 – 1005 + Ngày thứ ba xếp vào chuồng các đôi gà như sau: 3 – 2008; 2- 4;1-5;....; 1007 – 10056 Tương tự + Ngày thứ 2007 xếp vào chuồng các đôi gà như sau: 2007 – 2008; 2006- 1; ....; 1004 – 1003 Mặt khác không có qua 2007 ngày vì mỗi con gà chỉ có thể cùng chuồng với một trong 2007 con gà còn lại. Vậy tối đa có 2007 ngày để xếp gà theo yêu cầu của đề bài. Ví dụ 4: Xét 20 số nguyên dương đầu tiên 1,2,3, ,20.… Hãy tìm số nguyên dương k nhỏ nhất có tính chất: Với mỗi cách lấy ra k số phân biệt từ 20 số trên, đều lấy được hai số phân biệt a và b sao cho a b+ là một số nguyên tố. (Đề tuyển sinh THPT chuyên Vĩnh Phúc 2013-2014) Xét tập hợp { }2,4,6,8,10,12,14,16,18,20 , ta thấy tổng của hai phần tử bất kì của tập hợp này đều không phải là số nguyên tố. Do đó 11k ≥ , ta sẽ chứng minh 11k = là số nhỏ nhất thỏa mãn yêu cầu bài toán. Thật vậy, ta chia tập hợp { }1,2,3,...,20A = thành 10 cặp số sau: ( ) ( ) ( ) ( ) ( ) ( ) ( ) ( ) ( ) ( )1,2 , 3,16 , 4,19 , 5,6 , 7,10 , 8,9 , 11,20 , 12,17 , 13,18 , 14,15 . Tổng của hai số trong mỗi cặp số trên là số nguyên tố. Khi đó mỗi tập con của A có 11 phần tử thì tồn tại ít nhất hai phần tử thuộc cùng vào một trong 10 cặp số trên. Suy ra trong A luôn có hai phần tử phân biệt có tổnglà một số nguyên tố. Ví dụ 5: Hỏi có hay không 16 số tự nhiên, mỗi số có ba chữ số được tạo thành từ ba chữ số a, b, c thỏa mãn hai số bất kỳ trong chúng không có cùng số dư khi chia cho 16? (Đề tuyển sinh THPT chuyên Vĩnh Phúc 2013-2014) Trả lời: Không tồn tại 16 số như vậy. Thật vậy, giả sử trái lại, tìm được 16 số thỏa mãn. Khi đó, ta có 16 số dư phân biệt khi chia cho 16: 0,1,2,3,4,5,6,7,8,9,10,11,12,13,14,15; trong đó có 8 số chẵn, 8 số lẻ. Do đó, ba chữ số a, b, c khác tính chẵn lẻ, giả sử hai chữ số chẵn là a, b và chữ số lẻ là c. Có 9 số lẻ được tạo thành từ những chữ số này: , , , , , , , , .aac abc acc bac bbc bcc cac cbc ccc Gọi 1 2 9, , ,x x x… là các số có hai chữ số thu được từ các số ở trên bằng cách bỏ đi chữ số c (ở hàng đơn vị). Khi đó ( )mod16 16i jx c x c≡ ⇔/ không là ước của i jx c x c− tức là i jx x− không chia hết cho 8 29
  • 30. Nhưng trong 9 số 1 2 9, , ,x x x… chỉ có ba số lẻ , ,ac bc cc nên 8 số bất kỳ trong 9 số 1 2 9, , ,x x x… luôn có hai số có cùng số dư khi chia cho 8, mâu thuẫn. Tương tự, trường hợp trong ba số a, b, c có hai số lẻ, một số chẵn cũng không xảy ra Ví dụ 6: Cho 2011 điểm nằm trong mặt phẳng, trong đó không có 3 điểm nào thẳng hàng và mọi tam giác có 3 đỉnh là 3 trong 2011 điểm đã cho đều có diện tích nhỏ hơn 1. Chứng minh rằng có thể đặt 2011 điểm trên trong một tam giác có diện tích bằng 4. (Đề khảo sát HSG Huyện Vĩnh Tường năm học 2011-2012) Giải: C A B M N P D K E F Giả sử tam giác ABC là tam giác có 3 đỉnh trong 2011 diểm đã cho và có diện tích lớn nhất => SABC < 1. Qua A, B, C kẻ các đường thẳng song song với các cạnh của tam giác ABC ta được tam giác MNP có SMNP <4. Ta chứng minh ko có điểm nào trong 2011 điểm đã cho nằm ngoài tam giác MNP. Thật vậy: Giả sử có điểm D nằm ngoài tam giác MNP . Khi đó SDBC > SABC trái với cách chọn tam giác ABC có diện tích lớn nhất. Suy ra 2011 điểm đã cho không nằm ngoài tam giác MNP có diện tích nhỏ hơn 4. Vì vậy 2011 điểm đó nằm trong tam giác KEF đồng dạng với tam giác MNP(h.vẽ) và có diện tích bằng 4. Ví dụ 7: Mỗi điểm của mặt phẳng được tô bởi một trong ba màu Đỏ, Xanh, Vàng. Chứng minh rằng tồn tại hai điểm ,A B được tô bởi cùng một màu mà độ dài 1.=AB (Đề thi HSG Tỉnh Vĩnh Phúc năm học 2010 – 2011) - Giả sử trái lại, với mọi cách tô, không tồn tại hai điểm cùng màu mà có khoảng cách bằng 1. Xét hai điểm , : 3M N MN = thì tồn tại các điểm ,P Q sao cho các tam giác ,MPQ NPQ là các tam giác đều có độ dài cạnh bằng 1. Khi đó, do hai điểm có khoảng cách bằng 1 thì được tô bởi hai màu khác nhau, nên ,M N phải được tô bởi cùng một màu, chẳng hạn tô P: Đỏ, Q: Vàng thì M, N: phải tô cùng màu Xanh, (Hình vẽ). - Từ đó, nếu điểm M được tô màu Xanh, thì mọi điểm nằm trên đường tròn tâm M, bán kính 3 đều được tô màu Xanh. Nhưng trên đường tròn này luôn có hai điểm mà khoảng cách giữa chúng bằng 1. Mâu thuẫn với giả thiết phản chứng. Từ đó suy ra điều phải chứng minh 30 (Vàng) (Đỏ) (Xanh) (Xanh) M N P Q
  • 31. Ví dụ 8: Trong bảng hình vuông gồm 10 10× ô vuông (10 hàng, 10 cột), người ta viết vào các ô vuông các số tự nhiên từ 1 đến 100 theo cách như sau: ở hàng thứ nhất, từ trái sang phải, viết các số từ 1 đến 10; ở hàng thứ hai, từ trái sang phải, viết các số từ 11 đến 20; cứ như vậy cho đến hết hàng thứ 10. Sau đó cắt bảng hình vuông thành những hình chữ nhật nhỏ kích thức 1 2× hoặc 2 1× . Tính tích số của hai số trong mỗi hình chữ nhật nhỏ rồi cộng 50 tích lại. Cần phải cắt hình vuông như thế nào để tổng tìm được nhỏ nhất ? Hãy tính giá trị nhỏ nhất đó. (Đề KS HSG Huyện Vĩnh Tường năm học 2011-2012) Giải: Cắt hình vuông thành các hình chữ nhật cỡ 1 2× hoặc 2 1× thì được tất cả 50 hình. Giả sử trong hình thứ k có 2 số ,k ka b thì hoặc 1k ka b− = hoặc 10.k ka b− = Ta có ( ) 22 2 2 2 k kk k k k a ba b a b −+ × = − suy ra ( ) ( ) 50 50 50 22 2 1 1 1 1 1 2 2 k k k k k k k k k a b a b a b = = = × = + − −∑ ∑ ∑ Trong đó ( ) ( ) 50 2 2 1 2 2 1 1 1 1 100 101 201 1 2 100 169175 2 2 2 6 k k k a b = × × + = + + + = × =∑ L và mỗi số ( ) 2 k ka b− hoặc bằng 1 hoặc bằng 100. Do đó, để tổng thu được là nhỏ nhất, thì ( ) 2 100, 1,2,...,500.k ka b k− = ∀ = Vì vậy, cần cắt hình vuông thành các hình chữ nhật với kích thước 2 1× . Và khi đó giá trị nhỏ nhất của tổng bằng 169175 25 100 166675− × = . Ví dụ 9: Cho đa giác lồi 1 2 100A A AK . Tại mỗi đỉnh kA ( 1,2,...,100k = ), người ta ghi một số thực ka sao cho giá trị tuyệt đối của hiệu hai số trên hai đỉnh kề nhau chỉ bằng 2 hoặc 3. Tìm giá trị lớn nhất có thể được của giá trị tuyệt đối của hiệu giữa hai số ghi trên mỗi cặp đỉnh của đa giác đã cho, biết rằng các số ghi tại các đỉnh đã cho đôi một khác nhau. (Đề tuyển sinh THPT chuyên Vĩnh Phúc 2011-2012) Xét đa giác lồi 1 2 100A A AK như hình vẽ. Khi đó 1 2k ka a +− = hoặc 1 3k ka a +− = ( 1,2,...,99k = ). Không mất tính tổng quát, coi 1a là nhỏ nhất, na là lớn nhất (dễ thấy 2n ≥ ). Đặt max i j i j d a a ≠ = − khi đó 1nd a a= − . Ta sẽ chứng minh 149.d = Nằm giữa 1, nA A , theo chiều kim đồng hồ có 2n − đỉnh và có 100 n− đỉnh, theo chiều ngược kim đồng hồ. Hơn nữa giá trị tuyệt đối của hiệu giữa hai số kề nhau không vượt quá 3. Do đó ( )1 1 2 2 3 1... 3 1n n nd a a a a a a a a n−= − ≤ − + − + + − ≤ − và tương tự ta có ( )3 100 1d n≤ − + . Suy ra ( ) ( )3( 1) 3(100 1) 300 150 2 2 n n d − + − + ≤ = = 31
  • 32. 150d = khi và chỉ khi hiệu giữa hai số ghi trên hai đỉnh kề nhau đúng bằng 3 hay ta có 1 3, 1,2,...,99i ia a i+− = = ( )1 1 2 1 1 2 2 1,...,98 i i i i i i i i i i a a a a a a a a i a a + + + + + + + − = − ⇒ − = − ⇒ = = ( ) ( )1 100 1 2 2 3 99 100 1 2 1 100 1 2... 99 99 3 99.3a a a a a a a a a a a a a a⇒ − = − + − + + − = − ⇒ − = − ⇒ = Điều này không xảy ra suy ra 150d = không thỏa mãn. Ta xây dựng một trường hợp cho 149d = như sau: 1 2 10, 2, 3k ka a a a −= = = + với 53 52 12,3, ,52; 2, 3, 54,55, ,100k kk a a a a k−= … = − = − = … Khi đó hiệu lớn nhất 53 1 149a a− = . Các số 2 3 53, , ,a a a… có dạng 2 3t+ , các số 54 55 100, , ,a a a… có dạng 147 3k− . Rõ ràng không tồn tại ,k t sao cho ( )2 3 147 3 3 145t k k l+ = − ⇔ + = ( ,k t ∈¢ ). Suy ra điều phải chứng minh. Ví dụ 10: Một số tự nhiên dương được gọi là số “Đẹp”, nếu nó là hợp số và không chia hết cho 2, 3, 5, 7. Hỏi có tất cả bao nhiêu số tự nhiên “Đẹp” nhỏ hơn 2011 Giải: Số “Đẹp” là hợp số, không chia hết cho 2,3,5,7, nên nó phải là tích của các số nguyên tố lớn hơn 10. Do 133 =2197> 2010 , nên số cần tìm có dạng a b× hoặc a b c× × với a,b, c là các số nguyên tố 11 a b c≤ ≤ ≤ . Xét số “Đẹp” dạng a b× với 11 a b≤ ≤ , a,b là các số nguyên tố : Với a=11 thì 11 181b≤ ≤ , có 38 số Với a=13 thì 13 151b≤ ≤ , có 31 số Với a=17 thì 17 113b≤ ≤ , có 24 số Với a=139 thì 19 103b≤ ≤ , có 20 số Với a=23 thì 23 83b≤ ≤ , có 15 số Với a=29 thì 29 71b≤ ≤ , có 12 số Với a=31 thì 31 61b≤ ≤ , có 8 số Với a=37 thì 37 53b≤ ≤ , có 5 số Với a=41 thì 41 47b≤ ≤ , có 3 số Với a=43 thì b=43, có 1 số Xét số “Đẹp” dạng a b c× × với 11 13a b c≤ ≤ ≤ ≤ , a,b,c là các số nguyên tố Có 3 số có dạng này đó là: 11.11.11=1331; 11.11.13=1573 ; 11.13.13=1859 Suy ra số các số “Đẹp” cần tìm là S = 38+31+24+20+15+12+8+5+3+1+3 = 160 số Ví dụ 11: Trong một hộp có 2014 viên sỏi. Có hai người tham gia trò chơi, mỗi người lần lượt phải bốc ít nhất là 11 viên sỏi và nhiều nhất là 20 viên sỏi. Người nào bốc viên sỏi cuối cùng sẽ thua cuộc. Hãy tìm thuật chơi để đảm bảo người bốc đầu tiên luôn là người thắng cuộc. GIẢI: 32
  • 33. Để đảm bảo thắng cuộc, ở nước đi cuối cùng của mình người bốc sỏi đầu tiên phải để lại trong hộp 11 viên sỏi. Ở nước đi trước đó phải để lại trong hộp: 11 (20 11) 42+ + = viên sỏi. Suy ra người bốc sỏi đầu tiên phải đảm bảo trong hộp lúc nào cũng còn 11 31k+ viên sỏi. Ta có (2014 11):31 64− = dư 19. Như vậy người bốc sỏi đầu tiên ở lần thứ nhất của mình phải bốc 19 viên. Tiếp theo, khi đối phương bốc k viên sỏi ( 1, 2, ..., 20k = ) thì người bốc sỏi đầu tiên phải bốc 31 k− viên sỏi, cuối cùng sẽ để lại 11 viên sỏi cho đối phương. Ví dụ 12: Có điền được hay không 100 số gồm 10 số -2, 10 số -1, 30 số 0, 40 số 1, 10 số 2 vào các ô của bảng ×10 10 (mỗi ô điền một số và gọi số ở hàng i tính từ dưới lên trên và cột j tính từ trái sang phải là aij) sao cho thỏa mãn hai điều kiện: i) Tổng các số trên các hàng, các cột đều bằng m; ii) Tổng các số aij trong bảng thỏa mãn i  j  2 bằng 5m. Không điền được. Thật vậy, giả sử trái lại, điền được các số thỏa mãn. Khi đó [ ]= × − + − + + + = 1 m 10.( 2) 10.( 1) 30.0 40.1 10.2 3 10 là một số lẻ. Ta chia các ô của bảng thành 4 loại: - Loại 1 gồm các ô ở hàng lẻ, cột lẻ. - Loại 2 gồm các ô ở hàng lẻ, cột chẵn. - Loại 3 gồm các ô ở hàng chẵn, cột lẻ. - Loại 4 gồm các ô ở hàng chẵn, cột chẵn. Kí hiệu Sk là tổng tương ứng trên tất cả các ô loại k. Khi đó S1 + S2 là tổng các số trên tất cả các hàng lẻ, nên S1 + S2 = 5m. S2 + S4 là tổng các số trên tất cả các cột chẵn, nên S2 + S4 = 5m. Loại 1 và loại 4 đều gồm các ô mà i - j chẵn, do đó S1 + S4 = 5m. Suy ra 2(S1 + S2 + S4) = 15m (1) Do m lẻ, nên VP(1) lẻ. Mà VT(1) chẵn: vô lí. Do đó điều giả sử là sai. Vậy không thể điền được các số thỏa mãn. Ví dụ 13: Có 40 học sinh trong một lớp đứng thành vòng tròn quay mặt vào tâm đường tròn để tham gia trò chơi đếm số như sau: Mỗi học sinh đếm một trong dãy số tuần hoàn 1,2,1,2,1,2….lần lượt theo chiều kim đồng hồ bắt đầu từ học sinh A(lớp trưởng). Nếu học sinh nào đếm số 2 thì phải rời ngay khỏi vòng tròn. Việc đếm cứ tiếp tục như vậy cho đến khi chỉ còn 1 học sinh - Học sinh đó được coi là thắng cuộc. B là một học sinh giỏi Toán, B đã tìm ngay được vị trí đứng để mình là người thắng cuộc. Hỏi B đứng ở vị trí nào theo chiều kim đồng hồ kể từ A? GIẢI: 33
  • 34. Xét trường hợp lớp chỉ có 32 học sinh thì lớp trưởng ở vị trí thứ nhất sẽ luôn thắng cuộc. Như vậy, để B là người thắng cuộc thì B phải đứng ở vị trí thứ nhất sau khi loại được (40 – 32) = 8 người. Hay B phải đứng ở vị trí 8 x 2 + 1 = 17 theo chiều kim đồng hồ kể từ vị trí của lớp trưởng A. (GV có thể yêu cầu HS tổng quát hóa bài toán với n HS trong lớp. Nếu lớp có 2k HS thì B có cơ hội thắng cuộc ko?) 34
  • 35. BÀI TẬP TỔ HỢP CHỌN LỌC Bài 1. Lát bàn cờ 8 x 8 bằng 21 quân trimino kích thước 1 x 3. Hỏi ô trống còn lại có thể là ô nào ? Bài 2. Cho n là số nguyên dương lớn hơn hay bằng 2. Kí hiệu A = {1, 2, …, n}. Tập con B của tập A được gọi là 1 tập "tốt" nếu B khác rỗng và trung bình cộng của các phần tử của B là 1 số nguyên. Gọi Tn là số các tập tốt của tập A. Chứng minh rằng Tn – n là 1 số chẵn. Bài 3. Trên bàn cờ vua kích thước 8x8 được chia thành 64 ô vuông đơn vị, người ta bỏ đi một ô vuông đơn vị nào đó ở vị trí hàng thứ m và cột thứ n. Gọi S(m;n) là số hình chữ nhật được tạo bởi một hay nhiều ô vuông đơn vị của bàn cờ sao cho không có ô nào trùng với vị trí của ô bị xóa bỏ ban đầu. Tìm giá trị nhỏ nhất và giá trị lớn nhất của S(m;n). Bài 4. Ba nhóm đường thẳng song song chia mặt phẳng thành N miền. Hỏi số đường thẳng của cả 3 nhóm ít nhất là bao nhiêu để N > 2011. HD:Giả sử số đường thẳng của mỗi nhóm là p, q, r hãy chứng minh số miền tạo thành không vượt quá: pq + qr + rp + p + q + r + 1. Ý tưởng chính là: p đường thẳng song song chia mặt phẳng thành p+1 phần. Khi kẻ thêm 1 đường thẳng không song song với p đường thẳng này thì sẽ tạo ra thêm p+1 phần. Như thế, sau khi kẻ q đường thẳng nữa thì số phần là (p+1)(q+1). Cuối cùng, khi đã có 2 họ p + q đường thẳng, khi kẻ thêm đường thẳng của họ thứ 3 sẽ cắt hai họ trên ở nhiều nhất p+q điểm, do đó tạo ra p+q+1 miền mới. Suy ra số miền mới tối đa là (p+1)(q+1) + r(p+q+1). Bài 5. Cho 9 điểm nguyên trên mặt phẳng tọa độ, không có 3 điểm thẳng hàng. Chứng minh ta luôn chọn được 3 điểm thỏa mãn diện tích của tam giác tạo bởi chúng là số chẵn. Bài 6. Ta có 15 tấm thẻ được đánh số 1, 2, …, 15. Có bao nhiêu cách chọn ra một số (ít nhất 1) tấm thẻ sao cho tất cả các số viết trên các tấm thẻ này đều lớn hơn hoặc bằng số tấm thẻ được chọn. Bài 7. Trên một đường thẳng nằm ngang, cho 2005 điểm được đánh dấu trắng hoặc đen. Với mỗi điểm, xác định tổng tất cả các điểm trắng bên phải và điểm đen bên trái của nó. Biết rằng, trong 2005 tổng trên có đúng một số xuất hiện số lẻ lần. Hãy tìm tất cả các giá trị có thể có của số này. Bài 8: Cho số nguyên n ≥ 2. Chứng minh rằng trong mọi họ gồm ít nhất 2n-1 +1 tập hợp con không rỗng phân biệt của tập hợp {1, 2,…, n} đều tìm được ba tập hợp mà một trong chúng là hợp của hai tập hợp còn lại. Bài 9: Trong mặt phẳng cho 2011 điểm sao cho với ba điểm bất kỳ trong số các điểm đó ta luôn tìm được hai điểm để đoạn thẳng được tạo thành có độ dài bé hơn 1. Chứng minh luôn tồn tại một hình tròn bán kính là 1 chứa không ít hơn 1006 điểm đã cho. Bài 10. Cho n điểm trong mặt phẳng, trong đó không có ba điểm nào thẳng hàng, biết rằng ba điểm bất kỳ trong n điểm đã cho tạo thành một tam giác có diện tích 35
  • 36. không lớn hơn 1. Chứng minh rằng có thể phủ tất cả n điểm đã cho bằng một tam giác có diện tích không lớn hơn 4. Xét tam giác ABC có diện tích lớn nhất và vẽ tam giác DEF là tam giác nhận tam giác ABC là tam giác trung bình. Khi đó DEF là tam giác cần tìm. Bài 11. Có n đội bóng thi đấu vòng tròn 1 lượt. Hãy lập lịch thi đấu gồm n-1 vòng đấu sao cho trong mỗi vòng, mỗi đội chỉ thi đấu nhiều nhất 1 trận. Bài 12. Cho 2n+1 máy tính. Hai máy tính bất kỳ được nối với nhau bởi một sợi dây. Chứng minh rằng có thể tô các máy tính và các sợi dây bằng 2n+1 màu sao cho: i) Các máy tính được tô màu khác nhau ii) Các sợi dây xuất phát từ cùng một máy tính được tô màu khác nhau iii) Hai máy tính và sợi dây nối chúng được tô màu khác nhau. Xếp các máy tính lên đỉnh một 2n+1 giác đều A1A2…A2n+1 và tô màu lần lượt là 1, 2, …, 2n+1. Với mỗi i, Nối đường kính OAi và tô tất cả các cạnh và đường chéo vuông góc với OAi bằng màu i. Ta sẽ được phép tô thỏa mãn điều kiện. Bài 13: Có 6n+4 nhà toán học tham dự 1 hội nghị, trong đó có 2n+1 buổi thảo luận. Mỗi buổi thảo luận đều có 1 bàn tròn cho 4 người ngồi và n bàn tròn cho 6 người ngồi. Biết rằng 2 người bất kỳ không ngồi cạnh nhau hoặc đối diện nhau quá 1 lần. a. Hỏi có thể thực hiện được không với n=1? b. Hỏi có thể thực hiện được không với n>1? Bài 14. Trong đường tròn đơn vị có 4 điểm sao cho khoảng cách giữa hai điểm bất kỳ trong chúng lớn hơn 1. Chứng minh rằng có thể kẻ hai đường kính vuông góc của đường tròn sao cho trong mỗi một góc 1/4 có đúng 1 trong 4 điểm đã cho. Bài 15. Số nguyên dương A trong cách viết thập phân được gọi là số kỳ quặc nếu tổng của A và số thu được từ A bằng cách viết theo thứ tự ngược lại là một số có tất cả các chữ số đều lẻ. Hãy tìm số các số kỳ quặc có 3 chữ số. Bài 16. Có ba lớp học A, B, C, mỗi lớp có 30 học sinh. Biết rằng một học sinh bất kỳ đều quen với ít nhất 31 học sinh khác lớp. Chứng minh rằng tồn tại ba học sinh a, b, c lần lượt thuộc lớp A, B, C sao cho họ đôi một quen nhau. Chọn học sinh có số bạn quen nhiều nhất ở 1 lớp khác. Giả sử số bạn quen lớn nhất đó là k. Giả sử đó là học sinh a ở lớp A và a quen với k học sinh ở lớp B. Do a quen với ít nhất 31 học sinh và a ≤ 30 nên a quen với ít nhất một học sinh ở C, giả sử đó là c. Theo định nghĩa của k, c quen không quá k học sinh ở A, do đó c quen với ít nhất 31-k học sinh ở B. Vì tổng số người quen của a và c trong B ít nhất là k + 31-k = 31 nên a và c phải có ít nhất một người quen chung trong B. Gỉa sử đó là b thì a, b, c là 3 học sinh cần tìm. Bài 17. Phủ bàn cờ 8 x 8 bằng 21 quân trimino 1 x 3. Hỏi ô trống còn lại có thể là ô nào? 36
  • 37. HD: Gọi ε là số sao cho ε2 + ε + 1 = 0 thì ta có ε3 = 1. Đánh số các cột và các hàng theo thứ từ từ trái sang phải, từ dưới lên trên là 1, 2, …, 8. Ta điền vào ô (i, j) số εi+j . Khi đó dễ thấy tổng các số ở các ô mà một quân trimino 1 x 3 bất kỳ phủ sẽ bằng 0. Như vậy, nếu phủ bàn cờ 8 x 8 bằng 21 quân domino thì tổng các số ở các ô bị phủ bằng 0. Suy ra số còn lại bằng tổng tất cả các số được ghi. Ta có tổng các số được ghi bằng 8 8 8 8 2 1 , 8 1 1 1 1 ( ) 1i j i j i j i j i j i j ε ε ε ε ε ε+ + ≤ ≤ = = = =    = = = + = ÷ ÷    ∑ ∑∑ ∑ ∑ Suy ra số ở ô còn lại phải là số 1. Nếu (i, j) là ô còn lại thì từ đó suy ra i + j chia hết cho 3. Làm tương tự như vậy nhưng ở ô (i, j) ghi số εi-j ta cũng thu được i – j chia hết cho 3. Suy ra cả i và j đều chia hết cho 3. Như vậy (i, j) chỉ có thể là 1 trong 4 cặp (3, 3), (3, 6), (6, 3), (6, 6). Dễ dàng chỉ ra cách phủ cho các TH này (do tính đối xứng, chỉ cần chỉ cho 1 trường hợp). Bài 18. Xét số tự nhiên n ≥ 2. Bắt đầu các số 1, 2, …, 2n-1, 2n ta thực hiện các phép biến đổi như sau: Chọn 2 số a, b sao cho a – b ≥ 2, xóa hai số này và thay bằng hai số a – 1, b + 1; với bộ số thu được, ta lại thực hiện phép biến đổi tương tự, và cứ như vậy. a) Chứng minh rằng sau một số lần thực hiện các phép biến đổi như trên, ta phải đạt đến trạng thái dừng, tức là không thể thực hiện được một phép biến đổi nào nữa. b) Gọi k là số phép biến đổi cần thực hiện để đạt đến trạng thái dừng. Hãy tìm giá trị lớn nhất và giá trị nhỏ nhất của k. Bài 19. Một ngũ giác lồi có tất cả các góc trong bằng nhau và các cạnh đều là các số hữu tỷ. Chứng minh rằng ngũ giác đó là ngũ giác đều. Bài 20. Hai người A và B cùng chơi một trò chơi. Ban đầu trên bàn có 100 viên kẹo. Hai người thay phiên nhau bốc kẹo, mỗi lần được bốc k viên với k ∈ {1, 2, 6} . Hỏi ai là người có chiến thuật thắng, người đi trước hay người đi sau? Bài 21. a) Trên bảng có số 2010. Hai người A và B cùng luân phiên thực hiện trò chơi sau: Mỗi lần thực hiện, cho phép xoá đi số N đang có trên bảng và thay bằng N-1 hoặc [N/2]. Ai thu được số 0 trước là thắng cuộc. Hỏi ai là người có chiến thuật thắng, người đi trước hay người đi sau. b) Cùng câu hỏi với luật chơi thay đổi như sau: Mỗi lần thực hiện, cho phép xoá đi số N đang có trên bảng và thay bằng N-1 hoặc [(N+1)/2]. Bài 22. Hình tròn được bởi 5 đường kính thành thành 10 ô bằng nhau. Ban đầu trong mỗi ô có 1 viên bi. Mỗi lần thực hiện, cho phép chọn 2 viên bi bất kỳ và di chuyển chúng sang ô bên cạnh, 1 viên theo chiều kim đồng hồ và 1 viên ngược chiều kim đồng hồ. Hỏi sau một số hữu hạn lần thực hiện, ta có thể chuyển tất cả các viên bi về cùng 1 ô được không? 37
  • 38. HD: Ta tô màu các ô bằng hai màu đen trắng xen kẽ nhau. Gọi S là tổng số viên bi nằm ở các ô đen thì ở trạng thái ban đầu ta có S = 5. Nếu giả sử ngược lại rằng ta có thể đưa các viên bi về cùng 1 ô thì ở trạng thái cuối cùng này, ta sẽ có S = 0 (nếu ta dồn các viên bi về một ô trắng) hoặc S = 10 (nếu ta dồn các viên bi về một ô đen). Bây giờ ta sẽ thu được điều mâu thuẫn nếu ta chứng minh được qua các lần thực hiện thì tính chẵn lẻ của S sẽ không thay đổi, tức là nếu ban đầu S là số lẻ thì qua các lần thực hiện, S sẽ luôn là số lẻ (và sẽ không thể bằng 0 hoặc bằng 10). Nếu nhận xét rằng các ô đen trắng xen kẽ nhau thì điều mà chúng ta cần chứng minh khá hiển nhiên và chúng tôi xin dành phép chứng minh chi tiết cho bạn đọc. Bài 23. Có 2n người xếp thành 2 hàng dọc. Hỏi có bao nhiêu cách chọn ra một số người (ít nhất 1) từ 2n người này, sao cho không có hai người nào đứng kề nhau được chọn. Hai người đứng kề nhau là hai người có số thứ tự liên tiếp trong một hàng dọc hoặc có cùng số thứ tự ở hai hàng. Gọi Sn là số cách chọn ra một số người từ 2n người xếp thành 2 hàng dọc và Tn là số cách chọn ra một số người từ 2n-1 người xếp thành 2 hàng dọc, trong đó khuyết một chỗ ở đầu của một hàng. Ta có S1 = 2, T1 = 1. 1 3 4 Hình 1. Sn với n = 5 1 2 Hình 2. Tn với n = 5 Xét 2n người xếp thành 2 hàng dọc (như hình 1). Ta xét các cách chọn thoả mãn điều kiện đầu bài. Xảy ra các khả năng sau : 1) Người ở vị trí số 1 được chọn : Khi đó người ở vị trí số 2 và số 3 không được chọn  Có Tn-1 + 1 cách chọn (+1 là do bổ sung cách chọn « không chọn gì cả » ) 2) Người ở vị trí số 2 được chọn : Tương tự, có Tn-1 + 1 cách chọn. 3) Cả hai người ở vị trí số 1 và số 2 đều không được chọn: Có Sn-1 cách chọn. Vậy ta có Sn = Sn-1 + 2Tn-1+ 2 (1). Xét 2n-1 người xếp thành 2 hàng dọc (như hình 2). Ta xét các cách chọn thoả mãn điều kiện đầu bài. Xảy ra các khả năng sau : 1) Người ở vị trí số 1 được chọn : Khi đó người ở vị trí số 2 không được chọn  có Tn-1 + 1 cách chọn 2) Người ở vị trí số 1 không được chọn : có Sn-1 cách chọn. Vậy ta có Tn = Sn-1 + Tn-1 + 1 (2) 38
  • 39. Từ (1) ta suy ra 2Tn-1 = Sn – Sn-1 – 2, 2Tn = Sn+1 – Sn – 2. Thay vào (2), ta được Sn+1 – Sn – 2 = 2Sn-1+ Sn – Sn-1 – 2 + 2 Sn+1 = 2Sn + Sn-1 + 2 Từ đây dễ dàng tìm được 2 2)21()21( 11 −−++ = ++ nn nS Bài 24. Tìm số tất cả các bộ n số (x1, x2, …, xn) sao cho (i) xi = ± 1 với i = 1, 2, …, n. (ii) 0 ≤ x1 + x2 + … + xr < 4 với r = 1, 2, …, n-1 ; (iii) x1 + x2 + … + xn = 4. Bài 25. Trong một nhóm gồm 2n+1 người với mỗi n người tồn tại một người khác n người này quen với tất cả họ. Chứng minh rằng trong nhóm người này có 1 người quen với tất cả mọi người. HD: Ta sẽ chứng minh trong nhóm người này có một nhóm n+1 người đôi một quen nhau (gọi là nhóm A). Từ đó, xét n người còn lại. Theo giả thiết, có 1 người trong nhóm A quen với n người này. Suy ra người này quen với tất cả mọi người. Để chứng minh khẳng định về sự tồn tại của nhóm A, ta giả sử k là kích thước (số người) lớn nhất của một nhóm người đôi một quen nhau. Ta cần chứng minh k ≥ n+1. Giả sử ngược lại k ≤ n. Theo giả thiết,tồn tại 1 người trong số những người còn lại quen với k người này. Bổ sung người này vào nhóm, ta được nhóm gồm k+1 người đôi một quen nhau. Mâu thuẫn với điều giả sử k lớn nhất. Bài 26. Trong một đa giác lồi có chứa không ít hơn m2 +1 điểm nguyên. Chứng minh rằng trong đa giác lồi này tìm được m+1 điểm nguyên cùng nằm trên một đường thẳng. Bài 27. Chứng minh rằng trong 9 người bất kỳ, hoặc có 3 người đôi một quen nhau, hoặc có 4 người đôi một không quen nhau. Bài 28. Chọn ra 69 số nguyên dương từ tập hợp E = {1, 2, …, 100}. Chứng minh rằng tồn tại 4 số a < b < c < d trong 4 số được chọn sao cho a + b + c = d. Kết luận bài toán còn đúng không nếu ta thay 69 bằng 68? HD: Giả sử các số đó là a1 < a2 < … < a69. Xét 2 tập hợp A = {a2+a3, a2+a4, …., a2+a69}, B = {a3 – a1, …, a69 – a1} thì | A | = | B | = 67. Do a69 – a2 ≥ 67 nên ta suy ra a2 ≤ a69 – 67 ≤ 100 – 67 = 33. Suy ra a2 + a69 ≤ 133. Ngoài ra a3 – a1 ≥ 2. Do đây là các số lớn nhất của cả A và B nên ta có A, B ⊂ {2, 3, …., 133} = C. Vì | C | = 132 < 134 = |A| + |B| nên từ đây ta suy ra A ∩ B ≠ ∅. Suy ra tồn tại i, j ≥ 3 sao cho a2 + ai = aj – a1. Rõ ràng i < j và như thế ta có a1 + a2 + ai = aj. Dựa vào cách giải trên, có thể chỉ ra 68 số mà trong đó không tìm được 4 số thỏa mãn đều bài, đó là: 39
  • 40. 33, 34, …, 100. Do tổng ba số nhỏ nhất trong các số này đã là 33 + 34 + 35 = 102 > 100 là số lớn nhất nên không tồn tại ba số a, b, c, d sao cho a + b + c = d. Bài 29. Trong một nhóm n người có 3 người đôi một quen nhau và mỗi một người này quen nhiều hơn 1 nửa số người trong nhóm. Tìm số ít nhất có thể số bộ ba người đôi một quen nhau. Bài 30. Các số 1, 2, …, n được viết trên bảng. Mỗi một phút, một học sinh lên bảng, chọn hai số x và y, xóa chúng đi và viết lên bảng số 2x + 2y. Quá trình này tiếp diễn cho đến khi trên bảng chỉ còn lại một số. Chứng minh rằng số này không nhỏ hơn 3 4 9 n . Bài 31. Cho tập hợp A gồm các số nguyên dương có tính chất sau (i) min A = 1, max A = 100. (ii) Với mọi a thuộc A, a > 1 đều tồn tại b, c thuộc A (b không nhất thiết khác c) sao cho a = b + c. Tìm GTNN của | A |. Bài 32. Trong hệ thống các tuyến xe buýt của thành phố, hai tuyến bất kỳ có chung đúng một bến, và mỗi tuyến có ít nhất 4 bến. Chứng minh rằng có thể phân các bến thành hai nhóm sao cho mỗi một tuyến xe buýt đều có bến thuộc cả hai nhóm. Bài 33. Trong một giải cờ vua có 40 kỳ thủ. Có tổng cộng 80 ván đã được đấu, và hai kỳ thủ bất kỳ đấu với nhau nhiều nhất một lần. Tìm số nguyên dương n lớn nhất sao cho trong mọi trường hợp, ta đều tìm được n kỳ thủ chưa hề đấu với nhau. Bài 34. Trò chơi lô-tô ở Lotoland được tổ chức như sau: Người chơi chọn 6 số khác nhau từ các số 1, 2, 3, …, 36. Sau đó người ta sẽ bốc ra ngẫu nhiên 6 số từ các số 1, 2, 3, …, 36. Vé nào không chứa bất cứ số nào trong 6 số vừa bốc sẽ là vé thắng giải. Chứng minh rằng tồn tại một cách mua 9 vé để có thể đảm bảo luôn có ít nhất một vé thắng nhưng 8 vé thì nói chung không đủ để đảm bảo điều này. Bài 35. Bàn cờ 12 x 12 được tô màu đen trắng như bình thường. Mỗi một lần thực hiện ta có thể sơn lại hai ô cạnh nhau theo quy tắc: đen thành xanh, xanh thành trắng, trắng thành đen. Hỏi ta cần phải dùng tối thiểu bao nhiêu bước để sơn bàn cờ thành màu đen trắng ngược với bàn cờ ban đầu? 40
  • 41. C. KẾT QUẢ NGHIÊN CỨU Qua nghiên cứu giai đoạn 2 của chuyên đề tôi đã thu được một hệ thống các bài tập trên và có được sự sáng tạo khi vận dụng nó trong chứng minh và khai thác các bài toán tổ hợp. Từ kết quả nghiên cứu đó được sự nhất trí Ban giám hiệu, tổ khoa học tự nhiên. Chuyên đề nghiên cứu của tôi đã được thông qua tổ và áp dụng tại trường. *) Giai đoạn 3: Luyện đề về dạng bài tổ hợp tổng hợp. Củng cố phương pháp làm bài +) Mục đích: Nhằm nâng cao hiệu quả bồi dưỡng học sinh giỏi và bồi dưỡng học sinh thi vào trung học phổ thông chuyên. Trao đổi kinh nghiệm, lấy ý kiến đóng góp từ các đồng nghiệp để bổ sung và hoàn thiện và phát triển chuyên đề. +) Thời gian: Từ tháng 2 năm 2013 đến tháng 4 năm 2013 +) Cách tiến hành: Thông qua sinh hoạt chuyên môn báo cáo trước tổ, cụm Trao đổi thảo luận và lấy ý kiến đóng góp của các đồng nghiệp Áp dụng trực tiếp vào công tác bồi bưỡng học sinh giỏi. +) Kết quả giai đoạn 3 Sau khi thông qua chuyên đề trước tổ tôi đã tiến hành xin ý kiến đóng góp và nhận xét của đồng nghiệp và nhận được 100% giáo viên tán thành cách nghiên cứu này Trong quá trình áp dụng chuyên đề tôi đã tiến hành khảo sát với đối tượng là 25 học sinh giỏi. Qua kết quả khảo sát đó tôi thu được số học sinh biết vận dụng nguyên lí Dirichlet và nguyên lí cực hạn phụ tăng lên rõ rệt, hơn nữa đã có em bước đầu sáng tạo và tổng quát hóa về phương pháp và dạng bài tổ hợp. Kết quả cụ thể là: Giỏi Khá Trung bình yếu 6 10 7 2 B3 +) Phân tích kết quả Từ kết quả điều tra trên tôi có thể khẳng định rằng: Qua chuyên đề này thì các em học sinh đã biết cách trình bày, suy luận, khái quát hóa cho loại bài toán tổ hợp. Học sinh được phát triển tư duy và kỹ năng sáng tạo toán học. Học sinh tự tin hơn khi gặp các bài toán hay và khó về toán rời rạc. Hình thành phương pháp tìm lời giải bài toán, tư duy linh hoạt, phương pháp học toán một cách sáng tạo. Giúp cho học sinh khá giỏi không những hình thành kỹ năng giải toán mà còn giúp các em rèn luyện các thao tác tư duy: Phân tích, tổng hợp, khái quát hoá, đặc biệt 41
  • 42. hoá. Hình thành ở các em cách học sáng tạo, qua đó giúp các em đã có phương pháp tự học, tự nghiên cứu và phát triển khả năng tìm tòi, sáng tạo cho học sinh Hơn nữa mỗi giáo viên thấy được rằng chỉ có lao động nghiêm túc, khoa học mới có được hiệu quả cao. Từ đó nâng cao trình được trình độ chuyên môn và nâng cao năng lực sư phạm của mình trong quá trình giảng dạy đặc biệt là trong công tác bồi dưỡng học sinh giỏi. Tóm lại, với phương pháp nghiên cứu này giúp người thầy nâng cao được tay nghề của mình, xây dựng được hệ thống kiến thức cần có để định hướng cho học sinh trong quá trình học. Song quan trọng hơn là gây hứng thú học tập bộ môn cho học sinh, giúp các em có phương pháp học tập môn Toán một cách có hiệu quả. 42
  • 43. D. KẾT LUẬN Như vậy xuất phát từ nguyên lí Dirichlet được phát biểu thật đơn giản, bằng những thao tác tư duy, sự linh hoạt, sáng tạo của mình. Chúng ta sẽ xây dựng phương pháp và chứng minh được một số bài toán tổ hợp hay và khó. Thông qua chuyên đề này không chỉ cung cấp cho học sinh một hệ thống các dạng bài mà còn hình thành cho học kỹ năng phân tích bài toán để từ đó xây dựng và chứng minh các bài toán tổ hợp tổng quát để áp dụng nhằm phát triển năng lực tư duy sáng tạo toán học của học sinh, nhất là những học sinh khá giỏi. Về lý luận: Rèn luyện khả năng tư duy sáng tạo, kỹ năng phân tích tổng hợp, tính cẩn thận chính xác, tính kiên trì và sự linh hoạt trong vận dụng kiến thức đã học và viải toán cho học sinh. Giúp các em có hứng thú học tập, ham mê học Toán và đặc biệt là phát huy năng lực tư duy sáng tạo học sinh khi gặp các dạng toán khó. Về thực tiễn: Giúp học sinh vận dụng tốt nguyên lí Dirichlet và nguyên lí cực hạn trong việc giải các bài toán rời rạc nói chung và hình học tổ hợp nói riêng. Từ chỗ rất lúng túng khi gặp các bài toán tổ hợp thì nay phần lớn các em đã tự tin hơn, biết vận dụng những kỹ năng được bồi dưỡng để giải thành thạo các bài tập chứng minh mang tính phức tạp. Trong khi viết chuyên đề “Sử dụng nguyên lí Dirichlet và nguyên lí cực hạn trong các bài toán tổ hợp” tôi không tránh khỏi những thiếu sót cũng như nội dung chuyên đề chưa thực sự phong phú. Rất mong các bạn đồng nghiệp và các em học sinh đóng góp thêm các ý kiến để chuyên đề hoàn thiện và có hiệu quả hơn. Tôi xin chân thành cám ơn. Vĩnh Yên, ngày 10 tháng 1 năm 2014 Người viết chuyên đề Trần Thị Phi Nga 43
  • 44. CÁC TÀI LIỆU THAM KHẢO. [1] Báo toán tuổi thơ, toán học và tuổi trẻ [2] Phan Huy Khải (năm 2007) Các bài toán hình học tổ hợp. [3] Trịnh Đình Long (năm 2006) Bài giảng cho học sinh chuyên toán- Trường ĐHKHTN TP Hồ Chí Minh: Nguyên lí Dirichlet và các bài toán số học. [4] Vũ Hữu Bình (năm 1998): Phương pháp giảng dạy môn toán - NXB GD [5] Website http://baigiang.violet.vn/ [6] Website http://math.net.vn/ 44